Está en la página 1de 102

ONLY SEE

2019 - Test 9- History &


HIGHLIGHTED Exam Title :
Cu...
PARTS Email : vikasutsav@hotmail.com
Contact :

Note: If the exam is multi-lingual i.e. English and Hindi. Hindi solutions will be
after the completion of English solutions.
QUESTION 1. MTU4MzMyK1Zpa2FzIFV0c2F2K3Zpa2FzdXRzYXZAaG90bWFpbC5jb20rUVVFU1RJT04
gMA==
Consider the following statements regarding the tribal revolts started in India during the
British rule.

1. The violence was directed towards the outsider money-lenders and traders who were seen as
extensions of the colonial government.

2. They were the most frequent, militant and violent of all movements

3. Tribal identity or ethnic ties became the quintessential factor for their solidarity

Choose the correct code

a) 1 and 2 only
b) 2 and 3 only
c) 1 and 3 only
d) All of the above
Correct Answer: D
Your Answer: A
Explanation

Solution (d)

Tribal movements under British rule were the most frequent, militant and violent of all
movements.

There were some common characteristics of the tribal uprisings even though they were
separated from one another in time and space.

· Tribal identity or ethnic ties lay behind the solidarity shown by these groups. Not all outsiders
were, however, seen as enemies: the poor who lived by their manual labour or profession and
had a socially/economically supportive role in the village were left alone; the violence was
directed towards the money-lenders and traders who were seen as extensions of the colonial
government.

· A common cause was the resentment against the imposition of laws by the ‘foreign
government’ that was seen as an effort at destroying the tribals’ traditional socioeconomic
framework.

· Many uprisings were led by messiah-like figures who encouraged their people to revolt and
who held out the promise that they could end their suffering brought about by the outsiders.

· The tribal uprisings were doomed from the beginning, given the outdated arms they fought
with as against the modern weapons and techniques used by their opponents.

QUESTION 2. MTU4MzMyK1Zpa2FzIFV0c2F2K3Zpa2FzdXRzYXZAaG90bWFpbC5jb20rUVVFU1RJT04
gMQ==
Which of the following are correctly matched?

IASbaba
Score:
Web: http://ilp.iasbaba.com/
45.00 /
Email: ilp@iasbaba.com
Page 1 150
2019 - Test 9- History &
Exam Title :
Cu...
Email : vikasutsav@hotmail.com
Contact :

Revolts Place

1. Pahariyas ’ rebellion A) Rajmahal hills

2. Koya revolts B) Chotanagpur plateau

3. Kol mutiny C) eastern Godavari track

Choose the correct code

a) 1 only
b) 1 and 2 only
c) 1 and 3 only
d) 2 and 3 only
Correct Answer: A
Your Answer: Unanswered
Explanation

Solution (a)

The British expansion on their territory led to an uprising by the martial Pahariyas of the Raj
Mahal Hills in 1778. The British were forced to usher in peace by declaring their territory as
damnikol area.

The Kols, along with other tribes, are inhabitants of Chhotanagpur . This covered
Ranchi, Singhbhum, Hazaribagh, Palamau and the western parts of Manbhum. The trouble in
1831 started with large-scale transfers of land from Kol headmen to outsiders like Hindu, Sikh
and Muslim farmers and money-lenders who were oppressive and demanded heavy taxes.
Besides, the British judicial and revenue policies badly affected the traditional social conditions
of the Kols. The Kols resented this and in 1831, under the leadership of Buddho Bhagat, the Kol
rebels killed or burnt about a thousand outsiders.

The Koyas of the eastern Godavari track (modern Andhra), joined by Khonda Sara chiefs,
rebelled in 1803, 1840, 1845, 1858, 1861 and 1862. They rose once again in 1879-80 under
Tomma Sora. Their complaints were oppression by police and moneylenders, new regulations
and denial of their customary rights over forest areas. After the death of Tomma Sora, another
rebellion was organised in 1886 by Raja Anantayyar.
Sora/ Sara with
Koya/kaya

QUESTION 3. MTU4MzMyK1Zpa2FzIFV0c2F2K3Zpa2FzdXRzYXZAaG90bWFpbC5jb20rUVVFU1RJT04
gMg==
Arrange the following movements in the chronological order

1. Sanyasi revolt

2. Paika rebellion

3. Ahom revolt

4. Kuka movement

Choose the correct code

IASbaba
Score:
Web: http://ilp.iasbaba.com/
45.00 /
Email: ilp@iasbaba.com
Page 2 150
2019 - Test 9- History &
Exam Title :
Cu...
Email : vikasutsav@hotmail.com
Contact :

a) 1-2-3-4
b) 2-1-3-4
c) 1-3-2-4
d) 1-3-4-2
Correct Answer: A
Your Answer: Unanswered
Explanation

Solution (a)

Sanyasi Revolt (1763-1800) —Bihar and Bengal; Manju Shah, Musa Shah, Bhawani Pathak
and Debi Chaudhurani were some important leaders.

Paika Rebellion (1817)— Orissa; Bakshi Jagabandhu Bidyadhar, Mukunda Deva and
Dinabandhu Santra were important leaders. Paika Rebellion (1817)—Orissa; Bakshi Jagabandhu
Bidyadhar, Mukunda Deva and Dinabandhu Santra were important leaders.

Ahom Revolt (1828)— Assam; led by Gomdhar Konwar and Maharaja Purandhar Singh.
Narendra Gadadhar Singh and Kumar Rupchand were other leaders.

Kuka Movement (1840-1872)— Punjab; A religious movement started by Bhagat Jawahar


Mal transformed into political one. Ram Singh, a noted leader, deported to Rangoon.

Oye Kuki revolt


kardiye tune?
QUESTION 4. MTU4MzMyK1Zpa2FzIFV0c2F2K3Zpa2FzdXRzYXZAaG90bWFpbC5jb20rUVVFU1RJT04
gMw==
Which of the following are incorrectly matched?

Centres of revolt leaders

1. Kanpur A) Nana Saheb

2. Lucknow B) Begum Hazrat Mahal

3. Bihar C) Maulvi Ahmadullah

4. Faizabad D) Kunwar Singh

Choose the incorrect code

a) 1 and 3 only
b) 2 and 4 only
c) 3 and 4 only
d) 1 and 4 only
Correct Answer: C
Your Answer: A
Explanation

Solution (c)

Centres of Revolt and Leaders

· Delhi - General Bakht Khan

IASbaba
Score:
Web: http://ilp.iasbaba.com/
45.00 /
Email: ilp@iasbaba.com
Page 3 150
2019 - Test 9- History &
Exam Title :
Cu...
Email : vikasutsav@hotmail.com
Contact :

· Kanpur - Nana Saheb

· Lucknow - Begum Hazrat Mahal

· Bareilly - Khan Bahadur

· Bihar - Kunwar Singh

· Faizabad - Maulvi Ahmadullah

· Jhansi - Rani Laxmibai

· Baghpat - Shah Mal

QUESTION 5. MTU4MzMyK1Zpa2FzIFV0c2F2K3Zpa2FzdXRzYXZAaG90bWFpbC5jb20rUVVFU1RJT04
gNA==
Which of the following are said to be the causes for the failure of revolt of 1857?

1. Lack of a coherent ideology and a political perspective.

2. Lack of Hindu- Muslim unity was one of the main factors for the failure.

3. Crucial support of certain sections of Indian public to British authorities.

Choose the correct code

a) 1 and 2 only
b) 1 and 3 only
c) 2 and 3 only
d) All of the above
Correct Answer: B
Your Answer: B
Explanation

Solution (b)

Causes for the failure of the revolt

· Limited territorial and social base.

· Crucial support of certain sections of Indian public to British authorities.

· Lack of resources as compared to those of the British.

· Lack of coordination and a central leadership.

· Lack of a coherent ideology and a political perspective.

During the entire revolt, there was complete cooperation between Hindus and Muslims
at all levels —people, soldiers, leaders. All rebels acknowledged Bahadur Shah Zafar, a
Muslim, as the emperor and the first impulse of the Hindu sepoys at Meerut was to march to
Delhi, the Mughal imperial capital.

IASbaba
Score:
Web: http://ilp.iasbaba.com/
45.00 /
Email: ilp@iasbaba.com
Page 4 150
2019 - Test 9- History &
Exam Title :
Cu...
Email : vikasutsav@hotmail.com
Contact :

QUESTION 6. MTU4MzMyK1Zpa2FzIFV0c2F2K3Zpa2FzdXRzYXZAaG90bWFpbC5jb20rUVVFU1RJT04
gNQ==

Which of the following factors led to the social reform movements in the 19 th century India?

1. Indian society in the nineteenth century was caught in a vicious web created by religious
superstitions and social obscurantism.

2. Sheer misery and humiliation imparted on women through various ill practices like sati, child
marriage etc

3. The impact of modern Western culture and consciousness of defeat by a foreign power gave
birth to a new awakening.

Choose the correct code

a) 1 and 2 only
b) 2 and 3 only
c) 1 and 3 only
d) All of the above
Correct Answer: D
Your Answer: D
Explanation

Solution (d)

Factors which gave Rise to Reform Movements


A policy of opposition to enlightenment or the
· Presence of colonial government on Indian soil. spread of knowledge
· Various ills plaguing Indian society—obscurantism, superstition, polytheism, idolatry, degraded
position of women, exploitative caste hierarchy.

· Spread of education and increased awareness of the world.

· Impact of modern Western culture and consciousness of defeat by a foreign power.

· Rising tide of nationalism and democracy during the late 19th century.

QUESTION 7. MTU4MzMyK1Zpa2FzIFV0c2F2K3Zpa2FzdXRzYXZAaG90bWFpbC5jb20rUVVFU1RJT04
gNg==
Who among the following founded Bharat Mahila Parishad ?

a) Sarala devi Chaudhurani


b) Ramabai Ranade
c) Pandita Ramabai Sarasvati
d) Tarabai Premchand
Correct Answer: B
Your Answer: B
Explanation

IASbaba
Score:
Web: http://ilp.iasbaba.com/
45.00 /
Email: ilp@iasbaba.com
Page 5 150
2019 - Test 9- History &
Exam Title :
Cu...
Email : vikasutsav@hotmail.com
Contact :

Solution (b)

Ramabai Ranade founded the Ladies Social Conference (Bharat Mahila Parishad), under the
parent organisation National Social Conference, in 1904 in Bombay.

Add these in the list of Organistions before Congress, or make a new List

QUESTION 8. MTU4MzMyK1Zpa2FzIFV0c2F2K3Zpa2FzdXRzYXZAaG90bWFpbC5jb20rUVVFU1RJT04
gNw==
Consider the following statements regarding the nationalists in the moderate phase of Indian
history.

1. The moderate phase of the national movement had a wide social base and the masses played
an active role.

2. They trained people in political work and popularised modern ideas.

3. They gave a call for separation of judicial from executive functions.

Choose the correct code

a) 1 and 2 only
b) 2 and 3 only
c) 1 and 3 only
d) 2 only
Correct Answer: B
Your Answer: B
Explanation

Solution (b)

The early nationalists did a great deal to awaken the national sentiment, even though they
could not draw the masses to them. The moderate phase of the national movement had a
narrow social base and the masses played a passive role.

· They represented the most progressive forces of the time.

· They trained people in political work and popularised modern ideas.

· They exposed the basically exploitative character of colonial rule, thus undermining its moral
foundations.

· Their political work was based on hard realities, and not on shallow sentiments, religion, etc

· They gave a call for separation of judicial from executive functions.

QUESTION 9. MTU4MzMyK1Zpa2FzIFV0c2F2K3Zpa2FzdXRzYXZAaG90bWFpbC5jb20rUVVFU1RJT04
gOA==
Which of the following factors were responsible for the growth of modern nationalism?

1. The growth of press helped in spreading modern ideas of self-government, democracy, civil
rights and industrialisation.

IASbaba
Score:
Web: http://ilp.iasbaba.com/
45.00 /
Email: ilp@iasbaba.com
Page 6 150
2019 - Test 9- History &
Exam Title :
Cu...
Email : vikasutsav@hotmail.com
Contact :

2. Middle class intelligentsia provided effective leadership to the Indian political associations.

3. Socio-religious reform movements served a great deal in bringing the different sections
together.

Choose the correct code

a) 1 and 2 only
b) 2 and 3 only
c) 1 and 3 only
d) All of the above
Correct Answer: D
Your Answer: D
Explanation

Solution (d)

Factors responsible for the growth of Modern Nationalism

· Understanding of contradictions in Indian and colonial interests

· Political, administrative and economic unification of the country

· Western thought and education

· Role of press and literature

· Rediscovery of India’s past—historical researches

· Rise of middle class intelligentsia

· Impact of contemporary movements worldwide

· Reactionary policies and racial arrogance of rulers

QUESTION 10. MTU4MzMyK1Zpa2FzIFV0c2F2K3Zpa2FzdXRzYXZAaG90bWFpbC5jb20rUVVFU1RJT0


4gOQ==
Consider the following organisations

1. All India scheduled castes federation

2. All India depressed classes league

3. All India depressed classes association

4. Bahishkrit Hitakarini Sabha

Dr. B.R. Ambedkar was associated with which of the above organisations?

a) 1,2 and 3 only


b) 1,2 and 4 only
c) 1 and 4 only

IASbaba
Score:
Web: http://ilp.iasbaba.com/
45.00 /
Email: ilp@iasbaba.com
Page 7 150
2019 - Test 9- History &
Exam Title :
Cu...
Email : vikasutsav@hotmail.com
Contact :

d) 1,3 and 4 only


Correct Answer: C
Your Answer: C
Explanation

Solution (c)

Babasaheb Ambedkar, who had experienced the worst form of casteist discrimination during his
childhood, fought against upper caste tyranny throughout his life. He organised the All India
Scheduled Castes Federation, while several other leaders of the depressed classes founded the
All India Depressed Classes Association.

Dr Ambedkar established the Bahishkrit Hitakarini Sabha in 1924 to highlight the difficulties
and grievances of the Dalits before the government. Its motto was: ‘Educate, Agitate and
Organise’

All India depressed classes league was founded by Babu Jagjivan Ram.

QUESTION 11. MTU4MzMyK1Zpa2FzIFV0c2F2K3Zpa2FzdXRzYXZAaG90bWFpbC5jb20rUVVFU1RJT0


4gMTA=
Who is known as the father of Indian renaissance?

a) Raja Ram Mohan Roy 1777- 1833


b) Dayanand saraswathi 1824-83
c) Jyothibha Phule 1827-90
d) Ishwar Chandra Vidyanagar 1820-1891
Correct Answer: A
Your Answer: D
Explanation

Solution (a)

Raja Ram Mohan Roy (1772-1833), often called the father of Indian Renaissance and the maker
of Modern India, was a man of versatile genius.

Ram Mohan Roy believed in the modern scientific approach and principles of human dignity and
social equality. He put his faith in monotheism. He wrote Gift to Monotheists (1809) and
translated into Bengali the Vedas and the five Upanishads to prove his conviction that ancient
Hindu texts support monotheism. 158332

QUESTION 12. MTU4MzMyK1Zpa2FzIFV0c2F2K3Zpa2FzdXRzYXZAaG90bWFpbC5jb20rUVVFU1RJT0


4gMTE=
Which of the following were the features of Brahmo Samaj ?

1. It took a definite stand on the doctrine of karma and transmigration of soul.

2. It upheld that only Hindu scriptures could enjoy the status of ultimate authority transcending
human reason and conscience.

3. It denied the need for a priestly class for interpreting the religious texts.

IASbaba
Score:
Web: http://ilp.iasbaba.com/
45.00 /
Email: ilp@iasbaba.com
Page 8 150
2019 - Test 9- History &
Exam Title :
Cu...
Email : vikasutsav@hotmail.com
Contact :

Choose the correct code

a) 1 and 2 only
b) 1 and 3 only
c) 2 and 3 only
d) 3 only
Correct Answer: D
Your Answer: B
Explanation

Solution (d)

Raja Ram Mohan Roy founded the Brahmo Sabha in August 1828; it was later renamed Brahmo
Samaj. Through the Sabha he wanted to institutionalise his ideas and mission.

The Samaj was committed to “the worship and adoration of the Eternal, Unsearchable,
Immutable Being who is the Author and Preserver of the Universe”

The features of Brahmo Samaj may be summed thus—

1) It denounced polytheism and idol worship;

2) It discarded faith in divine avatars (incarnations);

3) It denied that any scripture could enjoy the status of ultimate authority transcending human
reason and conscience;

4) It took no definite stand on the doctrine of karma and transmigration of soul and left it to
individual Brahmos to believe either way;

5) It criticised the caste system

QUESTION 13. MTU4MzMyK1Zpa2FzIFV0c2F2K3Zpa2FzdXRzYXZAaG90bWFpbC5jb20rUVVFU1RJT0


4gMTI=
Who among the following gave a proposition ‘rationalism is our only preceptor’?

a) Akshay Kumar Dutt


b) Raj Ram Mohan Roy
c) Swami Vivekananda
d) Syed Ahmad khan
Correct Answer: A
Your Answer: B
Explanation

Solution (a)

Akshay Kumar Dutt declared that “rationalism is our only preceptor”, while doing so he held
that all natural and social phenomena could be analysed and understood by purely mechanical
processes. They thus used a rational approach to study tradition; they evaluated the
contemporary socio religious practices from the standpoint of social utility and to replace faith
with rationality.

IASbaba
Score:
Web: http://ilp.iasbaba.com/
45.00 /
Email: ilp@iasbaba.com
Page 9 150
2019 - Test 9- History &
Exam Title :
Cu...
Email : vikasutsav@hotmail.com
Contact :

Akshay Kumar Dutt, for instance, brought medical opinion to support his views against child
marriage. Reference to the past was to be used only as an aid and an instrument. Neither a
revival of the past nor a total break with tradition was envisaged.

QUESTION 14. MTU4MzMyK1Zpa2FzIFV0c2F2K3Zpa2FzdXRzYXZAaG90bWFpbC5jb20rUVVFU1RJT0


4gMTM=
Who among the following headed Tatvabodhini Sabha?

a) Devendranath Tagore
b) Rabindranath Tagore
c) M.G. Ranade
d) Keshab Chandra Sen
Correct Answer: A
Your Answer: A
Explanation

Solution (a)

Tatvabodhini Sabha

The main objective of the Sabhā was to promote a more rational and humanist form of
Hinduism based on the Vedānta, the Upanishads that form the last part of the Vedās. With
increasing missionary presence in Calcutta tending to view the Classical branch of 'Avaida'
Vedānta as amoral and renunciatory, the Tattwabodhinī Sabhā aimed to shield themselves and
their reformed faith from criticism by distancing themselves from this 'outdated' version.

It was founded by Debendranath Tagore in 1839 as a splinter group of Brahmo Samaj. In 1859,
the Tattwabodhinī Sabhā were dissolved back into the Brāhmo Samāj.

QUESTION 15. MTU4MzMyK1Zpa2FzIFV0c2F2K3Zpa2FzdXRzYXZAaG90bWFpbC5jb20rUVVFU1RJT0


4gMTQ=
Consider the following statements regarding social reform organisations.

1. The Prarthana Samaj relied on education and persuasion and not on confrontation with
Hindu orthodoxy.

2. The young Bengal movement drew inspiration from the great French Revolution.

3. Ishwar Chandra Vidyasagar introduced Western thought in Sanskrit College to break the self-
imposed isolation of Sanskritic learning.

Choose the incorrect code

a) 1 and 2 only
b) 2 and 3 only
c) 1 and 3 only
d) None of the above
Correct Answer: D

IASbaba
Score:
Web: http://ilp.iasbaba.com/
45.00 /
Email: ilp@iasbaba.com
Page 10 150
2019 - Test 9- History &
Exam Title :
Cu...
Email : vikasutsav@hotmail.com
Contact :
Your Answer: B
Explanation

Solution (d)

In 1867, Keshab Chandra Sen helped Atmaram Pandurang found the Prarthana Samaj in
Bombay. The Prarthana Sabha was very attached to the bhakti cult of Maharashtra. The Samaj
relied on education and persuasion and not on confrontation with Hindu orthodoxy. There was a
four-point social agenda also: (i) disapproval of caste system, (ii) women’s education, (iii) widow
remarriage, and (iv) raising the age of marriage for both males and females.

During the late 1820s and early 1830s, there emerged a radical, intellectual trend among the
youth in Bengal, which came to be known as the ‘Young Bengal Movement’. Drawing inspiration
from the great French Revolution, Derozio inspired his pupils to think freely and rationally,
question all authority, love liberty, equality and freedom, and oppose decadent customs and
traditions.

The great scholar and reformer, Vidyasagar’s ideas were a happy blend of Indian and Western
thought. He was determined to break the priestly monopoly of scriptural knowledge, and for
this he opened the Sanskrit College to non-Brahmins. He introduced Western thought in
Sanskrit College to break the self-imposed isolation of Sanskritic learning.

QUESTION 16. MTU4MzMyK1Zpa2FzIFV0c2F2K3Zpa2FzdXRzYXZAaG90bWFpbC5jb20rUVVFU1RJT0


4gMTU=
Who is the author of the work Gulamgiri ?

a) Jyothibha Phule
b) Gopalhari Deshmukh
c) Gopal Ganesh Agarkar
d) Gopal Krishna Gokhale
Correct Answer: A
Your Answer: Unanswered
Explanation

Solution (a)

Jyotiba Phule (1827-1890), born in Satara, Maharashtra, belonged to the Mali (gardener)
community and organised a powerful movement against upper caste domination and
brahminical supremacy. Phule’s works, Sarvajanik Satyadharma and Gulamgiri, became sources
of inspiration for the common masses. Phule used the symbol of Rajah Bali as opposed to the
Brahmins’ symbol of Rama.

QUESTION 17. MTU4MzMyK1Zpa2FzIFV0c2F2K3Zpa2FzdXRzYXZAaG90bWFpbC5jb20rUVVFU1RJT0


4gMTY=
Which of the following are correctly matched?

Founder Organisation

1. Gopal Krishna Gokhale A) Servants of India society

IASbaba
Score:
Web: http://ilp.iasbaba.com/
45.00 /
Email: ilp@iasbaba.com
Page 11 150
2019 - Test 9- History &
Exam Title :
Cu...
Email : vikasutsav@hotmail.com
Contact :

2. Narayan Malhar Joshi B) Social service league

3. Swami Vivekananda C) Ramakrishna mission

Choose the correct code

a) 1 and 2 only
b) 1 and 3 only
c) 2 and 3 only
d) All of the above
Correct Answer: D
Your Answer: B
Explanation

Solution (d)

Self-explanatory

QUESTION 18. MTU4MzMyK1Zpa2FzIFV0c2F2K3Zpa2FzdXRzYXZAaG90bWFpbC5jb20rUVVFU1RJT0


4gMTc=
Who among the following said Hindu race as ‘the children of children’?

a) Dayananda saraswati
b) Swami Vivekananda
c) Ishwar Chandra Vidyasagar
d) Lala Lajpath Rai
Correct Answer: A
Your Answer: Unanswered
Explanation

Solution (a)

Swami Dayananda once lamented the Hindu race as “the children of children”.

QUESTION 19. MTU4MzMyK1Zpa2FzIFV0c2F2K3Zpa2FzdXRzYXZAaG90bWFpbC5jb20rUVVFU1RJT0


4gMTg=
Which of the following started shuddhi movement to reconvert to the Hindu fold the converts to
Christianity and Islam?

a) Arya Samaj
b) Prarthana Samaj
c) Dharma Sabha
d) Self-respect movement
Correct Answer: A
Your Answer: A
Explanation

IASbaba
Score:
Web: http://ilp.iasbaba.com/
45.00 /
Email: ilp@iasbaba.com
Page 12 150
2019 - Test 9- History &
Exam Title :
Cu...
Email : vikasutsav@hotmail.com
Contact :

Solution (a)

The Arya Samaj started the shuddhi (purification) movement to reconvert to the Hindu
fold the converts to Christianity and Islam. This led to increasing communalisation of social life
during the 1920s and later snowballed into communal political consciousness.

QUESTION 20. MTU4MzMyK1Zpa2FzIFV0c2F2K3Zpa2FzdXRzYXZAaG90bWFpbC5jb20rUVVFU1RJT0


4gMTk=
Who launched the ‘Pledge Movement’ to inspire people to take a pledge against child marriage?

a) Indian social conference


b) Brahmo samaj
c) Theosophical society of India
d) Prarthana Samaj
Correct Answer: A
Your Answer: B
Explanation

Solution (a)

M.G. Ranade and Raghunath Rao founded the Indian Social Conference which met annually
from its first session in Madras in 1887 at the same time and venue as the Indian National
Congress. It focussed attention on the social issues of importance; it could be called the social
reform cell of the Indian National Congress , in fact. The conference advocated inter-caste
marriages, opposed polygamy and kulinism. It launched the ‘Pledge Movement’ to inspire
people to take a pledge against child marriage.

QUESTION 21. MTU4MzMyK1Zpa2FzIFV0c2F2K3Zpa2FzdXRzYXZAaG90bWFpbC5jb20rUVVFU1RJT0


4gMjA=
Which of the following statements regarding The Ilbert Bill is correct?

a) European nationals could be judged by Indian Magistrates.


b) Imposition of restrictions on newspapers and magazines published in Indian languages.
c) Imposition restrictions the Indians of certain to carry arms by Indians.
d) Imposition of additional tax on the land revenue during Famine.
Correct Answer: A
Your Answer: A
Explanation

Solution (a)

Illbert Bill

The 'Ilbert Bill' was a bill introduced in 1883 under the reign of the Viceroy Lord Ripon (who
earned the epithet Ripon-The good) which was written by Sir C.P Ilbert (The law member of the
Viceroy's Council). According to the said Act, Indian judges could try a European accused.

IASbaba
Score:
Web: http://ilp.iasbaba.com/
45.00 /
Email: ilp@iasbaba.com
Page 13 150
2019 - Test 9- History &
Exam Title :
Cu...
Email : vikasutsav@hotmail.com
Contact :

This led to organised lobbying and opposition by all Europeans in India. And an amendment was
brought as a compromise. As per amendment- Indian judges could preside over cases involving
Europeans BUT the Europeans got right to demand trial by jury, where atleast half of the jury
would be White Europeans.

This completely defeated the purpose of the original Ilbert Bill. Though the educated Indians
had also protested against the amendments, but they realized that to pressurise the
government, all India organisation and more coordination amongst Indians to press for equal
treatment. This is why Ilbert Bill controversy is seen as an important precursor to the formation
of the INC.

QUESTION 22. MTU4MzMyK1Zpa2FzIFV0c2F2K3Zpa2FzdXRzYXZAaG90bWFpbC5jb20rUVVFU1RJT0


4gMjE=
Which of the following political associations were started in Bengal?

1. The Zamindari Association

2. The British Indian Association

3. Indian National Association

4. The Poona Sarvajanik Sabha

Choose the correct code

a) 1,2 and 3 only


b) 2,3 and 4 only
c) 1 and 3 only
d) 2 and 3 only
Correct Answer: A
Your Answer: A
Explanation

Solution (a)

The Poona Sarvajanik Sabha was founded in 1867 by Mahadeo Govind Ranade and others in
Bombay with the object of serving as a bridge between the government and the people.

QUESTION 23. MTU4MzMyK1Zpa2FzIFV0c2F2K3Zpa2FzdXRzYXZAaG90bWFpbC5jb20rUVVFU1RJT0


4gMjI=
Who among the following presided over the first session of the Indian national congress?

a) W.C. Bannerjee
b) Dadabhai Naoroji
c) A.O. Hume
d) Badruddhin Tyabji
Correct Answer: A
Your Answer: A

IASbaba
Score:
Web: http://ilp.iasbaba.com/
45.00 /
Email: ilp@iasbaba.com
Page 14 150
2019 - Test 9- History &
Exam Title :
Cu...
Email : vikasutsav@hotmail.com
Contact :
Explanation

Solution (a)

The first session of the Indian National Congress was attended by 72 delegates and presided
over by Womesh Chandra Bannerjee. Hereafter, the Congress met every year in December, in a
different part of the country each time.

QUESTION 24. MTU4MzMyK1Zpa2FzIFV0c2F2K3Zpa2FzdXRzYXZAaG90bWFpbC5jb20rUVVFU1RJT0


4gMjM=
In criticising the reforms brought by which of the following acts, did nationalists gave a call for
‘No taxation without representation’?

a) Indian councils act 1892


b) Indian councils act 1909
c) Indian councils act 1919
d) Government of India act 1935
Correct Answer: A
Your Answer: A
Explanation

Solution (a)

The early nationalists worked with the long-term objective of a democratic self-government.
Their demands for constitutional reforms were meant to have been conceded in 1892 in the
form of the Indian Councils Act.

These reforms were severely criticised at Congress sessions, where the nationalists made no
secret of their dissatisfaction with them.

Now, they demanded (i) a majority of elected Indians, and (ii) control over the budget, i.e., the
power to vote upon and amend the budget. They gave the slogan —“No taxation without
representation” . Gradually, the scope of constitutional demands was widened.

QUESTION 25. MTU4MzMyK1Zpa2FzIFV0c2F2K3Zpa2FzdXRzYXZAaG90bWFpbC5jb20rUVVFU1RJT0


4gMjQ=
Who among the following remarked congress as ‘a factory of sedition’?

a) Lord Dufferin
b) Lord Curzon
c) Lord Lansdowne
d) Lord Chelmsford
Correct Answer: A
Your Answer: A
Explanation

Solution (a)

IASbaba
Score:
Web: http://ilp.iasbaba.com/
45.00 /
Email: ilp@iasbaba.com
Page 15 150
2019 - Test 9- History &
Exam Title :
Cu...
Email : vikasutsav@hotmail.com
Contact :

When the Congress was becoming increasingly critical of the colonial rule, the government
resorted to open condemnation of the Congress, calling the nationalists “seditious Brahmins”,
“disloyal babus”, etc. Dufferin called the Congress “a factory of sedition”

QUESTION 26. MTU4MzMyK1Zpa2FzIFV0c2F2K3Zpa2FzdXRzYXZAaG90bWFpbC5jb20rUVVFU1RJT0


4gMjU=
Who of the following personalities gave the title ‘Raja’ to Rammohun Roy?

a) Akbar II
b) Farrukhsiyar
c) Siraj Ud Daula
d) Lord Bentick
Correct Answer: A
Your Answer: B
Explanation

Solution (a)

Akbar Shah II

By 1828, Raja Rammohun Roy had become a well-known figure in India. In 1830, he had gone
to England as an envoy of the Mughal Emperor , Akbar Shah II , who invested him with the
title of Raja to the court of King William IV.

QUESTION 27. MTU4MzMyK1Zpa2FzIFV0c2F2K3Zpa2FzdXRzYXZAaG90bWFpbC5jb20rUVVFU1RJT0


4gMjY=
Consider the following statements regarding the India Council.

1. It was constituted under the Government of India Act 1858.


2. It was headed by the Viceroy of India.
3. The Secretary of State of India was the Chairman of the council.
4. It had nominees of the Court of Directors.

Which of the above statements is/are correct?

a) 1 and 2 only
b) 2 and 3 only
c) 1 and 3 only
d) 2 and 4 only
Correct Answer: C
Your Answer: Unanswered
Explanation

Solution (c)

The India Council was constituted under the Government of India Act of 1858. It was headed by
the Secretary of States for India. Since the Court of Directors was dissolved by the act there
were no nominees of the Court of Directors.

IASbaba
Score:
Web: http://ilp.iasbaba.com/
45.00 /
Email: ilp@iasbaba.com
Page 16 150
2019 - Test 9- History &
Exam Title :
Cu...
Email : vikasutsav@hotmail.com
Contact :

QUESTION 28. MTU4MzMyK1Zpa2FzIFV0c2F2K3Zpa2FzdXRzYXZAaG90bWFpbC5jb20rUVVFU1RJT0


4gMjc=
Consider the following statements with reference to the Famine Commission of 1880.

1. It was appointed by Lord Rippon.


2. It was headed by Richard Starchey .
3. The First famine code was formulated based on the commission’s report.

Which of the above statements are correct?

a) 1 and 2 only
b) 2 and 3 only
c) 1 and 3 only
d) All of the above.
Correct Answer: B
Your Answer: Unanswered
Explanation

Solution (b)

In 1880 the Governor General of India Lord Lytton appointed a Famine commission headed by
Sir Richard Starchey. It recommended providing employment to the able bodied persons, and to
provide relief to the physically infirmed, It suggested that the famine relief should be primary
responsibility of the Provincial governments with the centre’s help wherever necessary.

Based on the recommendations of the committee, First Famine Code was formulated in 1883.

The second Famine Commission was appointed by Lord Curzon in 1901, after the draught of
1899-1900.

QUESTION 29. MTU4MzMyK1Zpa2FzIFV0c2F2K3Zpa2FzdXRzYXZAaG90bWFpbC5jb20rUVVFU1RJT0


4gMjg=
Which of the following pairs is matched correctly?

1. Mayo’s resolution on financial decentralization : 1870


2. The Royal Commission on Decentralization : 1905
3. Ripon’s Resolution on Decentralization : 1882

Select the code from following:

a) 1 and 2 only
b) 2 and 3 only
c) 1 and 3 only
d) None of the above
Correct Answer: C
Your Answer: Unanswered
Explanation

Solution (c)

IASbaba
Score:
Web: http://ilp.iasbaba.com/
45.00 /
Email: ilp@iasbaba.com
Page 17 150
2019 - Test 9- History &
Exam Title :
Cu...
Email : vikasutsav@hotmail.com
Contact :

Lord Ripon is known to have granted the Indians first taste of freedom by introducing the Local
Self Government in 1882. His scheme of local self-government developed the Municipal
institutions which had been growing up in the country ever since India was occupied by the
British Crown. He led a series of enactments in which larger powers of the Local self-
government were given to the rural and urban bodies and the elective people received some
wider rights. Lord Ripon is known as Father of Local Self Government in India.

Mayo's Resolution of 1870

The Indian Council of Act of 1861 introduced the policy of legislative devolution and Mayo's
Resolution of 1870 on financial decentralisation was its likely consequence. Administrative
suitability and financial severity instigated the Imperial Government to reassign to the
management of provincial governments specific departments of administration, which, along
with others, incorporated education, medical services and roads. The Indian Council of Act of
1861 introduced the policy of legislative devolution and Mayo's Resolution of 1870 on financial
decentralisation was its likely consequence. Administrative suitability and financial severity
instigated the Imperial Government to reassign to the management of provincial governments
specific departments of administration, which, along with others, incorporated education,
medical services and roads.

The Royal Commission upon Decentralisation was appointed in 1907 under the
chairmanship of Sir Henry William Primrose. It was a six-member body including the
Chairman, other five members being Frederic Lely, Steyning Edgerley, Romesh Chunder Dutt,
William Meyer and William Hichons.

The Royal Commission upon Decentralization in India was appointed on 12 September 1907 to
inquire into the relations now existing for financial and administrative purposes between the
Supreme Government (i.e., the Government of India) and the various Provincial Governments in
India, and between the Provincial Governments and the authorities subordinate to them and to
report whether, by measures of decentralization or otherwise, those relations can be simplified
and improved, and the system of Government better adapted both to meet the requirements
and promote the welfare of the different provinces and, without improving its strength and
unity, to bring the executive power into closer touch with local conditions’. The Commission
submitted its report in February 1909.

QUESTION 30. MTU4MzMyK1Zpa2FzIFV0c2F2K3Zpa2FzdXRzYXZAaG90bWFpbC5jb20rUVVFU1RJT0


4gMjk=
Consider the following statements about the Deccan Riots of 1875.

1. The riots occurred in tribal regions of Madras Province.


2. It was a peasant uprising against the Marwari and Gujrati money lenders.
3. The rioters' specific purpose was to obtain and destroy the bonds, decrees, and other
documents in the possession of the moneylenders.

Which of the above statements is/are correct?

a) 1 and 2
b) 2 and 3
c) 1 and 3
d) All of the above
Correct Answer: B

IASbaba
Score:
Web: http://ilp.iasbaba.com/
45.00 /
Email: ilp@iasbaba.com
Page 18 150
2019 - Test 9- History &
Exam Title :
Cu...
Email : vikasutsav@hotmail.com
Contact :
Your Answer: Unanswered
Explanation

Solution (b)

In May and June 1875, peasants of Maharashtra in some parts of Pune, Satara and Ahmednagar
districts revolted against increasing agrarian distress. The Deccan Riots of 1875 targeted
conditions of debt peonage (kamiuti) to moneylenders. The rioters' specific purpose was to
obtain and destroy the bonds, decrees, and other documents in the possession of the
moneylenders.

The Deccan riots of 1875 were directed mainly against the Marwari and Gujrati moneylenders.

A combination of excess land revenue demands by the government and the world cotton prices
slump due to the American Civil War had pushed the peasants into deep indebtedness. The
Moneylenders had confiscated the lands of the local peasants. The Rioting peasants targeted
the Bonds they had signed with the moneylenders, the entire Poona district was ablaze in June
1875. The government had to call in the army to control the situation.

QUESTION 31. MTU4MzMyK1Zpa2FzIFV0c2F2K3Zpa2FzdXRzYXZAaG90bWFpbC5jb20rUVVFU1RJT0


4gMzA=
Which of the following events are not attributed to Curzon’s tenure as Governor-General of
India?

a) Appointment of the University Commission.


b) Appointment of Police Commission
c) Ancient Monuments Act
d) Surat Split in Congress
Correct Answer: D
Your Answer: C
Explanation

Solution (d)

Police Commission {Sir Andrew Frazer}

Lord Curzon set up a Police Commission under Sir Andrew Frazer in 1902-03. On the basis of
recommendation of this commission, a basic structure of Indian domestic intelligence developed
and remained almost same till 1947. He attached Department of Criminal Intelligence (DCI) to
the Government of India while set up the Criminal Investigation Departments (CIDs) in the
provinces of British India. In 1903, the Thugi and Dakaiti Department was abolished.

Raleigh Commission

The Indian Universities and the colleges were slowly becoming cradle of propaganda against
the Government. To bring the universities under control, Lord Curzon appointed Raleigh
Commission under Sir Thomas Raleigh. The Raleigh Commission had only one Indian member,
Syed Hussain Belgrami. When Hindus protested about this, Justice Guru Das Banerjee was
called from the High Court of Calcutta and made a member. This commission submitted its
report in 1902 and this followed introduction of a Bill called Raleigh Bill. The Raleigh Bill when
became an act, it was called Indian Universities Act 1904. Indian Universities Act 1904 This Act
reorganised the constitution of the Syndicates; provided for the official inspection of the

IASbaba
Score:
Web: http://ilp.iasbaba.com/
45.00 /
Email: ilp@iasbaba.com
Page 19 150
2019 - Test 9- History &
Exam Title :
Cu...
Email : vikasutsav@hotmail.com
Contact :
colleges and placed the final decision concerning the affiliation and disaffiliation of colleges in
the hands of the Government of India. This act was severely criticized by scholars like Gopal
Krishna Gokhle. The first provision of this act was that the governing bodies of the universities
were to be reconstituted and the size of the Senates was reduced. Now the number in the
senate could be minimum 50 and maximum 100. Each of them would hold the office for 6 years.
For the Universities of Bombay, Calcutta and Madras, the elected fellows were to be 50 and for
rest of the universities, the number was fixed 15. This act allowed the Government to appoint a
majority of the fellows in a university. The Governor General was now empowered to decide a
University’s territorial limits and also affiliation between the universities and colleges. The
Indian Universities Act made the universities and colleges completely under the Government
control. However, for better education and research a grant of Rs. 5 Lakh per year for 5 years
was also accepted. This was the beginning of university grants in India which later became a
permanent feature in the structure of India education.

Ancient Monuments Preservation Act 1904

One of the most remarkable acts passed during the times of Lord Curzon was the Ancient
Monuments Preservation Act of 1904. This act made any injury to the protected monuments an
offence punishable under the law. This act also established the Archaeological Department
which was to collect the historical documents and importance, conduct excavations and bring
the ancient historical information into light.

QUESTION 32. MTU4MzMyK1Zpa2FzIFV0c2F2K3Zpa2FzdXRzYXZAaG90bWFpbC5jb20rUVVFU1RJT0


4gMzE=
Who among the following was/were economic critic/critics of colonialism in India?

1. Dadabhai Naoroji
2. M.G. Ranade
3. R. C. Dutt

Select the code from following:

a) 1 and 3 only
b) 2 and 3 only
c) 1 and 2 only
d) All of the above
Correct Answer: D
Your Answer: A
Explanation

Solution (d)

Dadabhai Naoroji, R. C. Dutt, Ranade, Gokhale, G. Subramania Iyer, were among those who
grounded Indian nationalism firmly on the foundation of anti-imperialism by fashioning the
world’s first economic critique of colonialism, before Hobson and Lenin.
The drain theory was established by Dadabhai Naoroji. Increased poverty and lower wages
were among the indirect products of colonial rule according to Dutt.

IASbaba
Score:
Web: http://ilp.iasbaba.com/
45.00 /
Email: ilp@iasbaba.com
Page 20 150
2019 - Test 9- History &
Exam Title :
Cu...
Email : vikasutsav@hotmail.com
Contact :

QUESTION 33. MTU4MzMyK1Zpa2FzIFV0c2F2K3Zpa2FzdXRzYXZAaG90bWFpbC5jb20rUVVFU1RJT0


4gMzI=
Q.33 ) Which of the following were the main objectives of Queen Victoria's Proclamation (1858)?

1. To disclaim any intention to annex Indian States


2. To place the Indian administration under the British Crown.
3. To regulate East India Company's trade with India

Select the code from following:

a) 1 and 2 only
b) 2 and 3 only
c) 1 and 3 only
d) All of the above
Correct Answer: A
Your Answer: A
Explanation

Solution (a)

The Company as a commercial entity ceased to exist after the Proclamation of 1858. Hence
statement (3) is incorrect. The Proclamation intended to create a group of local kingdoms and
provinces which could be used to safeguard the Indian territories if a revolt like 1857 happened
again. This was the beginning of Policy of Subordinate Union which lasted till 1935. The
Queen’s proclamation also transferred all the Company’s territories and their administration
into the hands of British Government.

QUESTION 34. MTU4MzMyK1Zpa2FzIFV0c2F2K3Zpa2FzdXRzYXZAaG90bWFpbC5jb20rUVVFU1RJT0


4gMzM=
Consider the following statements with reference to the Shyamji Krishna Varma:

1. He Published a Monthly Journal called The Indian Sociologist.


2. He founded the India House in England
3. He instituted Scholarships for Indians studying in foreign countries.

Which of the above statements are correct?

a) 1 and 3 only
b) 1 and 2 only
c) 2 and 3 only
d) None of the above
Correct Answer: B
Your Answer: C
Explanation

Solution (b)

Shyamji Krishna Varma was an Indian revolutionary fighter, lawyer and journalist who founded
the Indian Home Rule Society , India House and The Indian Sociologist in London .

IASbaba
Score:
Web: http://ilp.iasbaba.com/
45.00 /
Email: ilp@iasbaba.com
Page 21 150
2019 - Test 9- History &
Exam Title :
Cu...
Email : vikasutsav@hotmail.com
Contact :

In 1905 he founded the India House and The Indian Sociologist , which rapidly developed as an
organised meeting point for radical nationalists among Indian students in Britain at the time
and one of the most prominent centres for revolutionary Indian nationalism outside India. Most
famous among the members of this organisation was Veer Savarkar . Krishna Varma moved to
Paris in 1907, avoiding prosecution. He died in 1935

Later in 1905, Shyamji attended the United Congress of Democrats held at Holborn Town Hall a
s a delegate of the India Home Rule Society. His resolution on India received an enthusiastic
ovation from the entire conference. Shyamji’s activities in England aroused the concern of the
British government: He was disbarred from Inner Temple and removed from the membership
list on 30 April 1909 for writing anti-British articles in The Indian Sociologist . Most of the
British press were anti–Shyamji and carried outrageous allegations against him and his
newspaper. He defended them boldly. The Times referred to him as the "Notorious
Krishnavarma". Many newspapers criticised the British progressives who supported Shyamji
and his view. His movements were closely watched by British Secret Services, so he decided to
shift his headquarters to Paris , leaving India House in charge of Vir Savarkar . Shyamji left
Britain secretly before the government tried to arrest him.

QUESTION 35. MTU4MzMyK1Zpa2FzIFV0c2F2K3Zpa2FzdXRzYXZAaG90bWFpbC5jb20rUVVFU1RJT0


4gMzQ=
"Home Charges" constituted a major thread in the economic critique of the British government.
Which of the following funds constituted "Home Charges"?

1. Funds used to support the India office in London.


2. Funds used to pay salaries and pensions of British personnel engaged in India.
3. Funds used for waging wars outside India by the British.

Select the code from following:

a) 1 and 2 only
b) 1 and 3 only
c) 2 and 3 only
d) All of the above
Correct Answer: A
Your Answer: A
Explanation

Solution (a)

It is clear from Economic History of India by RC Datt: " The Indian Tribute whether weighted
in the scales of justice or viewed in the light of our interest, will be found to be at variance with
humanity, with the commonsense and with the received maxims of economical science. It would
be true wisdom then to provide for the future payment of such of the Home Charges of the
Indian Government as really from the tribute out of Indian Exchequer. These charges would be
probably found to be the dividends on East India Stock, interest in Home debt, the salaries of
the officers, establishments of the and building connected with the Home Department of Indian
Government, furlough and retired pay to members of the Indian Military and Civil Services
when at Home, Charges of all descriptions paid in this country connected with the British
troops serving in India and portion of the cost of transporting the British troops to and from
India".

IASbaba
Score:
Web: http://ilp.iasbaba.com/
45.00 /
Email: ilp@iasbaba.com
Page 22 150
2019 - Test 9- History &
Exam Title :
Cu...
Email : vikasutsav@hotmail.com
Contact :

QUESTION 36. MTU4MzMyK1Zpa2FzIFV0c2F2K3Zpa2FzdXRzYXZAaG90bWFpbC5jb20rUVVFU1RJT0


4gMzU=
Which of the following events is not related to tenure of Lord Lytton as the Governor General of
India?

a) Creation of Statutory Civil Services.


b) The Second Afghan War.
c) Passing of the Vernacular Press Act.
d) Passing of First Factory Act
Correct Answer: D
Your Answer: A
Explanation

Solution (d)

The First Factory Act was passed in 1881 by lord Ripon, Successor of Lord Lytton. The
Statutory Civil Services was created in 1878-89. Under this the Government could employ some
Indians of “good family and standing” to the SCC on the recommendation of the Provincial
Governments, subject to the final approval of the Secretary of States for India.

The vernacular press Act was passed in 1878 to control the publications in oriental/Vernacular
languages. Under this a magistrate was empowered to call for any publisher to furnish a bond
undertaking not to publish content considered as degrading the British Government. He could
demand security and forfeit it, and could seize the printing equipment.

QUESTION 37. MTU4MzMyK1Zpa2FzIFV0c2F2K3Zpa2FzdXRzYXZAaG90bWFpbC5jb20rUVVFU1RJT0


4gMzY=
Which of the following were limitations of Social Reform Movements?

1. Narrow Social Base

2. Movement did not reach rural India

3. Casteism remained strong

4. Social Harmony increased

Select the code from following:

a) 1, 2 and 3 only
b) 1, 3 and 4 only
c) 1 and 4 only
d) All the above
Correct Answer: A
Your Answer: B
Explanation

Solution (a)

IASbaba
Score:
Web: http://ilp.iasbaba.com/
45.00 /
Email: ilp@iasbaba.com
Page 23 150
2019 - Test 9- History &
Exam Title :
Cu...
Email : vikasutsav@hotmail.com
Contact :

Limitations of the Social Reform Movement

Narrow Social Base

Reform in practice in any case affected a very small minority. Only the educated and urban
middle class was involved in the social reform movement, while the needs of vast majority of
peasants and the urban poor were ignored.

Movement did not reach rural India

Given the situation of widespread illiteracy in the rural areas and because of the absence of
modern and diversified communications network, they were doomed to have a very limited
audience, mainly urban-based. Thus, even in terms of its practical appeal the movement
remained urban, besides its other limitations.

Casteism remained strong

Caste distinctions remained strong and the religious and social practices did not die away.
Caste and customs proved to be hard to eradicate from Indian consciousness. The tendency of
the reformers to appeal to the greatness of the past and to rely on scriptural authority led to
compartmentalizing religions as also alienating high caste Hindus from lower caste Hindus.

Communal Consciousness

Overemphasis on religious, philosophical aspects of culture while underemphasizing secular


aspects led to the Hindus praising ancient Indian History and Muslims confining to the
medieval history. This created a notion of two separate segments of people and increased
communal consciousness.

Do you know?

· Indian National Social Conference which was founded in 1887 by M.G. Ranade and
Raghunath Rao, met on the side-lines of the Congress annual cells, deliberated on social issues.
It has also been called as the social cell of Congress. However, due to opposition of Tilak
and other extremists in 1895, Social Conference has to disassociate itself completely
from Congress.

THINK

· Character of the Social Reform Movement.

QUESTION 38. MTU4MzMyK1Zpa2FzIFV0c2F2K3Zpa2FzdXRzYXZAaG90bWFpbC5jb20rUVVFU1RJT0


4gMzc=
Consider the following statements with regard to Young Bengal group of students:

1. They posed an intellectual challenge to the religious and social orthodoxy of Hinduism.

2. They had complete faith in everything British and Western learning which alienated them
from the masses.

3. They were influenced by Henry Vivian Derozio and were responsible for the establishment of
the Society for Translating European Sciences.

IASbaba
Score:
Web: http://ilp.iasbaba.com/
45.00 /
Email: ilp@iasbaba.com
Page 24 150
2019 - Test 9- History &
Exam Title :
Cu...
Email : vikasutsav@hotmail.com
Contact :

Which of the statements given above are correct?

a) 1 and 2 only
b) 1 and 3 only
c) 2 and 3 only
d) All of the above
Correct Answer: A
Your Answer: B
Explanation

Solution (a)

Young Bengal, a controversial group of students, influenced by a Eurasian teacher of Hindu


College in Calcutta (Henry Vivian Derozio), were responsible for the establishment of Society
for Acquisition of General Knowledge .

This group became infamous for their social rebellion. Their rebellion extended to the religious
sphere and posed an intellectual challenge to the religious and social orthodoxy of Hinduism.

Their very radicalism and complete faith in everything British and Western learning alienated
them from the masses.

Do you know?

Derozio was one of those rare teachers whose attachment to knowledge, love of truth, and
hatred of evil made a profound impression upon those who came in contact with him. Like
Socrates, he pursued what was right and was accused of misguiding the youth. Some influential
Hindus had him removed from service and shortly thereafter, he died. But the Derozian
influence continued and it was known as the Young Bengal Movement. All the leading
movements of the period were connected with it.

THINK

· Young Bengal Movement

· Henry Louis Vivian Derozio contributions and works

QUESTION 39. MTU4MzMyK1Zpa2FzIFV0c2F2K3Zpa2FzdXRzYXZAaG90bWFpbC5jb20rUVVFU1RJT0


4gMzg=
Which among the following were the characteristics of early nineteenth century socio-religious
reformers?

1. These reformers gained from western education and were West-sponsored .

2. The reforms promoted by these reformers were not meant to structurally reorganize the
whole society .

3. Issues dealt by these reformers were applicable and common to the higher classes of the
Hindu society and had lesser meaning for the lower classes .

Choose appropriate answer:

IASbaba
Score:
Web: http://ilp.iasbaba.com/
45.00 /
Email: ilp@iasbaba.com
Page 25 150
2019 - Test 9- History &
Exam Title :
Cu...
Email : vikasutsav@hotmail.com
Contact :

a) 1 and 2 only
b) 1 and 3 only
c) 2 and 3 only
d) All of the above
Correct Answer: C
Your Answer: A
Explanation

Q. 39 ) Solution (c)

The early nineteenth century was marked by the transformation of the Indian society and the
initiative for such reforms came from individuals who revolted against the prescribed rules of
the society.

· These reformers though gained from western education were not West-sponsored but they
themselves created the reforms and promoted them.

· It is also undeniable that the reform movements had a strong intellectual base which kept
them aloof from the masses and were confined to the educated sections of the society.

· The reforms were not meant to structurally reorganize the whole society for the benefit of the
underprivileged or the depressed sections of the society. The aim of the reforms was to infuse a
new life into the existing social structure.

· It has been noted by most historians that the issues dealt with by the social reformers in the
early part of the nineteenth century were applicable and common to the higher classes of the
Hindu society and had no meaning for the lower classes.

· The movements of the later part of the nineteenth century along with those of the twentieth
century were the ones that reached across to the wider society including the lower and
depressed sections of the society. (Note: Question is about early 19 th century).

QUESTION 40. MTU4MzMyK1Zpa2FzIFV0c2F2K3Zpa2FzdXRzYXZAaG90bWFpbC5jb20rUVVFU1RJT0


4gMzk=
Consider the following statements regarding the “Self-Respect Movement”:

1. It was founded by E.V. Ramaswamy against Brahmanism in Kerala.

2. It aimed to achieve a society where backward classes have equal rights.

3. The movement promoted inter-caste marriages which were not officiated by a Brahmin
Priest.

Which of the above statements are correct?

a) 2 and 3 only
b) 1 and 2 only
c) 1 only
d) None of the above
Correct Answer: A
Your Answer: B

IASbaba
Score:
Web: http://ilp.iasbaba.com/
45.00 /
Email: ilp@iasbaba.com
Page 26 150
2019 - Test 9- History &
Exam Title :
Cu...
Email : vikasutsav@hotmail.com
Contact :
Explanation

Solution (a)

The Self-Respect Movement is a movement with the aim of achieving a society where backward
castes have equal human rights, and encouraging backward castes to have self-respect in the
context of a caste-based society that considered them to be a lower end of the hierarchy.

It was started by E.V. Ramaswamy (popularky known as Periyar) in Tamil Nadu (not Kerala).

Its main aim was to improve upon the socio-economic conditions of the low castes Tamils. Later
it had profound implications. The main objectives of this movement were inculcation and
dissemination of knowledge of political education. Right to lead life with dignity and self-respect
and do away with the exploitative system based on superstitions and beliefs. Abolition of the
evil social practices and protection of women rights. Establishment and maintenance of homes
for orphans and widow and opening of educational institutions for them. This movement gained
popularity in no time and became a political platform.

THINK

· What were the impacts of Self-Respect Movement on society?

QUESTION 41. MTU4MzMyK1Zpa2FzIFV0c2F2K3Zpa2FzdXRzYXZAaG90bWFpbC5jb20rUVVFU1RJT0


4gNDA=
Consider the following statements regarding the office of Viceroy of India:

1. After the Act of 1858, the office of Governor General was abolished and replaced by the office
of Viceroy.

2. The First Viceroy of India was Lord Canning

Which of the above statements are NOT correct?

a) 1 only
b) 2 only
c) Both 1 and 2
d) Neither 1 nor 2
Correct Answer: A
Your Answer: D
Explanation

Solution (a)

Note: Incorrect options have been asked.

The office of Viceroy was established by the Government of India Act of 1858, but the office of
Governor General was not abolished.

Lord Canning served as Governor General of India from 1856 to 1862. During his tenure, the
Government of India Act, 1858 was passed which created the office of Viceroy to be held by the
same person who was Governor General of India . Thus, Lord Canning also served as first
Viceroy.

IASbaba
Score:
Web: http://ilp.iasbaba.com/
45.00 /
Email: ilp@iasbaba.com
Page 27 150
2019 - Test 9- History &
Exam Title :
Cu...
Email : vikasutsav@hotmail.com
Contact :

Think

· First Governor General of India

QUESTION 42. MTU4MzMyK1Zpa2FzIFV0c2F2K3Zpa2FzdXRzYXZAaG90bWFpbC5jb20rUVVFU1RJT0


4gNDE=
Consider the following statements regarding ‘Swadeshi movement’ of 1905:

1. It was the first Gandhian Movement.

2. It was triggered because of the partition of Bengal.

3. There was large scale participation of peasants in picketing foreign shops.

Which of the above statements are INCORRECT?

a) 1 only
b) 1 and 3
c) 2 only
d) 2 and 3
Correct Answer: B
Your Answer: C
Explanation

Solution (b)

Note: The incorrect options have been asked.

Swadeshi Movement

Swadeshi movement was the first mass movement of India. I was triggered by the decision of
Government to divide Bengal. According to the government this step was taken for the
administrative convenience as Bengal was a big presidency. But the actual reason was to divide
and Rule.

Most of the Indian intelligentsia was from Bengal and they used to criticize British policies. The
division was made in such a way that the Bengali speaking Muslim majority went to the Eastern
Bengal and in West Bengal, Bengali speaking people became a minority with Hindi and Oriya
speaking people.

The point to be remembered is that, that this was an urban movement. So there was no
participation of peasants.

Note: Till now Mahatma Gandhi hadn’t come in the picture of Indian Independence Struggle.

Think

· Achievements of Moderates

· Surat Split

IASbaba
Score:
Web: http://ilp.iasbaba.com/
45.00 /
Email: ilp@iasbaba.com
Page 28 150
2019 - Test 9- History &
Exam Title :
Cu...
Email : vikasutsav@hotmail.com
Contact :

QUESTION 43. MTU4MzMyK1Zpa2FzIFV0c2F2K3Zpa2FzdXRzYXZAaG90bWFpbC5jb20rUVVFU1RJT0


4gNDI=
The first Population census of India was held in:

a) 1872
b) 1881
c) 1891
d) 1901
Correct Answer: A
Your Answer: B
Explanation

Solution (a)

A systematic and modern population census, in its present form was conducted non
synchronously between 1865 and 1872 in different parts of the country. This effort culminating
in 1872 has been popularly labeled as the first population census of India However, the first
synchronous census in India was held in 1881. Since then, censuses have been undertaken
uninterruptedly once every ten year.

QUESTION 44. MTU4MzMyK1Zpa2FzIFV0c2F2K3Zpa2FzdXRzYXZAaG90bWFpbC5jb20rUVVFU1RJT0


4gNDM=
Which of the following are the associated with Dadabhai Naoroji ?

1. Gyan Prasarak Mandali


2. Poverty and un-British Rule in India
3. Bombay Gazette
4. Bombay Association

Select the code from following:

a) 1 and 2
b) 1, 2 and 4
c) 1, 2 and 3
d) 2 and 3
Correct Answer: B
Your Answer: D
Explanation

Q. 44 ) Solution ( b )

Contributions/works of Dadabhai Naoroji

‘Congress’ means assembly of the people. The name Congress was suggested to the
organisation by Dadabhai Naoroji.

Dadabhai Naoroji founded the East Indian Association in 1866.

IASbaba
Score:
Web: http://ilp.iasbaba.com/
45.00 /
Email: ilp@iasbaba.com
Page 29 150
2019 - Test 9- History &
Exam Title :
Cu...
Email : vikasutsav@hotmail.com
Contact :

Dadabhai Naoroji is the author of the book ‘‘Poverty and UnBritish Rule in India’’ which
contains the famous ‘‘drain theory’’.

Dadabhai Naoroji is known as ‘‘The Grand Old Man of India’’. He was the first Indian to become
a member of the House of Commons on the Liberal Party ticket. He became the president of
INC thrice, in 1886, 1893 and 1906. He founded ‘Gyan Prakash Mandali’ and Bombay
Association in 1852. He is also known as ‘Father of Indian Economics and Politics’.

QUESTION 45. MTU4MzMyK1Zpa2FzIFV0c2F2K3Zpa2FzdXRzYXZAaG90bWFpbC5jb20rUVVFU1RJT0


4gNDQ=
Who among the following was the first foreigner to become the president of INC?

a) A O Hume
b) Annie Besant
c) George Yule
d) Henry Cotton
Correct Answer: C
Your Answer: C
Explanation

Solution (c)

George Yule was a Scottish merchant in England and India who served as the fourth President
of the Indian National Congress in 1888 at Allahabad, the first non-Indian to hold that office.

Having had for its first three Presidents a Hindu, a Parsi and a Muslim respectively, the fourth
Congress which met in Allahabad, turned for the first time, to a non - Indian for its Presidential
chair.

In doing so, it thought of one who was not unfamiliar to Indians, but rather someone genuinely
interested in their welfare and progress: George Yule.

Under friendly pressure W. C. Bonnerjee persuaded him to accept the invitation of the Congress
to preside over the Allahabad session.

Throughout his Indian career, George Yule won the respect, the admiration, and the regard of
everybody with whom he came in contact - Indian and European, official and non - official.

THINK

· Who were the other foreigners who became the president of INC?

QUESTION 46. MTU4MzMyK1Zpa2FzIFV0c2F2K3Zpa2FzdXRzYXZAaG90bWFpbC5jb20rUVVFU1RJT0


4gNDU=
Bahramji Malabari was one of the most prominent Parsi social reformers of India. Which of the
following statements are correct about him?

IASbaba
Score:
Web: http://ilp.iasbaba.com/
45.00 /
Email: ilp@iasbaba.com
Page 30 150
2019 - Test 9- History &
Exam Title :
Cu...
Email : vikasutsav@hotmail.com
Contact :

1. He criticized the prohibition of widow remarriage by Hindus and placed the blame squarely
with that religion's "priestly class" and the "social monopolists" (i.e. the Brahmin caste) for their
"vulgar prejudices.

2. Malabari published a set of Notes on Infant Marriage and Enforced Widowhood, which he
sent to 4,000 leading Englishmen and Hindus.

3. He was instrumental in passing the Age of Consent Act in 1894.

4. Malabari published a book in England “An Appeal from the daughters of India”.

Select the correct code from the following:

a) 1,2 and 3
b) 2,3 and 4
c) 1,3 and 4
d) All of the above
Correct Answer: D
Your Answer: Unanswered
Explanation

Solution (d)

Behramji Merwanji Malabari was an Indian poet, publicist, author, and social reformer best
known for his ardent advocacy for the protection of the rights of women.

He wrote extensively about raising the age of girls for marriage which gained sympathy of the
people of India. The pundits and shastris of Benares and Mathura listening to his lectures on
social reform started accepting the fairness of his arguments. Malabari published a book in
England “An Appeal from the daughters of India”.

He chose journalism as his career and devoted his life to fighting the evils of child marriage and
enforced widowhood among the Hindus. He was a fearless journalist. He travelled extensively
throughout India and made three trips to England. He was vociferous in his struggle to get
infant marriage and correlated evils abolished. Seeking women’s emancipation he advocated,
“If new India is to be blessed with a generation of free and enlightened sons, a nation to
manage its own affairs the Hindus of today might to see in their midst a race of free,
enlightened mothers”. Lokmanya Bal Gangadhar Tilak advised Malabari to look after the
interests of women in his own community but Malabari relentlessly persisted in his efforts and
succeeded.

He became famous for raising the issue of freedom of Hindu widows to remarry, which was
debated throughout India. His successful campaign from 1884-1891 resulted in passing the Age
of Consent Act in 1894. His stand was based on the principles of justice and humanity. His
invaluable “Notes on Infant Marriage and Enforced Widowhood” published in 1894 was the vad
e mecum of social reformers.

QUESTION 47. MTU4MzMyK1Zpa2FzIFV0c2F2K3Zpa2FzdXRzYXZAaG90bWFpbC5jb20rUVVFU1RJT0


4gNDY=

The second half of the 19 th century saw a remarkable growth in the vernacular press. They
played a critical role in creating socio-political awareness in the masses. They also became a
vehicle in exposing the exploitative policies of the British government. Alarmed, by its growth,

IASbaba
Score:
Web: http://ilp.iasbaba.com/
45.00 /
Email: ilp@iasbaba.com
Page 31 150
2019 - Test 9- History &
Exam Title :
Cu...
Email : vikasutsav@hotmail.com
Contact :
the government passed a Vernacular Press Act. Which of the following statements are correct
about this act?

1. It was passed by Lord Ripon in 1878.

2. The magistrates of the districts were empowered, to call upon a printer and publisher of any
kind to enter into a Bond, undertaking not to publish anything which might “rouse” feelings of
disaffection against the government.

3. The magistrate was authorized to deposit a security, which could be confiscated if the printer
violated the Bond.

Select the code from the following:

a) 1 and 2
b) 2 and 3
c) 1 and 3
d) All of the above
Correct Answer: B
Your Answer: D
Explanation

Solution (b)

Vernacular Press Act was passed by Lord Lytton and it was later repealed by Lord Ripon.

Provisions of the Act are given correctly in the question.

QUESTION 48. MTU4MzMyK1Zpa2FzIFV0c2F2K3Zpa2FzdXRzYXZAaG90bWFpbC5jb20rUVVFU1RJT0


4gNDc=
Consider the following statements regarding the Indigo revolt of Bengal:

1. It was a largely non – violent and farmers used legal means of protest.

2. The modern educated middle class supported the peasants.

3. Indigo commission was appointed by the government which held the planters guilty, and
criticized them for the coercive methods they used with indigo cultivators.

Which of the above statements are correct?

a) 1 only
b) 2 and 3
c) 1 and 3
d) All of the above
Correct Answer: D
Your Answer: D
Explanation

Solution (d)

Indigo Revolt:

IASbaba
Score:
Web: http://ilp.iasbaba.com/
45.00 /
Email: ilp@iasbaba.com
Page 32 150
2019 - Test 9- History &
Exam Title :
Cu...
Email : vikasutsav@hotmail.com
Contact :

The revolt began from Govindpur village in Nadia district of Bengal where Biswas brothers gave
up indigo cultivation. This was followed by a struggle with the Lathiyals and revolt spread in
many parts of Bengal. Strikes, legal actions, violence, social boycott of planters etc. were some
of the tools used in the revolt. Peasant organization to some extent, Hindu Muslim Unity,
support from Bengal intelligentsia made the revolt more effective. Finally, Indigo commission
was appointed which held the planters guilty, and criticized them for the coercive methods they
used with indigo cultivators. The company asked ryots to fulfill their existing contracts but also
told them that they could refuse to produce indigo in future. This was a big relief for the
peasants and gradually the plantations of Indigo came to an end in Bengal.

The revolt was largely non – violent with very few incidents. The key leaders of this revolt were
Biswas Brothers of Nadia {Digambar Biswas and Bishnu Biswas}, Kader Molla of Pabna,
Rafique Mondal of Maida etc.

The plight of these indigo farmers have been depicted by Deen Bandu Mitra’s Bengali play,
‘Neel Darpan’.

QUESTION 49. MTU4MzMyK1Zpa2FzIFV0c2F2K3Zpa2FzdXRzYXZAaG90bWFpbC5jb20rUVVFU1RJT0


4gNDg=
Indian National Movement got an impetus after the foundation of the Indian National Congress
in 1885. Moderates dominated till 1906. Which of the following were demands of the
Moderates?

1. Dominion status for India.

2. Removal of poverty by the rapid development of agriculture and modern industries.

3. Indianisation of Higher administrative services

4. Freedom of speech and press

Select the code from below:

a) 1,2 and 3
b) 2,3 and 4
c) 1,3 and 4
d) All of the above
Correct Answer: B
Your Answer: C
Explanation

Solution (b)

Aims and Objectives of the Congress Leaders from 1885 to 1905:

The early phase of the National Movement was dominated by the moderate leaders like Dada
Bhai Nauroji, Surendra Nath Banerjee, Ferozshah Mehta, Gopal Krishna Gokhale etc. They
believed in peaceful and constitutional methods.

They tried to achieve the following objectives:

· Demand for wider powers for the councils and training in self-government.

IASbaba
Score:
Web: http://ilp.iasbaba.com/
45.00 /
Email: ilp@iasbaba.com
Page 33 150
2019 - Test 9- History &
Exam Title :
Cu...
Email : vikasutsav@hotmail.com
Contact :

· Removal of poverty by the rapid development of agriculture and modern industries.

· Indianization of the higher administrative services.

· Freedom of speech and press for the defence of their civil rights.

The moderate leaders, thus, tried to create the national consciousness and raise the public
opinion against the British imperialism. They created a common platform for the national
struggle.

Methods of the Moderate Leaders:

· They adopted peaceful and constitutional means to achieve their aims.

· They had full faith in the British sense of justice, so they adopted a friendly attitude towards
the British rulers.

· They believed in constitutional reforms. They would send petitions to the British Government
in the hope that it would grant them freedom.

· However, the moderates failed to achieve their aim, as the British government did not
cooperate with them. It gave way to the rise of extremism in the Congress after 1905.

QUESTION 50. MTU4MzMyK1Zpa2FzIFV0c2F2K3Zpa2FzdXRzYXZAaG90bWFpbC5jb20rUVVFU1RJT0


4gNDk=
Who is being quoted in the following statement?

“The only hope for India is from the masses. The upper classes are physically and morally
dead.”

a) Mahatma Gandhi
b) Rabindranth Tagore
c) Sardar Patel
d) Swami Vivekananda
Correct Answer: D
Your Answer: A
Explanation

Solution (d)

Self-Explanatory

The statement was given by Swami Vivekananda to criticise the upper castes and upper rich
class which according to him were indulged in personal luxuries and had no concern for the
motherland.

QUESTION 51. MTU4MzMyK1Zpa2FzIFV0c2F2K3Zpa2FzdXRzYXZAaG90bWFpbC5jb20rUVVFU1RJT0


4gNTA=
‘Industrial Security Annex (ISA) was in news recently in context of

IASbaba
Score:
Web: http://ilp.iasbaba.com/
45.00 /
Email: ilp@iasbaba.com
Page 34 150
2019 - Test 9- History &
Exam Title :
Cu...
Email : vikasutsav@hotmail.com
Contact :

a) Global Environment Facility


b) International Atomic Energy Agency
c) Nuclear Suppliers Groups
d) India-US Defence Cooperation
Correct Answer: D
Your Answer: Unanswered
Explanation

Solution (d)

Industrial Security Annex (ISA) would allow Indian private sector to collaborate with the U.S.
defence industry.

Source: https://www.thehindu.com/news/national/india-us-sign-landmark-comcasa-
deal/article24881277.ece

QUESTION 52. MTU4MzMyK1Zpa2FzIFV0c2F2K3Zpa2FzdXRzYXZAaG90bWFpbC5jb20rUVVFU1RJT0


4gNTE=
The term ‘Vishing’ is used in the context of

a) Cryptocurrency
b) Initial Public Offering (IPOs)
c) First Information Reports (FIRs)
d) None of the above
Correct Answer: D
Your Answer: D
Explanation

Solution (d)

“Vishing” or voice “phishing”

Posing as bank representatives, they call up unsuspecting account holders to extract their
personal details from them on various pretexts.

The information is used to illegally transfer money to e-wallets.

Source: https://www.thehindu.com/news/national/lens-on-premises-of-jharkhand-cyber-
crime-gangs/article24884642.ece

QUESTION 53. MTU4MzMyK1Zpa2FzIFV0c2F2K3Zpa2FzdXRzYXZAaG90bWFpbC5jb20rUVVFU1RJT0


4gNTI=
Allowing someone to die by not doing something that would prolong life is called

a) Voluntary euthanasia
b) Passive euthanasia
c) Non-voluntary euthanasia
d) Active euthanasia

IASbaba
Score:
Web: http://ilp.iasbaba.com/
45.00 /
Email: ilp@iasbaba.com
Page 35 150
2019 - Test 9- History &
Exam Title :
Cu...
Email : vikasutsav@hotmail.com
Contact :
Correct Answer: B
Your Answer: B
Explanation

Solution (b)

Passive Euthanasia: To forgo treatment or life-prolonging medical support

QUESTION 54. MTU4MzMyK1Zpa2FzIFV0c2F2K3Zpa2FzdXRzYXZAaG90bWFpbC5jb20rUVVFU1RJT0


4gNTM=
Which of the following states share the longest border with Bangladesh?

a) Assam
b) Tripura
c) Mizoram
d) Meghalaya
Correct Answer: B
Your Answer: D
Explanation

Solution (b)

Bangladesh and India share a 4,156 km (2,582 mi)-long international border, the fifth-longest
land border in the world, including 262 km (163 mi) in Assam, 856 km (532 mi) in Tripura, 180
km (110 mi) in Mizoram, 443 km (275 mi) in Meghalaya, and 2,217 km (1,378 mi) in West
Bengal. lowest

Source: https://www.thehindu.com/news/national/other-states/assam-governor-flags-
cattle-smuggling-on-the-border/article24886692.ece

QUESTION 55. MTU4MzMyK1Zpa2FzIFV0c2F2K3Zpa2FzdXRzYXZAaG90bWFpbC5jb20rUVVFU1RJT0


4gNTQ=
Consider the following statements about ‘Section 377’ of the Indian Penal Code (IPC)’

1. It is modelled on the England’s ‘Buggery Act of 1533’.

2. It was introduced in India post ‘ Sepoy Mutiny’.

3. Supreme Court ruled unanimously in ‘ Navtej Singh Johar v. Union of India’ that Section 377
was unconstitutional "in so far as it criminalises consensual sexual conduct between adults of
the same sex".

Select the correct statements

a) 1 and 3
b) 2 and 3
c) 3 Only
d) 1, 2 and 3
Correct Answer: D

IASbaba
Score:
Web: http://ilp.iasbaba.com/
45.00 /
Email: ilp@iasbaba.com
Page 36 150
2019 - Test 9- History &
Exam Title :
Cu...
Email : vikasutsav@hotmail.com
Contact :
Your Answer: C
Explanation

Solution (d)

Section 377 of the Indian Penal Code is a section of the Indian Penal Code introduced in 1861
during the British rule of India. Modelled on the Buggery Act of 1533, it is used to criminalize
sexual activities "against the order of nature". On 6 September 2018, the Supreme Court of
India decided to announce that the application of Section 377 to consensual homosexual sex
between adults was unconstitutional, "irrational, indefensible and manifestly arbitrary", but
that Section 377 remains in force relating to sex with minors, non-consensual sexual acts, and
bestiality.
Sexual activity between a person and an animal
On 6 September 2018, the Court ruled unanimously in Navtej Singh Johar v. Union of India that
Section 377 was unconstitutional "in so far as it criminalises consensual sexual conduct
between adults of the same sex".

Section 377 verdict - Highlights

· SC struck down provisions of Section 377 of the Indian Penal Code that criminalised same-sex
relationships.

· The Supreme Court held that the law violated the fundamental rights of citizens.

· The court noted that Section 377 of the IPC was used as a weapon to harass the members of
the LGBTQ community, resulting in discrimination.

· Majoritarian views and popular morality cannot dictate constitutional rights

· It said that history owes the LGBTQ community an apology Choice of a partner is part of the
fundamental right to privacy

· It held that the Section would apply to “unnatural” sexual acts like bestiality. Sexual act
without consent continues to be a crime under Section 377

Source: https://www.thehindu.com/opinion/lead/drawing-a-curtain-on-the-past/
article24937833.ece

QUESTION 56. MTU4MzMyK1Zpa2FzIFV0c2F2K3Zpa2FzdXRzYXZAaG90bWFpbC5jb20rUVVFU1RJT0


4gNTU=
India has signed which of the following foundational agreements with United States of
America?

1. General Security Of Military Information Agreement (GSOMIA)

2. Basic Exchange and Cooperation Agreement for Geo-spatial Cooperation (BECA)

3. Communications Compatibility and Security Agreement (COMCASA)

4. Logistics Exchange Memorandum of Agreement (LEMOA)

Select the correct code:

a) 3 and 4 Only

IASbaba
Score:
Web: http://ilp.iasbaba.com/
45.00 /
Email: ilp@iasbaba.com
Page 37 150
2019 - Test 9- History &
Exam Title :
Cu...
Email : vikasutsav@hotmail.com
Contact :

b) 1, 3 and 4
c) 2, 3 and 4
d) All of the above
Correct Answer: B
Your Answer: Unanswered
Explanation

Solution (b)

India has signed all but Basic Exchange and Cooperation Agreement for Geo-spatial
Cooperation (BECA).

COMCASA

· India and the United States on Thursday sealed the landmark Communications Compatibility
and Security Agreement (COMCASA)

· It will allow the U.S. to transfer specialised equipment for encrypted communications for U.S.-
origin platforms like C-17, C-130 and P-8I aircraft.

· It comes into force immediately and is valid for 10 years.

General Security Of Military Information Agreement (GSOMIA)

· It allows sharing of classified information from the U.S. government and American companies
with the Indian government and defence Public Sector Undertakings (PSUs) but not with Indian
private companies.

Read More - https://www.financialexpress.com/defence/what-is-comcasa-security-


agreement-between-india-us-signed-during-22-summit-a-boost-for-defence-
preparedness/1304439/

Source: https://www.thehindu.com/news/national/lemoa-already-fully-operational/
article24904359.ece

QUESTION 57. MTU4MzMyK1Zpa2FzIFV0c2F2K3Zpa2FzdXRzYXZAaG90bWFpbC5jb20rUVVFU1RJT0


4gNTY=
Defence Technology and Trade Initiative (DTTI) is often in news in the context of India and

a) United States of America


b) Israel
c) Japan
d) France
Correct Answer: A
Your Answer: A
Explanation

Solution (a)

News: MoU was signed between the U.S. Defence Innovation Unit (DIU) and the Indian
Defence Innovation Organization – Innovation for Defence Excellence (DIO-iDEX), which will

IASbaba
Score:
Web: http://ilp.iasbaba.com/
45.00 /
Email: ilp@iasbaba.com
Page 38 150
2019 - Test 9- History &
Exam Title :
Cu...
Email : vikasutsav@hotmail.com
Contact :
look into joint projects for co-production and co-development projects through the Defence
Technology and Trade Initiative (DTTI).

The DTTI aims to:

· Transform the bilateral defence relationship into one that is limited only by independent
strategic decisions, rather than bureaucratic obstacles or inefficient procedures

· Strengthen India’s defence industrial base by moving away from the traditional “buyer-seller”
dynamic toward a more collaborative approach

· Explore new areas of technological collaboration from science and technology cooperation
through co-development and co-production

· Expand U.S.-Indian business ties

Source: https://www.thehindu.com/todays-paper/tp-national/indias-interests-secured-
in-new-pact-officials/article24887267.ece

QUESTION 58. MTU4MzMyK1Zpa2FzIFV0c2F2K3Zpa2FzdXRzYXZAaG90bWFpbC5jb20rUVVFU1RJT0


4gNTc=
In the context, what is the implication of getting access to ‘CENTRIXS’

a) The country will have the privilege to but uranium from the Nuclear Suppliers Group.
b) The country automatically becomes a member of the Nuclear Suppliers Group.
c) The military nuclear installations come under the inspection of International Atomic Energy
Agency (IAEA)
d) None of the above
Correct Answer: D
Your Answer: A
Explanation

Solution (d)

CENTRIXS

· Combined Enterprise Regional Information Exchange System, or CENTRIXS, which is the


secure communication system network of the US.

· India will get access to it as part of COMCASA

· Navy ships with CENTRIXS on board can communicate securely with the U.S. Navy when
needed and can benefit from the wider situational picture of the region as they have a large
number of ships and aircraft deployed.

· CENTRIXS consists of a collection of coalition wide area networks (WAN) known as ‘enclaves’

· It is a great enabler, allowing ship-to-ship operational dialogue between the two nations in text
and web-based formats.

Source: https://www.thehindu.com/todays-paper/tp-national/india-can-track-chinas-
ocean-moves-better/article24897825.ece

IASbaba
Score:
Web: http://ilp.iasbaba.com/
45.00 /
Email: ilp@iasbaba.com
Page 39 150
2019 - Test 9- History &
Exam Title :
Cu...
Email : vikasutsav@hotmail.com
Contact :

QUESTION 59. MTU4MzMyK1Zpa2FzIFV0c2F2K3Zpa2FzdXRzYXZAaG90bWFpbC5jb20rUVVFU1RJT0


4gNTg=
‘Czech Republic’ is NOT bordered by which of the following?

1. Germany

2. Slovakia

3. Poland

4. Slovenia

Select the correct code:

a) 1 and 3
b) 2 and 4
c) 3 Only
d) 4 Only
Correct Answer: D
Your Answer: Unanswered
Explanation

Solution (d)

Czech Republic is a landlocked country in Central Europe bordered by Germany to the west,
Austria to the south, Slovakia to the east and Poland to the northeast.

QUESTION 60. MTU4MzMyK1Zpa2FzIFV0c2F2K3Zpa2FzdXRzYXZAaG90bWFpbC5jb20rUVVFU1RJT0


4gNTk=
Consider the following statements with respect to ‘ MobiliseYourCity ’ Partnership

1. It an international transport alliance under the UN Marrakesh Partnership for Global Climate
Action

IASbaba
Score:
Web: http://ilp.iasbaba.com/
45.00 /
Email: ilp@iasbaba.com
Page 40 150
2019 - Test 9- History &
Exam Title :
Cu...
Email : vikasutsav@hotmail.com
Contact :

2. It aims at assisting beneficiary partners – i.e. national and local governments – in their
preparation of National Urban Mobility Policies and Investment Programs (NUMPs) and
Sustainable Urban Mobility Plans (SUMPs).

3. It is supported by the French and the German Governments and was launched at 21st
Conference of Parties (COP21) meeting.

Select the correct statements

a) 1 and 2
b) 2 and 3
c) 1 and 3
d) All of the above
Correct Answer: D
Your Answer: Unanswered
Explanation

Solution (d)

Mobilise Your City (MYC)

· The MobiliseYourCity Partnership is a global and inclusive network of cities and countries as
well as an umbrella brand of European development cooperation particularly related to the field
of sustainable urban transport. The Partnership aims at assisting beneficiary partners – i.e.
national and local governments – in their preparation of National Urban Mobility Policies and
Investment Programs (NUMPs) and Sustainable Urban Mobility Plans (SUMPs). It facilitates the
establishment of framework conditions for effective investment and sustainable development of
urban transport infrastructure & services as well as regulatory soft measures.

· It is part of an international initiative which is supported by the French and the German
Governments and was launched at 21st Conference of Parties (COP21) meeting in December,
2015.

· Furthermore, the MobiliseYourCity Partnership is a global climate partnership for integrated


urban mobility planning in emerging, developing and EU neighborhood countries, and an
international transport alliance under the UN Marrakesh Partnership for Global Climate Action.
It is a multi-donor action, jointly co-financed by the European Commission’s Directorate-General
for International Cooperation and Development (DG DEVCO), the French Ministry of Ecological
Transition and Solidarity (MTES), the French Facility for Global Environment (FFEM), and the
German Federal Ministry for the Environment, Nature Conservation and Nuclear Safety (BMU).
The Partnership is implemented jointly by its implementing partners ADEME, AFD, CEREMA,
CODATU, EBRD, GIZ, KfW, and WUPPERTAL INSTITUTE. Besides contribution to the
international climate process, it contributes to the UN’s Agenda 2030, specifically Sustainable
Development Goal (SDG) 11: Make cities inclusive, safe, resilient and sustainable.

In News

· Based on a proposal made by Agence Française de Développement (AfD) in 2015, the


European Union has agreed to provide funds of Euro 3.5 million through the AFD to contribute
to specific investments and technical assistance components within the Mobilise Your City
(MYC) programme in India.

· It aims at supporting three pilot cities viz. Nagpur, Kochi and Ahmedabad in their efforts to
reduce their Green House Gas (GHG) emissions related to urban transport by implementing

IASbaba
Score:
Web: http://ilp.iasbaba.com/
45.00 /
Email: ilp@iasbaba.com
Page 41 150
2019 - Test 9- History &
Exam Title :
Cu...
Email : vikasutsav@hotmail.com
Contact :
urban mobility plans at local level and to help India at national level to improve their
sustainable transport policy.

· The three pilot cities selected under the programme as well as MoHUA will benefit from the
Technical Assistance activities.

The main components of the proposed assistance are:

· To support planning and implementation of sustainable urban transport projects,

· Support to strengthening institutional capacity for regulating, steering and planning urban
mobility, and

· Learning and exchange formats with other cities across India for exchanges on best practices.

Source: http://pib.nic.in/PressReleseDetail.aspx?PRID=1545277

QUESTION 61. MTU4MzMyK1Zpa2FzIFV0c2F2K3Zpa2FzdXRzYXZAaG90bWFpbC5jb20rUVVFU1RJT0


4gNjA=
Which of the following statements about ‘SLINEX’ is correct?

a) It is a naval exercise between India and Sri Lanka


b) It is an Air Combat Exercise between India and Singapore
c) It is a naval exercise between India and Singapore
d) It is an Air Combat Exercise between India and Sri Lanka
Correct Answer: A
Your Answer: A
Explanation

Solution (a)

SLINEX (Sri Lanka India Naval Exercise) are a series of naval exercises between the Indian
Navy and the Sri Lanka Navy.

Source: http://pib.nic.in/newsite/PrintRelease.aspx?relid=183328

QUESTION 62. MTU4MzMyK1Zpa2FzIFV0c2F2K3Zpa2FzdXRzYXZAaG90bWFpbC5jb20rUVVFU1RJT0


4gNjE=
Consider the following statements about the ‘East Godavari River Estuarine Ecosystem
(EGREE)’

1. EGREE encompassing Godavari mangroves is the second largest mangrove area, after the Su
nderbans in West Bengal.

2. It is funded by Global Environment Facility (GEF)

Select the correct statements

a) 1 Only
b) 2 Only

IASbaba
Score:
Web: http://ilp.iasbaba.com/
45.00 /
Email: ilp@iasbaba.com
Page 42 150
2019 - Test 9- History &
Exam Title :
Cu...
Email : vikasutsav@hotmail.com
Contact :

c) Both 1 and 2
d) Neither 1 nor 2
Correct Answer: C
Your Answer: C
Explanation

Solution (c)

East Godavari River Estuarine Ecosystem (EGREE) Region

· EGREE in Andhra Pradesh is the second largest mangrove forests on the east coast of India.

· In recognition of its national and global biodiversity significance, a part of the EGREE is
notified as Coringa Wildlife Sanctuary.

· The region is inhabited by rural communities who are also dependent on mangroves and other
marine resources for livelihood.)

· EGREE Foundation has been established under the Andhra Pradesh Society Registration Act
2001.

· This Foundation is a cross-sectoral platform to facilitate implementation of biodiversity


conservation initiatives through the production sectors operating in the EGREE.

· It is a GOI-UNDP-GEF project to protect the flora and fauna in the EGREE Region

Source: https://www.thehindu.com/news/national/andhra-pradesh/pondicherry-shark-
spotted-near-kakinada/article24910627.ece

QUESTION 63. MTU4MzMyK1Zpa2FzIFV0c2F2K3Zpa2FzdXRzYXZAaG90bWFpbC5jb20rUVVFU1RJT0


4gNjI=
‘Bathukamma Festival’ is a floral festival celebrated for nine days during ‘ Durga Navratri’. It is
predominantly celebrated in which of the following states?

a) Gujarat
b) Telangana
c) West Bengal
d) Madhya Pradesh
Correct Answer: B
Your Answer: B
Explanation

Solution (b)

Bathukamma Festival

· It is floral festival celebrated predominantly by the Hindu women of Telangana.

· It is celebrated for nine days during Durga Navratri.

· Bathukamma is a stack of flowers arranged in the shape of Gopuram of a Hindu Temple.

IASbaba
Score:
Web: http://ilp.iasbaba.com/
45.00 /
Email: ilp@iasbaba.com
Page 43 150
2019 - Test 9- History &
Exam Title :
Cu...
Email : vikasutsav@hotmail.com
Contact :

· This festival has been declared as Telangana’s State Festival along with Bonalu because of the
significance of these festivals which reflect the culture of Telangana.

· Bathukamma is followed by Boddemma, which is a 7-day festival.

· Boddemma festival that marks the ending of Varsha Ruthu whereas Bathukamma festival
indicates the beginning of Sarad or Sharath Ruthu.

Source: https://www.thehindu.com/news/national/telangana/heritage-tag-for-two-
irrigation-facilities-in-telangana/article24910458.ece

QUESTION 64. MTU4MzMyK1Zpa2FzIFV0c2F2K3Zpa2FzdXRzYXZAaG90bWFpbC5jb20rUVVFU1RJT0


4gNjM=
‘ Anicut ’ refers to

a) Rainfall Bund
b) Land grants made to military officers
c) Waterwheel used in the irrigation of land
d) Wasteland and converted to cultivated land
Correct Answer: A
Your Answer: B
Explanation

Solution (a)

News:

· For the first time in India an irrigation project has been recognized as heritage structure by
the ICID.

· ICID recognised 2 structures Sadarmat anicut across river Godavari in Nirmal district and
Pedda Cheruvu in Kamareddy district in Telangana.

· The anicut, which is English word for Telugu’s ana-katta, meaning a rainfall bund

International Commission on Irrigation and Drainage (ICID)

· It is a leading scientific, technical, international not-for-profit, non-governmental organization.


ICID

· It is a professional network of experts from across the world in the field of irrigation, drainage,
and flood management.

· The main mission is to promote ‘Sustainable agriculture water management’ to achieve ‘Water
secure world free of poverty and hunger through sustainable rural development’.

· ICID is a knowledge sharing platform dedicated to issues that covers the entire spectrum of
agricultural water management practices ranging from rainfed agriculture to supplemental
irrigation, land drainage, deficit irrigation to full irrigation, etc.

IASbaba
Score:
Web: http://ilp.iasbaba.com/
45.00 /
Email: ilp@iasbaba.com
Page 44 150
2019 - Test 9- History &
Exam Title :
Cu...
Email : vikasutsav@hotmail.com
Contact :

· In addition, drainage of agricultural lands forms the core theme of commission’s activities.
Floods and drought; the two extremes of increasingly variable climate as a result of potential
climate change, also form the focus of activities.

Source: https://www.thehindu.com/news/national/telangana/heritage-tag-for-two-
irrigation-facilities-in-telangana/article24910458.ece

QUESTION 65. MTU4MzMyK1Zpa2FzIFV0c2F2K3Zpa2FzdXRzYXZAaG90bWFpbC5jb20rUVVFU1RJT0


4gNjQ=
The ‘Hatti Community’ is majorly found in

a) Vidarbha
b) Rayalaseema
c) Himachal Pradesh
d) Odisha
Correct Answer: C
Your Answer: C
Explanation

Solution (c)

Hatti Community

News: Demand to accord tribal status to the Hatti community

About

· The Hattis are a unique people living in the trans-Giri area of Himachal Pradesh.

· Some of them are in Uttarakhand, enjoying the Scheduled Tribe status and the resultant
benefits. In Himachal, they continue to feel neglected

· The Tran-Giri area of district Sirmaur was part of the erstwhile Jaunsar Bawar area of
Uttarakhand where the Hatti community has been granted the ST status.

· Both areas were part of the royal Sirmaur state.

· Age-old traditions such as animal sacrifice, folk dances and their unique attire-Thalka or
Lohiya not only make them a distinct community but also one which is yet to embrace
modernization.

· Separated by the Tons, the Trans-Giri area represents the socio-economic and socio-
educational divide between the people residing across the river.

Source: https://www.thehindu.com/todays-paper/tp-national/give-tribal-status-to-hatti-
community-hp-cm/article24912023.ece

QUESTION 66. MTU4MzMyK1Zpa2FzIFV0c2F2K3Zpa2FzdXRzYXZAaG90bWFpbC5jb20rUVVFU1RJT0


4gNjU=

IASbaba
Score:
Web: http://ilp.iasbaba.com/
45.00 /
Email: ilp@iasbaba.com
Page 45 150
2019 - Test 9- History &
Exam Title :
Cu...
Email : vikasutsav@hotmail.com
Contact :

Consider the following statements about ‘Washington Accord’

1. It is an International Agreement among bodies responsible for accrediting undergraduate


engineering degree programs

2. National Board of Accreditation, India has become the permanent signatory member of the
Washington Accord in 2018

Select the correct statements

a) 1 Only
b) 2 Only
c) Both 1 and 2
d) Neither 1 nor 2
Correct Answer: A
Your Answer: A
Explanation

Solution (a)

The Washington Accord, originally signed among six countries in 1989, is an International
Agreement among bodies responsible for accrediting undergraduate engineering degree
programs. It recognizes the substantial equivalency of programs accredited by those bodies and
recommends that graduates of programs accredited by any of the signatory bodies be
recognized by the other bodies as having met the academic requirements for entry to the
practice of engineering in the area of their jurisdiction.

The membership of Washington Accord is an international recognition of the quality of


undergraduate engineering education offered by the member country and is an avenue to bring
it into the world class category. It encourages and facilitates the mobility of engineering
graduates and professionals at international level.

National Board of Accreditation, India has become the permanent signatory member of the
Washington Accord on 13th June 2014.

The NBA accredited programs offered by the Tier -1 Institution are eligible for the recognition
of the programs by other signatories of the Washington Accord.

THINK!

· World Summit on Accreditation (WOSA)

Source: http://pib.nic.in/PressReleseDetail.aspx?PRID=1545431

QUESTION 67. MTU4MzMyK1Zpa2FzIFV0c2F2K3Zpa2FzdXRzYXZAaG90bWFpbC5jb20rUVVFU1RJT0


4gNjY=
‘Malignant Catarrhal Fever (MCF)’ is related to which of the following?

a) Chlamydia Bacteria
b) Herpes Virus
c) Gonorrhea Bacteria

IASbaba
Score:
Web: http://ilp.iasbaba.com/
45.00 /
Email: ilp@iasbaba.com
Page 46 150
2019 - Test 9- History &
Exam Title :
Cu...
Email : vikasutsav@hotmail.com
Contact :

d) Syphilis Bacteria
Correct Answer: B
Your Answer: B
Explanation

Solution (b)

Malignant Catarrhal Fever (MCF)

· It is a generally fatal disease of cattle and many other species of Artiodactyla, which occurs
following infection with certain herpesviruses of the genus Macavirus.

· It is an infectious viral disease originally from South Africa that affects cattle

· It was first spotted in Karnataka 10 years ago.

· It has neither a cure nor a vaccination that can prevent its spread.

· It is herpes virus that spreads from sheep to cattle. Once it strikes, the cattle have barely six
days to live.

· The disease has been most often described as affecting species of the subfamily Bovinae and
family Cervidae, but is also recognised in domestic pigs, giraffe and species of antelope
belonging to the subfamily Tragelaphinae.

Source: https://www.thehindu.com/todays-paper/tp-national/tp-karnataka/catarrhal-
fever-sheep-screened-in-tumakuru/article24939222.ece

QUESTION 68. MTU4MzMyK1Zpa2FzIFV0c2F2K3Zpa2FzdXRzYXZAaG90bWFpbC5jb20rUVVFU1RJT0


4gNjc=
‘MILEX-18’ is a military exercise related to which of the following Multilaterla Organisation?

a) Shanghai Cooperation Organisation


b) Organisation of Islamic Cooperation
c) Bay of Bengal Initiative for Multi-Sectoral Technical and Economic Cooperation
d) Comprehensive and Progressive Agreement for Trans-Pacific Partnership
Correct Answer: C
Your Answer: C
Explanation

Solution (c)

First BIMSTEC military exercise -MILEX-18

· It began in Pune

· Nepal and Thailand have sent observers while other countries are full participants

Source: https://www.thehindu.com/news/national/bimstec-embarrassment-for-india/
article24928750.ece

IASbaba
Score:
Web: http://ilp.iasbaba.com/
45.00 /
Email: ilp@iasbaba.com
Page 47 150
2019 - Test 9- History &
Exam Title :
Cu...
Email : vikasutsav@hotmail.com
Contact :

QUESTION 69. MTU4MzMyK1Zpa2FzIFV0c2F2K3Zpa2FzdXRzYXZAaG90bWFpbC5jb20rUVVFU1RJT0


4gNjg=
Consider the following Joint Military Exercises and participating countries

1. Garuda Shakti - India and Sri Lanka

2. Yudh Abhyas - India and Nepal

3. Nomadic Elephant - India and Bhutan

Which of the following is INCORRECTLY matched?

a) 1 and 2
b) 2 and 3
c) 1 and 3
d) All of the above
Correct Answer: D
Your Answer: A
Explanation

Solution (d)

· Garuda Shakti :: India and Indonesia

· Yudh Abhyas - India and USA

· Nomadic Elephant - India and Mongolia

QUESTION 70. MTU4MzMyK1Zpa2FzIFV0c2F2K3Zpa2FzdXRzYXZAaG90bWFpbC5jb20rUVVFU1RJT0


4gNjk=
Consider the following statements and identify the tribe

1. They are the second most populous tribe of Tripura after the Tripuris

2. They speak a dialect of Kokborok language which is of Tibeto -Burmese origin

Select the correct code:

a) Kapus
b) Dhangars
c) Brus
d) Gonds
Correct Answer: C
Your Answer: C
Explanation

Solution (c)

Brus/Reang

IASbaba
Score:
Web: http://ilp.iasbaba.com/
45.00 /
Email: ilp@iasbaba.com
Page 48 150
2019 - Test 9- History &
Exam Title :
Cu...
Email : vikasutsav@hotmail.com
Contact :

· They are one of the 21 scheduled tribes of the Indian state of Tripura.

· The Bru can be found all over the Tripura. However, they may also be found in Mizoram,
Assam, Manipur.

· They speak the Reang dialect of Kokborok language which is of Tibeto-Burmese origin and is
locally referred to as Kau Bru.

· The Bru are the second most populous tribe of Tripura after the Tripuris.

· The majority of the Reang belong to the Vaishnav school of Hinduism and claim Kshatriya
status. A growing number of Christians, exists in both Tripura and Mizoram.

· They are polytheists and believe in multiple Gods and Goddesses

DO YOU KNOW?

· In 1997, a bout of ethnic conflict forced thousands of people from the Bru tribe to leave their
homes in Mizoram & ever since, the displaced Brus have taken refuge in a town called
Kanchanpur in northern Tripura, on the Mizoram-Tripura border.

· Now, they are spread across seven refugee camps on the Jamui hills, which separate Tripura
from Mizoram and Bangladesh.

· They are set to repatriated to Mizoram after a tripartite agreement was signed between the
Centre, Tripura and Mizoram

Source: https://www.thehindu.com/news/national/other-states/return-to-mizoram-or-
give-up-sops-centre-to-brus/article24930313.ece

QUESTION 71. MTU4MzMyK1Zpa2FzIFV0c2F2K3Zpa2FzdXRzYXZAaG90bWFpbC5jb20rUVVFU1RJT0


4gNzA=
‘Red-bellied pacu ’, an alien species is a threat to the endemic aquatic ecosystem and
biodiversity in Kerala. It is endemic to

a) Amazon Basin
b) Congo Basin
c) Mississippi Basin
d) Nile Basin
Correct Answer: A
Your Answer: A
Explanation

Solution (a)

Alien fish species in Kerala post floods

News:

· Alien species of fish into waterbodies raises a threat to the endemic aquatic ecosystem and
biodiversity

IASbaba
Score:
Web: http://ilp.iasbaba.com/
45.00 /
Email: ilp@iasbaba.com
Page 49 150
2019 - Test 9- History &
Exam Title :
Cu...
Email : vikasutsav@hotmail.com
Contact :

· Globally floodwaters have proved to be a major route for the spread of invasive species and,
therefore, a precautionary approach is warranted while farming potential invasives along
floodplains

Red-bellied pacu (Piaractus brachypomus)

· It is native to the Amazon and Orinoco basins in tropical South America

· It is omnivorous, devouring everything from fruits, seeds and nuts to insects, small fish,
crustaceans and zooplankton.

Arapaima (Arapaima gigas)

· It is endemic to the Amazon and Essequibo basins of South America

· It grows up to 4.5 metres in length and 200 kg in body weight

· It feeds voraciously on fish and hence have the potential to cause serious threat to the
indigenous fish in the rivers of Kerala.

Alligator gar (Atractosteus spatula)

· It is native to the US and Mexico

· It reaches a body length up to 3 m and weighs around 137 kg

Source: https://www.thehindu.com/news/national/kerala/an-alien-invasion-post-floods-
in-state-waters/article24929272.ece

QUESTION 72. MTU4MzMyK1Zpa2FzIFV0c2F2K3Zpa2FzdXRzYXZAaG90bWFpbC5jb20rUVVFU1RJT0


4gNzE=
Which one of the following is the best description of ‘VC 11184’, that was in the news recently?

a) Ballistic Missile Tracking Ship


b) Nuclear-powered submarine
c) Torpedo launch and recovery vessel
d) Nuclear-powered aircraft carrier
Correct Answer: A
Your Answer: A
Explanation

Solution (a)

India’s first missile tracking ship (VC 11184)

· It is built by Hindustan Shipyard Limited (HSL)

· It will be India's first Ballistic Missile Tracking Ship

· It is being built for the National Technical Research Organisation, the technical intelligence
agency working directly under the supervision of the Prime Minister’s Office and the National
Security Adviser.

IASbaba
Score:
Web: http://ilp.iasbaba.com/
45.00 /
Email: ilp@iasbaba.com
Page 50 150
2019 - Test 9- History &
Exam Title :
Cu...
Email : vikasutsav@hotmail.com
Contact :

· It will be named after its induction into the Navy. For now, it is simply referred as VC 11184.

· It will propel India into an Elite Club Four Countries (US, Russia, China and France)

· It will be propelled by two 9000KW diesel engines

· It can move at an average speed of 21 knots

THINK!

· National Technical Research Organisation (NTRO)

Source: https://www.thehindu.com/sci-tech/science/indias-first-missile-tracking-ship-is-
readying-for-sea-trials/article24929012.ece

QUESTION 73. MTU4MzMyK1Zpa2FzIFV0c2F2K3Zpa2FzdXRzYXZAaG90bWFpbC5jb20rUVVFU1RJT0


4gNzI=
Which of the following statements about ‘Eritrea’ is/are correct?

a) It opens into the Red Sea


b) It is bordered by Ethiopia and Sudan only
c) Both (a) and (b)
d) Neither (a) nor (b)
Correct Answer: A
Your Answer: Unanswered
Explanation

Solution (a)

Eritrea is a country in the Horn of Africa, with its capital at Asmara. It is bordered by Sudan in
the west, Ethiopia in the south, and Djibouti in the southeast. The northeastern and eastern
parts of Eritrea have an extensive coastline along the Red Sea.

IASbaba
Score:
Web: http://ilp.iasbaba.com/
45.00 /
Email: ilp@iasbaba.com
Page 51 150
2019 - Test 9- History &
Exam Title :
Cu...
Email : vikasutsav@hotmail.com
Contact :

QUESTION 74. MTU4MzMyK1Zpa2FzIFV0c2F2K3Zpa2FzdXRzYXZAaG90bWFpbC5jb20rUVVFU1RJT0


4gNzM=
Consider the following statements with respect to Pradhan Mantri Annadata Aay Sanrakshan A
bhiyan (PM-AASHA)

1. The primary objectives of PM-AASHA are to attract investments in irrigation system at field
level, develop and expand cultivable land in the country, enhance ranch water use in order to
minimize wastage of water, enhance crop per drop by implementing water-saving technologies
and precision irrigation.

2. It is an umbrella scheme incorporating ongoing schemes of the Ministry like Mega Food
Parks, Integrated Cold Chain and Value Addition Infrastructure, Food Safety and Quality
Assurance Infrastructure, etc.

3. It ensures that farmers growing oilseeds, pulses and copra actually get the minimum support
price (MSP) they are promised for their crops every year.

Select the correct statements

a) 1 and 2
b) 1 and 3
c) 3 Only
d) 2 and 3
Correct Answer: C
Your Answer: C
Explanation

Solution (c)

Pradhan Mantri Annadata Aay Sanrakshan Abhiyan (PM-AASHA)

· It is aimed at ensuring remunerative prices to the farmers for their produce as announced in
the Union Budget for 2018.

· It ensures that farmers growing oilseeds, pulses and copra actually get the minimum support
price (MSP) they are promised for their crops every year.

· Apart from the ₹15,053 crore to be spent over a two-year period to implement the scheme, the
Cabinet approved an additional government credit guarantee of ₹16,550 crore for agencies
undertaking procurement.

It includes the mechanism of ensuring remunerative prices to the farmers and is


comprised of (Three Components/Sub-Schemes)

· Price Support Scheme (PSS)

· Price Deficiency Payment Scheme (PDPS)

· Pilot of Private Procurement & Stockist Scheme (PPPS).

Private sector in procurement operation

IASbaba
Score:
Web: http://ilp.iasbaba.com/
45.00 /
Email: ilp@iasbaba.com
Page 52 150
2019 - Test 9- History &
Exam Title :
Cu...
Email : vikasutsav@hotmail.com
Contact :

· It has been decided that for oilseeds, states have the option to roll out Private Procurement
Stockist Scheme (PPSS) on pilot basis in selected district/APMC(s) of district involving the
participation of private stockiest.

· The pilot district/selected APMC(s) of district will cover one or more crop of oilseeds for which
MSP is notified.

· Since this is akin to PSS, in that in involves physical procurement of the notified commodity, it
shall substitute PSS/PDPS in the pilot districts.

PSS

· In Price Support Scheme (PSS), physical procurement of pulses, oilseeds and Copra will be
done by Central Nodal Agencies with proactive role of State governments.

· It is also decided that in addition to NAFED, Food Cooperation of India (FCI) will take up PSS
operations in states /districts.

· The procurement expenditure and losses due to procurement will be borne by Central
Government as per norms.

PDPS

· Under Price Deficiency Payment Scheme this scheme (PDPS), it is proposed to cover all
oilseeds for which MSP is notified.

· In this direct payment of the difference between the MSP and the selling/modal price will be
made to pre-registered farmers selling his produce in the notified market yard through a
transparent auction process.

· All payment will be done directly into registered bank account of the farmer.

· This scheme does not involve any physical procurement of crops as farmers are paid the
difference between the MSP price and Sale/modal price on disposal in notified market.

· The support of central government for PDPS will be given as per norms.

· It has been framed on the lines of the Madhya Pradesh government’s Bhavantar Bhugtan
Yojana (BBY) to protect oilseed farmers.

DO YOU KNOW?

· Other existing schemes of the Department of Food and Public Distribution for the procurement
of paddy, wheat and nutri-cereals/coarse grains and of the Ministry of Textile for cotton and jute
will be continued to provide MSP to farmers for these crops.

· The selected private agency shall procure the commodity at the MSP in the notified markets
during the notified period from the registered farmers in consonance with the Private
Procurement Stockist Scheme guidelines, whenever prices in the market fall below the notified
MSP and whenever authorised by the State/UT government to enter the market; a maximum
service charge of up to 15 per cent of the notified MSP will be payable.

· The Food Corporation of India, the government’s nodal agency for procurement and
distribution of foodgrains, already procures wheat and rice at MSP for supply through ration
shops and welfare schemes.

IASbaba
Score:
Web: http://ilp.iasbaba.com/
45.00 /
Email: ilp@iasbaba.com
Page 53 150
2019 - Test 9- History &
Exam Title :
Cu...
Email : vikasutsav@hotmail.com
Contact :

Source: https://www.thehindu.com/todays-paper/new-procurement-policy-cleared/
article24939202.ece

QUESTION 75. MTU4MzMyK1Zpa2FzIFV0c2F2K3Zpa2FzdXRzYXZAaG90bWFpbC5jb20rUVVFU1RJT0


4gNzQ=
‘Global Burden of Disease Study (GBD)’ is a comprehensive regional and global research
program of disease burden that assesses mortality and disability from major diseases, injuries,
and risk factors. It is based out of

a) Médecins Sans Frontières


b) Institute for Health Metrics and Evaluation
c) World Health Organization
d) Lancet Global Health
Correct Answer: B
Your Answer: B
Explanation

Solution (b)

GBD is based out of the Institute for Health Metrics and Evaluation (IHME) at the University of
Washington and funded by the Bill and Melinda Gates Foundation.

‘India State-level Disease Burden Initiative’ report

· It is a joint initiative of the Indian Council of Medical Research (ICMR), the Public Health
Foundation of India (PHFI) and the Institute for Health Metrics and Evaluation (IHME) in
collaboration with the Ministry of Health and Family Welfare, along with experts and
stakeholders associated with over 100 Indian institutions.

· The study was based on an analysis of all identifiable epidemiological data from India between
1990 and 2016, as part of the Global Burden of Disease study.

Highlights

· Indians have registered a 50% increase in the prevalence of ischemic heart disease and stroke
over the period from 1990 to 2016

· Number of diabetes cases climbed from 26 million to 65 million.

· Number of people ailing from chronic obstructive lung disease went up from 28 million to 55
million

· Punjab has been ranked at the top for the burden of ischemic heart disease, followed by Tamil
Nadu, and vice-versa for diabetes.

· West Bengal topped with the largest number of stroke cases followed by Odisha.

· Kerala was ranked at the top for the burden of cancer, followed by Assam.

· The highest rate of increase in ischemic heart disease and diabetes is in the less developed
States of India.

IASbaba
Score:
Web: http://ilp.iasbaba.com/
45.00 /
Email: ilp@iasbaba.com
Page 54 150
2019 - Test 9- History &
Exam Title :
Cu...
Email : vikasutsav@hotmail.com
Contact :

· The proportional contribution of cancers to disease burden in India has doubled since 1990,
but the incidence of individual cancers varies widely between the States

Suicides

· Suicide is the leading cause of death in the 15-39 years age group in India.

· Thirty-seven per cent of such deaths among women globally occur in India

· Suicide Death Rate (SDR) in India is higher than expected for its socio-demographic index
level, especially for women, with substantial variations in the magnitude and men-to-women
ratio between the states

Source: https://www.thehindu.com/sci-tech/health/indias-health-report-reads-worse/
article24937845.ece

IASbaba
Score:
Web: http://ilp.iasbaba.com/
45.00 /
Email: ilp@iasbaba.com
Page 55 150
2019 - Test 9- History &
Exam Title :
Cu...
Email : vikasutsav@hotmail.com
Contact :
Review in Hindi
QUESTION 1.
���टश�काल�के�दौरान�भारत�म��शु���ए�जनजातीय��व�ोह��के�बारे�म���न�न�ल�खत�कथन��पर��वचार�क��जए।�

1. इनम���ह�सा�उन�बाहरी�सा�कार��और��ापा�रय��के���त��नद� �शत�थी��ज�ह��औप�नवे�शक�सरकार�के��व�तार��के��प�म��दे खा�गया�था।�

2. इनक��बारंबारता, आ�ामकता�और��ह�सा�सभी�आंदोलन��म��सबसे�अ�धक�थी।�

3. जनजातीय�पहचान�अथवा�न�लीय�संबंध�उनक��एकता�का��मुख�आधार�बने।�

सही�कूट�चु�नए�

a) केवल�1 और�2
b) केवल�2 और�3
c) केवल�1 और�3
d) उपयु���सभी�
Correct Answer: D
Your Answer:
Explanation

Solution (d)

���टश�काल�के�दौरान�भारत�म��शु���ए�जनजातीय��व�ोह��क��बारंबारता, आ�ामकता�और��ह�सा�सभी�आंदोलन��म��सबसे�अ�धक�थी।�

�व�भ��जनजातीय��व�ोह��म��समय�और��थान�का�अंतर�होने�के�प�ात�भी�कुछ�साझी��वशेषताएं�थी।�

1) जनजातीय�पहचान�अथवा�न�लीय�संबंध�इन�समूह���ारा��द�श�त�क��गई�एकता�के�पीछे �के�त�व�थे।�हालां�क, सभी�बाहरी�लोग��को�श�ु


नह��समझा�गया�था: वे�गरीब�जो�शारी�रक��म�अथवा�कोई�धंधा�करके�गुजर�करते�थे�और��ाम��म���जनक��सामा�जक/आ�थ�क��प�से�सहायक
भू�मका�थी, को�अकेले�छोड़��दया�गया�था�; इनम���ह�सा�उन�सा�कार��और��ापा�रय��के���त��नद� �शत�थी��ज�ह��औप�नवे�शक�सरकार�के
�व�तार��के��प�म��दे खा�गया�था।�

2) ‘�वदे शी�सरकार’ �ारा�लादे �गए�कानून�असंतोष�का�एक�आम�कारण�थे��य��क�इसे�जनजा�तय��के�सामा�जक-आ�थ�क�ढ़ांचे�को�तोड़ने�के


एक��यास�के��प�म��दे खा�गया�था।�

3) अ�धकतर�आंदोलन��का�नेतृ�व�मसीहा�जैसा�माने�जाने�वाले�उन����य���ारा��कया�गया�था��ज�ह�ने�लोग��को��व�ोह�करने�के��लए
�ो�सा�हत��कया�और�यह�वादा��कया��क�वे�उन�तकलीफ��को�समा�त�कर�द� गे�जो�बाहरी�लोग��ने�उ�ह��द��ह�।�

4) जनजातीय��व�ोह��का�शु��से�ही��वफल�होना�तय�था��य��क��जन�ह�थयार��के�साथ�वे�लड़े, वे�उनके��वरो�धय���ारा��योग��कए�गए
आधु�नक�ह�थयार��तथा�तकनीक��क��अपे�ा�अ�च�लत�थे।�

QUESTION 2.
�न�न�ल�खत�म��से�कौन�सही�सुमे�लत�है�?

�व�ोह��थान�

1. पहा�ड़या��व�ोह�A) राजमहल�पहा�ड़यां�

2. कोया��व�ोह�B) छोटानागपुर�पठार�

3. कोल��व�ोह�C) पूव��गोदावरी�माग��

सही�कूट�चु�नए�

a) केवल�1

IASbaba
Score:
Web: http://ilp.iasbaba.com/
45.00 /
Email: ilp@iasbaba.com
Page 56 150
2019 - Test 9- History &
Exam Title :
Cu...
Email : vikasutsav@hotmail.com
Contact :

b) केवल�1 और�2
c) केवल�2 और�3
d) केवल�1 और�3
Correct Answer: A
Your Answer:
Explanation

Solution (a)

अं�ेज���ारा�उनके��े���म���कया�गया��व�तार�1778 म��राजमहल�क��पहा�ड़य��पर��नवास�करने�वाले�लड़ाकू�पहा�ड़या�लोग���ारा��व�ो
ह�करने�का�कारण�बना।�अं�ेज��को�शां�त�बनाए�रखने�के��लए�मजबूरन�उनके��े��को�दामन-ए-कोह�घो�षत�करना�पड़ा।�

अ�य�जनजा�तय��के�साथ�ही�कोल�छोटानागपुर�के��नवासी�ह��।�इसम��रांची, �स�हभूम, हजारीबाग, पलामू�और�मनभूम�के�प��मी��ह�से


शा�मल�ह�।�1831 क��सम�या�बड़े��तर�पर�कोल�सरदार��क��जमीन���ह��, �सख�और�मु��लम��कसान��जैसे�उन�बाहरी�लोग��तथा�सा�कार��को
ह�तां�रत�करने�के�चलते�आरंभ��ई�जो�उ�पीड़क�थे�और�अ�धक�कर�क��माँग�करते�थे।�इसके�अ�त�र�, अं�ेज��क���या�यक�और�राज�व
नी�तय��ने�कोल�लोग��क��पारंप�रक�सामा�जक�दशा�को�बुरी�तरह��भा�वत��कया।�इससे�कोल�लोग��म��असंतोष�पैदा��आ�और�1831 म�
उ�ह�ने�कोल��व�ोही�बु�ो�भगत�के�नेतृ�व�म��एक�हजार�के�लगभग�बाहरी�लोग��को�मार��दया�अथवा��ज�दा�जला��दया।�

पूव��गोदावरी�माग��( आधु�नक�आं��दे श) के�कोया�लोग��ने�ख�ड�सरा�सरदार��के�साथ��मलकर�1803, 1840, 1845, 1858,


1861 तथा�1862 म���व�ोह��कया।�उ�ह�ने�1879-80 म��एक�बार�पुन: टोमा�सोरा�के�नेतृ�व�म���व�ोह��कया।�उनक���शकायत��म��पु�लस
और�सा�कार���ारा�उ�पीड़न, नए��व�नयम�तथा�वन��े���पर�पारंप�रक�अ�धकार��के�मनाही�शा�मल�थे।�टोमा�सोरा�क��मृ�यु�के�प�ात�एक�अ�य
आंदोलन�1886 म��राजा�अनंत�यर��ारा�संग�ठत��कया�गया।�

QUESTION 3.
�न�न�ल�खत�आंदोलन��को�उनके�काल�मानुसार��व��थत�क��जए-

1. स�यासी��व�ोह�

2. पाइका��व�ोह�

3. अहोम��व�ोह�

4. कूका��व�ोह�

सही�कूट�चु�नए�

a) 1-2-3-4
b) 2-1-3-4
c) 1-3-2-4
d) 1-3-4-2
Correct Answer: A
Your Answer:
Explanation

Solution (a)

स�यासी��व�ोह�(1763-1800) — �बहार�और�बंगाल: मंजू�शाह, मूसा�शाह, भवानी�पाठक�और�दे बी�चौधरानी�इसके�कुछ�मह�वपूण�


नेता�थे।�

पाइका��व�ोह�(1817)— ओ�डशा�; ब�शी�जगबंधु��ब�ाधर, मुकुंद�दे व�और�द�नबंधु�संतरा�मह�वपूण��नेता�थे।�

अहोम��व�ोह�(1828)— असम: गोमधर�कुँवर�तथा�महाराजा�पुरंदर��स�ह�का�नेतृ�व�।�नर���गदाधर��स�ह�और�कुमार��पचंद�अ�य�नेता�थे।�

IASbaba
Score:
Web: http://ilp.iasbaba.com/
45.00 /
Email: ilp@iasbaba.com
Page 57 150
2019 - Test 9- History &
Exam Title :
Cu...
Email : vikasutsav@hotmail.com
Contact :

कूका��व�ोह�(1840-1872)— पंजाब�; यह�भगत�जवाहर�मल��ारा�आरंभ��कया�गया�एक�धा�म�क�आ�दोलन�था�जो�राजनी�तक


आंदोलन�म��बदल�गया।�एक���यात�नेता�राम��स�ह�को�रंगून��नवा��सत�कर��दया�गया।�

QUESTION 4.
�न�न�ल�खत�म��से�कौन�सही�सुमे�लत�नह��ह��?

�व�ोह�के�क���नेता�

1. कानपुर�A) नाना�साहेब�

2. लखनऊ�B) बेगम�हजरत�महल�

3. �बहार�C) मौलवी�अहम��लाह�

4. फैजाबाद�D) कुंवर��स�ह�

गलत�कूट�चु�नए�

a) केवल�1 और�3
b) केवल�2 और�4
c) केवल�3 और�4
d) केवल�1 और�4
Correct Answer: C
Your Answer:
Explanation

Solution (c)

�व�ोह��के�क���और�उनके�नेता�

�द�ली�- जनरल�ब�त�खान�

कानपूर�– नाना�साहेब�

लखनऊ�- बेगम�हजरत�महल�

बरेली�– खान�बहा�र�

�बहार�– कुँवर��स�ह�

फैजाबाद�- मौलवी�अहम��लाह�

झाँसी�– रानी�ल�मीबाई�

बागपत�– शाह�मल�

QUESTION 5.
�न�न�ल�खत�म��से��क�ह��1857 के��व�ोह�क��असफलता�के�कारण��म���गना�जाता�है�?

1. सुसंगत��वचारधारा�और�राजनी�तक�प�र�े�य�का�अभाव।�

2. �ह��-मु��लम�एकता�का�अभाव�असफलता�का�एक��मुख�कारण�था।�

3. भारतीय�जनता�के�कुछ�मह�वपूण��तबक��का����टश�अ�धका�रय��को�समथ�न।�

IASbaba
Score:
Web: http://ilp.iasbaba.com/
45.00 /
Email: ilp@iasbaba.com
Page 58 150
2019 - Test 9- History &
Exam Title :
Cu...
Email : vikasutsav@hotmail.com
Contact :

सही�कूट�चु�नए�

a) केवल�1 और�2
b) केवल�1 और�3
c) केवल�2 और�3
d) उपयु���सभी�
Correct Answer: B
Your Answer:
Explanation

Solution (b)

�व�ोह�क��असफलता�के�कारण�

1) सी�मत��े�ीय�और�सामा�जक�आधार।�

2) भारतीय�जनता�के�कुछ�मह�वपूण��तबक��का����टश�अ�धका�रय��को�समथ�न।�

3) अं�ेज��के�मुकाबले�संसाधन��का�अभाव।�

4) सम�वय�और�एक�क���य�नेतृ�व�का�अभाव।�

5) सुसंगत��वचारधारा�और�राजनी�तक�प�र�े�य�का�अभाव।�

�व�ोह�क��संपूण��अव�ध�के�दौरान, सभी��तर��( लोग�, �सपा�हय�, नेता�) पर��ह����और�मु��लम��के�बीच�पूण��सहयोग�था�।�सभी


�व�ो�हय��ने�एक�मु��लम�बहा�र�शाह�जफ़र�को�अपना�स�ाट��वीकार��कया�और��ह����सपा�हय��क��पहली���त��या�थी��द�ली�(जो��क�मुगल�
क��सा�ा��यक�राजधानी�थी) क��ओर�माच��करना।�

QUESTION 6.

�न�न�ल�खत�म��से�कौन�से�कारक��के�चलते�19 व��सद��के�भारत�म��सामा�जक�सुधार�आंदोलन�आरंभ��ए�?

1. उ�ीसव��सद��म��भारतीय�समाज�धा�म�क�अंध�व�ास��और�सामा�जक���ढ़वाद��ारा��न�म�त�कुच��म��जकड़ा�था।�

2. सती, बाल-�ववाह�जैसी�अनेक�कु�था��के�मा�यम�से�म�हला��पर�आरो�पत�पूरी��वपदा�और�अपमान।�

3. आधु�नक�प��मी�सं�कृ�त�के��भाव�एवं�एक��वदे शी�श����ारा�हरा��दए�जाने�के�भाव�ने�नए�जाग�कता�को�ज�म��दया।�

सही�कूट�चु�नए�

a) केवल�1 और�2
b) केवल�2 और�3
c) केवल�1 और�3
d) उपयु���सभी�
Correct Answer: D
Your Answer:
Explanation

Solution (d)

वे�कारक��ज�ह�ने�सुधार�आंदोलन��को�उभार��दया�

1) भारतीय�जमीन�पर�औप�नवे�शक�सरकार�क��उप��थ�त।�

IASbaba
Score:
Web: http://ilp.iasbaba.com/
45.00 /
Email: ilp@iasbaba.com
Page 59 150
2019 - Test 9- History &
Exam Title :
Cu...
Email : vikasutsav@hotmail.com
Contact :

2) भारतीय�समाज�को�अपनी�जकड़�म���लए�अनेक�बुराइयाँ�— ��ढ़वाद�, अंध�व�ास�, ब�दे ववाद�, मू�त��पूजा�, म�हला��क���गरी��ई�दशा�


, शोषक�जातीय�सोपान।�

3) �श�ा�का��सार�और��व��क��बढ़���ई�जाग�कता।�

4) आधु�नक�प��मी�सं�कृ�त�का��भाव�एवं�एक��वदे शी�श����ारा�हरा��दए�जाने�का�भाव।�

5) 19 व��सद��के�दौरान�रा�वाद�का�उभरता��वार�तथा�लोकतं�।�

QUESTION 7.
�न�न�ल�खत�म��से��कसने�भारत�म�हला�प�रषद�क���थापना�क��?

a) सरला�दे वी�चौधरानी�
b) रमाबाई�रानाडे�
c) पं�डता�रमाबाई�सर�वती�
d) ताराबाई��ेमचंद�
Correct Answer: B
Your Answer:
Explanation

Solution (b)

रमाबाई�रानाडे�ने�1904 म��बॉ�बे�म��नेशनल�सोशल�कां��स�नामक��पतृ�संगठन�के�अंतग�त�लेडीज�सोशल�कां��स�( भारत�म�हला�प�रषद�) क�


�थापना�क�।�

QUESTION 8.
भारतीय�इ�तहास�के�उदारवाद��चरण�म��रा�वा�दय��के��वषय�म���न�न�ल�खत�कथन��पर��वचार�क��जए।�।

1. रा�ीय�आंदोलन�के�उदारवाद��चरण�का��ापक�सामा�जक�आधार�था�और�जनसमूह�ने�एक�स��य�भू�मका�अदा�क�।�

2. उ�ह�ने�लोग��को�राजनी�तक�काय��का���श�ण��दया�और�आधु�नक��वचार��को�लोक��य�बनाया।�

सही�कूट�चु�नए�

a) केवल�1 और�2
b) केवल�2 और�3
c) केवल�1 और�3
d) केवल�2
Correct Answer: B
Your Answer:
Explanation

Solution (b)

आरं�भक�रा�वा�दय��ने�रा�ीय�चेतना�को�जगाने�म��एक�महान�भू�मका��नभाई, भले�ही�वे�एक�बड़े�जनसमूह�को�अपनी�ओर�नह��ख�च�पाएं�ह�।
रा�ीय�आंदोलन�के�उदारवाद��चरण�का�सामा�जक�आधार�संक�ण��था�और�जनसमूह�क��भू�मका�इसम���न���य�रही।�

1) उ�ह�ने�उस�समय�क��सवा��धक��ग�तशील�श��य��का���त�न�ध�व��कया।�

2) उ�ह�ने�लोग��को�राजनी�तक�काय��का���श�ण��दया�और�आधु�नक��वचार��को�लोक��य�बनाया।�

3) उ�ह�ने�औप�नवे�शक�शासन�के�आधारभूत��प�से�शोषक�च�र��को�उजागर��कया�और�इस��कार�इसक��नै�तक�न�व�को�कमजोर�बनाया।�

IASbaba
Score:
Web: http://ilp.iasbaba.com/
45.00 /
Email: ilp@iasbaba.com
Page 60 150
2019 - Test 9- History &
Exam Title :
Cu...
Email : vikasutsav@hotmail.com
Contact :

4) उनका�राजनी�तक�काय��कठोर�स�चाइय��पर�आधा�रत�था�न��क��छछली�भावना�, धम��आ�द�पर।�

5) उ�ह�ने��या�यक�और�काय�कारी�काय��को�अलग��कए�जाने�का�आ�ान��कया।�

QUESTION 9.
�न�न�ल�खत�म��से�कौन�से�कारक�आधु�नक�रा�वाद�क��संवृ���के��लए�उ�रदायी�थे�?

1. �ेस�क��वृ���ने��वशासन, लोकतं�, नाग�रक�अ�धकार��तथा�औ�ो�गक�करण�जैसे�आधु�नक��वचार��के��सार�म��सहायता�क�।�

2. म�यवग�य�बु��जी�वय��ने�भारतीय�राजनी�तक�संघ��को��भावी�नेतृ�व��दान��कया।�

3. सामा�जक-धा�म�क�सुधार�आंदोलन��ने��व�भ��तबक��को�साथ�लाने�म��मह�वपूण��भू�मका�अदा�क�।�

सही�कूट�चु�नए�

a) केवल�1 और�2
b) केवल�2 और�3
c) केवल�1 और�3
d) उपयु���सभी�
Correct Answer: D
Your Answer:
Explanation

Solution (d)

आधु�नक�रा�वाद�क��संवृ���के��लए�उ�रदायी�कारक�

1) भारतीय�और�औप�नवे�शक��हत��के��वरोधाभास�को�समझना�

2) दे श�का�राजनी�तक, �शास�नक�तथा�आ�थ�क�एक�करण�

3) प��मी��वचार�और��श�ा�

4) �ेस�और�सा�ह�य�क��भू�मका�

5) भारत�के�भूतकाल�के�ऐ�तहा�सक�शोध��क��पुनख�ज�

6) म�यवग�य�बु��जी�वय��का�उभरना�

7) �व�भर�के�त�कालीन�आंदोलन��का��भाव�

8) शासक��क����त��यावाद��नी�तयां�तथा�न�लीय�दं भ�

QUESTION 10.
�न�न�ल�खत�संगठन��पर��वचार�क��जए-

1. अ�खल�भारतीय�अनुसू�चत�जा�त�फेडरेशन�

2. ऑल�इं�डया��ड�े�ड��लासेज�लीग�

3. ऑल�इं�डया��ड�े�ड��लासेज�एसो�सएशन�

4. ब�ह�कृत��हतका�रणी�सभा�

डॉ. बी. आर. अंबेडकर��न�न�ल�खत�म��से��कन�संगठन��के�साथ�संब��थे�?

IASbaba
Score:
Web: http://ilp.iasbaba.com/
45.00 /
Email: ilp@iasbaba.com
Page 61 150
2019 - Test 9- History &
Exam Title :
Cu...
Email : vikasutsav@hotmail.com
Contact :

a) केवल�1,2 और�3
b) केवल�1,2 और�4
c) केवल�1 और�4
d) केवल�1,3 और�4
Correct Answer: C
Your Answer:
Explanation

Solution (c)

बाबासाहेब�अंबेडकर, �ज�ह�ने�अपने�बचपन�म��जातीय�भेदभाव�का�सबसे�भयावह��प�दे खा�था, ने�जीवन�भर�उ�च�जा�त�क���नरंकुशता�के


�व���लड़ाई�लड़ी।�उ�ह�ने�अ�खल�भारतीय�अनुसू�चत�जा�त�फेडरेशन�का�संगठन��कया�जब�क�द�मत�वग��के�अ�य�ब�त�सारे�नेता��ने�ऑल
इं�डया��ड�े�ड��लासेज�एसो�सएशन�क���थापना�क�।�

डॉ. अंबेडकर�1924 म��सरकार�के�सम��द�लत��क��क�ठनाइय��और��शकायत��को�सामने�लाने�के��लए�ब�ह�कृत��हतका�रणी�सभा�क�


�थापना�क�।�इसका��स�ांत�था�: ‘ �श��त�बनो, आंदोलन�कर��और�संग�ठत�रहो�’ ।

ऑल�इं�डया��ड�े�ड��लासेज�लीग�क���थापना�बाबू�जगजीवन�राम��ारा�क��गई�थी।�

QUESTION 11.
�कसे�भारतीय�पुनजा�गरण�के��पता�के�तौर�पर�जाना�जाता�है�?

a) राजा�राम�मोहन�राय�
b) दयानंद�सर�वती�
c) �यो�तबा�फुले�
d) ई�र�चं���व�ासागर�
Correct Answer: A
Your Answer:
Explanation

Solution (a)

राजा�राम�मोहन�राय�(1772-1833) को��ाय: भारतीय�पुनजा�गरण�के��पता�और�आधु�नक�भारत�के��नमा�ता�के��प�म��जाना�जाता�है, और


वह�एक�ब�मुखी���तभा�के�धनी�����थे।�

राजा�राम�मोहन�राय�का�आधु�नक�वै�ा�नक�सोच�और�मानव�ग�रमा�व�सामा�जक�समता�म���व�ास�था।�उ�ह�ने�एके�रवाद�म��आ�था��दखाई।
उ�ह�ने�एके�रवा�दय��को�उपहार�(1809) नामक�पु�तक��लखी�और�अपना�यह��व�ास��स��करने�के��लए��क��ह����ंथ�एके�रवाद�का
समथ�न�करते�ह�, वेद��तथा�पाँच�उप�नषद��का�बां�ला�भाषा�म��अनुवाद��कया।�

QUESTION 12.
�हा�समाज�क���न�न�ल�खत�म��से�कौन�सी��वशेषताएं�थी�?

1. इसने�कम��के��स�ांत�और�आ�मा�के�दे हांतरण�को�लेकर�एक��न��त��ख�अपनाया।�

2. इसने�बताया��क�केवल��ह���धम���ंथ�ही�अं�तम�स�ा�माने�जा�सकते�ह��और�वे�मानव�तक��और�अंतःकरण�से�बढ़कर�ह��।

3. इसने�धा�म�क��ंथ��क���ा�या�हेतु�पुरो�हत�वग��क��आव�यकता�को�खा�रज��कया।�

सही�कूट�चु�नए�

a) केवल�1 और�2
b) केवल�1 और�3

IASbaba
Score:
Web: http://ilp.iasbaba.com/
45.00 /
Email: ilp@iasbaba.com
Page 62 150
2019 - Test 9- History &
Exam Title :
Cu...
Email : vikasutsav@hotmail.com
Contact :

c) केवल�2 और�3
d) केवल�3
Correct Answer: D
Your Answer:
Explanation

Solution (d)

राजा�राम�मोहन�राय�ने�अग�त�1828 म���हा�सभा�क���थापना�क��; �जसका�नाम�बदलकर�बाद�म���हा�समाज�कर��दया�गया।�इस�सभा�के


मा�यम�से�वे�अपने��वचार��और��मशन�को�सं�थागत��प�दे ना�चाहते�थे।�

यह�समाज�“ उस�सनातन, खोज�से�परे, अप�रवत�नीय�क��पूजा�और�उपासना�करने�को�सम�प�त�था�जो�इस���ांड�का�रच�यता�और�र�क�है�”


�हा�समाज�क���वशेषता��का�सार�इस��कार�है�—

1) इसने�ब�दे वतावाद�तथा�मू�त��पूजा�को�खा�रज��कया�;

2) इसने��द��अवतार��म��आ�था�को�बाहर��कया�;

3) इसने�इस�बात�को�खा�रज��कया��क��कसी�धम���ंथ�को�ही�अं�तम�स�ा�माना�जा�सकता�है�और�वह�मानव�तक��और�अंतःकरण�से�बढ़कर�ह��।
;

4) इसने�कम��के��स�ांत�और�आ�मा�के�दे हांतरण�को�लेकर�कोई��न��त��ख�नह��अपनाया�और�इसे�मानना�अथवा�न�मानना����गत��प�से
��समा�जय��पर�छोड़��दया�;

5) इसने�जा�त��व�था�क��आलोचना�क�।�

QUESTION 13.
�न�न�ल�खत�म��से��कसने�यह��वचार��दया��क�‘तक�वाद�ही�हमारा��श�क�है’ ?

a) अ�य�कुमार�द��
b) राजा�राम�मोहन�राय�
c) �वामी��ववेकानंद�
d) सैयद�अहमद�खान�
Correct Answer: A
Your Answer:
Explanation

Solution (a)

अ�य�कुमार�द��ने�घो�षत��कया��क�‘तक�वाद�ही�हमारा��श�क�है’ , उ�ह�ने�कहा��क�ऐसा�करते�समय�सभी��ाकृ�तक�और�सामा�जक
प�रघटना��को�पूण�त: यां��क����या���ारा��व�े�षत��कया�व�समझा�जा�सकता�है।�उ�ह�ने�इस��कार�परंपरा��को�समझने�के��लए
ता�क�क�उपागम�का��योग��कया�; उ�ह�ने�त�कालीन�सामा�जक-धा�म�क��था��को�सामा�जक�उपयो�गता�के����कोण�से�मू�यां�कत��कया�और
आ�था�को�तक��से���त�था�पत�कर��दया।�

उदाहरण�के��लए, अ�य�कुमार�द��ने�बाल��ववाह�के��व���अपने����कोण�को�समथ�न�दे ने�के��लए��च�क�सक�य�राय�सामने�रखी।�भूतकाल


के�संदभ��का��योग�केवल�एक�सहायक�और�उपकरण�के�तौर�पर��कया�गया।�न�तो�भूतकाल�को�पूरी�तरह�से�जी�वत�करने�और�न�ही�परंपरा�के
साथ�पूण���व�छे द�का����कोण�अपनाया�गया।�

QUESTION 14.
�न�न�ल�खत�म��से��कसने�त�वबो�धनी�सभा�का�नेतृ�व��कया�?

IASbaba
Score:
Web: http://ilp.iasbaba.com/
45.00 /
Email: ilp@iasbaba.com
Page 63 150
2019 - Test 9- History &
Exam Title :
Cu...
Email : vikasutsav@hotmail.com
Contact :

a) दे वे��नाथ�टै गोर�
b) र�ब��नाथ�टै गोर�
c) एम.जी. रानाडे�
d) केशव�च���सेन�
Correct Answer: A
Your Answer:
Explanation

Solution (a)

त�वबो�धनी�सभा�

सभा�का�मु�य�उ�े �य�वेद��के�अं�तम�भाग�उप�नषद��(वेदांत) पर�आधा�रत��ह���धम��के�एक�अ�धक�ता�क�क�एवं�मानवीय����कोण�को�बढ़ावा


दे ना�था।�अवेद�वेदांत�क��शा��ीय�शाखा�को�अनै�तक�और�आ�मो�सग��मानने�वाले��मशन�रय��क��कलक�ा�म��बढ़ती�सं�या�को�दे खते��ए,
त�वबो�धनी�सभा�ने�इस�‘अ�च�लत’ सं�करण�से��वयं�को��र�कर�खुद�को�और�अपनी�सुधरी�आ�था�को�आलोचना�से�बचाने�का�उ�े �य�बनाया।�

इसक���थापना�दे वे��नाथ�टै गोर��ारा�1839 म�����समाज�के�एक�अलग��ए�समूह�के��प�म��क��गई�थी।�1859 म��, त�वबो�धनी�सभा�को


भंग�कर�पुन: उसका����समाज�म���वलय�कर��दया�गया।�

QUESTION 15.
सामा�जक�सुधार�संगठन��के�बारे�म���न�न�ल�खत�कथन��पर��वचार�क��जए�

1. �ाथ�ना�समाज�ने��श�ा�और�अनुनय�का�सहारा��लया�न��क��ह�����ढ़वाद�से�ट�कर�लेने�का।�

2. यंग�बंगाल�आंदोलन�ने�महान��ांसीसी��ां�त�से��ेरणा��ा�त�क�।�

3. ई�र�चं���व�ासागर�सं�कृत�अ�ययन�का��व-आरो�पत�अलगाव�समा�त�करने�के��लए�सं�कृत�महा�व�ालय�म��प��मी��वचार�लेकर�आए।�

गलत�कूट�चु�नए�

a) केवल�1 और�2
b) केवल�2 और�3
c) केवल�1 और�3
d) उपरो��म��से�कोई�नह��
Correct Answer: D
Your Answer:
Explanation

Solution (d)

1867 म��, केशब�च���सेन�ने�बॉ�बे�म���ाथ�ना�समाज�क���थापना�करने�म��आ�माराम�पांडुरंग�क��सहायता�क�।��ाथ�ना�सभा�महारा��के�भ��


स��दाय�से�काफ��जुड़ा��आ�था।��ाथना�समाज�ने��श�ा�और�अनुनय�का�सहारा��लया�न��क��ह�����ढ़वाद�से�ट�कर�लेने�का।�एक�चार�सू�ीय
सामा�जक�एज�डा�भी�था�: (i) जा�त��व�था�को�खा�रज�करना�, (ii) म�हला��श�ा�, (iii) �वधवा�पुन�व�वाह�, तथा�(iv) पु�ष��और
म�हला��दोन��के��लए��ववाह�क��आयु�को�बढ़ाना।�

1820 के�दशक�के�आ�खरी�दौर�और�1830 के�दशक�के�शु�आती�दौर�म��, बंगाल�के�युवा��म��एक�प�रवत�नकारी�और�बौ��क��वृ�त�उभरी


, �जसे�‘यंग�बंगाल�आंदोलन’ के�नाम�से�जाना�गया।�महान��ांसीसी��ां�त�से��ेरणा�लेकर�डेरो�जयो�ने�अपने��श�य��को�मु��और�ता�क�क�ढं ग
से�सोचने�, सभी�स�ा��पर�सवाल�उठाने�, �वत��ता, समानता�तथा�आजाद��से��ेम�करने�, व�अपकष���रवाज��तथा�परंपरा��का��वरोध
करने�हेतु��े�रत��कया।�

महान��व�ान�और�सुधारक��व�ासागर�के��वचार�भारतीय�और�प��मी��वचार��का�एक�सुखद��म�ण�थे।�वह�धम��ंथ��पर�पुरो�हत�वग��के
एका�धकार�को�समा�त�करने�हेतु���तब��थे�तथा�इस�हेतु�उ�ह�ने�सं�कृत�महा�व�ालय�को�गैर-�ा�ण��के��लए�खोल��दया।�वे�सं�कृत�अ�ययन
का��व-आरो�पत�अलगाव�समा�त�करने�के��लए�सं�कृत�महा�व�ालय�म��प��मी��वचार�लेकर�आए।�

IASbaba
Score:
Web: http://ilp.iasbaba.com/
45.00 /
Email: ilp@iasbaba.com
Page 64 150
2019 - Test 9- History &
Exam Title :
Cu...
Email : vikasutsav@hotmail.com
Contact :
QUESTION 16.
गुलाम�गरी�पु�तक�का�लेखक�कौन�है�?

a) �यो�तबा�फुले�
b) गोपाल�ह�र�दे शमुख�
c) गोपाल�गणेश�अगरकर�
d) गोपाल�कृ�ण�गोखले�
Correct Answer: A
Your Answer:
Explanation

Solution (a)

सतारा, महारा��म��ज�मे��यो�तबा�फुले�(1827-1890), माली�समुदाय�से�थे�और�उ�ह�ने�उ�च�जा�त�के��भु�व�तथा��ा�णीय�सव��चता�के


�व���एक�श��शाली�आंदोलन�को�संग�ठत��कया।�साव�ज�नक�स�यधम��और�गुलाम�गरी�नामक�फुले�क��पु�तक��आम�जनता�के��लए��ेरणा�का
��ोत�बनी।�फुले�ने��ा�ण��के��तीक�राम�को��योग�करने�क��बजाए�राजा�ब�ल�के��तीक�का��योग��कया।�

QUESTION 17.
�न�न�ल�खत�म��से�कौन�से�सही�सुमे�लत�ह��?

सं�थापक�संगठन�

1. गोपाल�कृ�ण�गोखले�A) सव��ट् स�ऑफ़�इं�डया�सोसाइट��

2. नारायण�म�हार�जोशी�B) सोशल�स�व�स�लीग�

3. �वामी��ववेकानंद�C) रामकृ�ण��मशन�

सही�कूट�चु�नए�

a) केवल�1 और�2
b) केवल�1 और�3
c) केवल�2 और�3
d) उपयु���सभी�
Correct Answer: D
Your Answer:
Explanation

Solution (d)

�वत: �प��

QUESTION 18.
�न�न�ल�खत�म��से��कसने��ह���न�ल�को�‘ब�चो��ं�के�ब�चे’ कहा�?

a) दयानंद�सर�वती�
b) �वामी��ववेकानंद�
c) ई�र�चं���व�ासागर�
d) लाला�लाजपत�राय�
Correct Answer: A
Your Answer:

IASbaba
Score:
Web: http://ilp.iasbaba.com/
45.00 /
Email: ilp@iasbaba.com
Page 65 150
2019 - Test 9- History &
Exam Title :
Cu...
Email : vikasutsav@hotmail.com
Contact :
Explanation

Solution (a)

�वामी�दयान�द�ने�एक�बार��ह���न�ल�को�‘ब�चो��ं�के�ब�चे’ कहकर�खेद�जताया।�

QUESTION 19.
�न�न�ल�खत�म��से��कसने�ईसाई�और�इ�लाम�धम��अपना�चुके�लोग��को�वापस��ह���धम��म��लाने�हेतु�शु���आंदोलन�आरंभ��कया�?

a) आय��समाज�
b) �ाथ�ना�समाज�
c) धम��सभा�
d) �व-स�मान�आंदोलन�
Correct Answer: A
Your Answer:
Explanation

Solution (a)

आय��समाज�ने�ईसाई�और�इ�लाम�धम��अपना�चुके�लोग��को�वापस��ह���धम��म��लाने�हेतु�शु���आंदोलन�आरंभ��कया�।�इससे�1920
के�दशक�के�दौरान�सामा�जक�जीवन�का�सां�दा�यक�करण�बढ़ा�और�आगे�चलकर�यह�सां�दा�यक�राजनी�तक�चेतना�के��प�म��तेजी�बढ़�गया।�

QUESTION 20.
�कसने�लोग��को�बाल��ववाह�के��व�����त�ा�लेने�हेतु��े�रत�करने�के��लए�‘��त�ा�आंदोलन’ आरंभ��कया�?

a) इं�डयन�सोशल�कां��स�
b) ���समाज�
c) �थयोसो�फकल�सोसाइट��ऑफ़�इं�डया�
d) �ाथ�ना�समाज�
Correct Answer: A
Your Answer:
Explanation

Solution (a)

एम.जी. रानाडे�तथा�रघुनाथ�राव�ने�इं�डयन�सोशल�कां��स�क���थापना�क��जो�म�ास�म��1887 म���ए�अपने�पहले�स��के�प�ात�हर�वष��उसी


समय�और��थान�पर�बैठक�करता�रहा, जहाँ�भारतीय�रा�ीय�कां�ेस�अपनी�बैठक�करती�थी।�इसने�मह�व�के�सामा�जक�मु���पर��यान�क���त
�कया�; इसे�असल�म��भारतीय�रा�ीय�कां�ेस�का�सामा�जक�सुधार��को��कहा�जा�सकता�है।�कां��स�ने�अंतरजातीय��ववाह��का�समथ�न
�कया, ब��ववाह�तथा�कुलीनवाद�का��वरोध��कया।�इसने�लोग��को�बाल��ववाह�के��व�����त�ा�लेने�हेतु��े�रत�करने�के��लए�‘��त�ा
आंदोलन’ आरंभ��कया।�

QUESTION 21.
इ�बट� ��वधेयक�के�बारे��न�न�ल�खत�म��से�कौन�सा�कथन�स�य�है�?

a) यूरोपीय�नाग�रक��को�भारतीय�म�ज��े ट��के��ारा��न�ण�त��कया�जा�सकता�था�।
b) भारतीय�भाषाआ��म���का�शत�होने�वाले�समाचार�प���व�प��का��पर���तबंध��का�आरोपण�।
c) भारतीय���ारा��च��त�श���रखे�जाने�पर���तबंध��का�आरोपण�।
d) अकाल�के�दौरान�भू-राज�व�पर�अ�त�र��कर�का�आरोपण�।
Correct Answer: A
Your Answer:
Explanation

IASbaba
Score:
Web: http://ilp.iasbaba.com/
45.00 /
Email: ilp@iasbaba.com
Page 66 150
2019 - Test 9- History &
Exam Title :
Cu...
Email : vikasutsav@hotmail.com
Contact :

Solution (a)

इ�बट� ��वधेयक�

इ�बट� ��वधेयक�1883 म��वाइसराय�लॉड���रपन�(�ज�ह�ने��रपन-द�गुड�के��प�म���या�त�पाई) के�काय�काल�म��लाया�गया��जसे�सर�सी. पी.


इ�बट� �(वाइसराय�प�रषद�के��व�ध�सद�य) �ारा��लखा�गया�था�।�इस�क�थत�अ�ध�नयम�के�अनुसार, भारतीय�जज�यूरोपीय�आरो�पय��के�मुक�मे
क��सुनवाई�कर�सकते�थे।�

इसके�चलते�भारत�म��रहने�वाले�सभी�यूरोपीय�लोग��ने�संग�ठत�होकर��वरोध��कया�।�इसके�प�ात�समझौते�के��प�म��संशोधन�लाया�गया।
संशोधन�के�अनुसार, भारतीय�जज�उन�मुक�म��क��सुनवाई�क��अ�य�ता�कर�सकते�थे��जनम��यूरोपीय�लोग�शा�मल�ह���क�तु�यूरोपीय�लोग��को
यह�अ�धकार��दया�गया��क�वे�ऐसी�जूरी��ारा�सुनवाई�क��माँग�कर�ल���जसके�कम�से�कम�आधे�सद�य�गोरे�यूरोपीय�ह�।�

इसने�इ�बट� ��वधेयक�के�मूल�उ�े �य�को�पूण�त: समा�त�कर��दया�।�हालां�क��श��त�भारतीय��ने�भी�संशोधन�का��वरोध��कया�था, �क�तु�उ�ह�ने


महसूस��कया��क�सरकार�पर�समान�बता�व�का�दबाव�डालने�के��लए�एक�अ�खल�भारतीय�संगठन�तथा�भारतीय��के�बीच�अ�धक�सम�वय�क�
आव�यकता�है।�इसी�लए�इ�बट� ��वधेयक�के��ववाद�को�आईएनसी�क���थापना�का�एक�मह�वपूण��पूव�कारण�माना�जाता�है�।

QUESTION 22.
�न�न�ल�खत�म��से�कौन�से�राजनी�तक�संघ��क��शु�आत�बंगाल�म���ई�थी�?

1. जम�दारी�एसो�सएशन�

2. ���टश�इं�डयन�एसो�सएशन�

3. इं�डयन�नेशनल�एसो�सएशन�

4. पूना�साव�ज�नक�सभा�

सही�कूट�चु�नए�

a) केवल�1,2 और�3
b) केवल�2,3 और�4
c) केवल�1 और�3
d) 2 and 3 only
Correct Answer: A
Your Answer:
Explanation

Solution (a)

पूना�साव�ज�नक�सभा�क���थापना�महादे व�गो�व�द�रानाडे�तथा�अ�य���ारा�बॉ�बे�म��1867 म��क��गई�थी�और�इसका�उ�े �य�था�लोग��और


सरकार�के�बीच�एक�सेतु�का�काय��करना।�

QUESTION 23.
�न�न�ल�खत�म��से��कसने�भारतीय�रा�ीय�कां�ेस�के��थम�स��क��अ�य�ता�क��?

a) ड��यू. सी. बनज��


b) दादाभाई�नौरोजी�
c) ए. ओ. ��म�
d) बद���न�तैयबजी�
Correct Answer: A
Your Answer:
Explanation

IASbaba
Score:
Web: http://ilp.iasbaba.com/
45.00 /
Email: ilp@iasbaba.com
Page 67 150
2019 - Test 9- History &
Exam Title :
Cu...
Email : vikasutsav@hotmail.com
Contact :

Solution (a)

भारतीय�रा�ीय�कां�ेस�के��थम�अ�धवेशन�म��72 ��त�न�धय��ने�भाग��लया�था�और�इसक��अ�य�ता��ोमेश�चं��बनज��ने�क��थी।�यह��से
कां�ेस���येक�वष���दसंबर�म��दे श�के�अलग-अलग�भाग��म��बैठक�करने�लगी।�

QUESTION 24.
�न�न�ल�खत�म��से�कौन�से�कानून�के�अंतग�त�लाए�गए�सुधार��क��आलोचना�करते��ए�रा�वा�दय��ने�‘��त�न�ध�व�नह��तो�कर�नह�’ का�आ�ान
�कया�?

a) भारतीय�प�रषद�अ�ध�नयम�1892
b) भारतीय�प�रषद�अ�ध�नयम�1909
c) भारतीय�प�रषद�अ�ध�नयम�1919
d) भारत�सरकार�अ�ध�नयम�1935
Correct Answer: A
Your Answer:
Explanation

Solution (a)

आरं�भक�रा�वा�दय��ने�लोकतां��क��व-शासन�के�द�घ�कालीन�उ�े �य�के�साथ�काय���कया।�संवैधा�नक�सुधार��क��उनक��मांगे�1892 म�
भारतीय�प�रषद�अ�ध�नयम�के��प�म���वीकार�कर�ली�गई।�

इन�सुधार��क��कां�ेस�के�स���म��कड़ी�आलोचना�क��गई, जहाँ�रा�वा�दय��ने�इनके�साथ�अपने�असंतोष�को��ब�कुल�नह���छपाया।�

अब, उनक��मांगे�थे�(i) �नवा��चत�भारतीय��का�ब�मत�, तथा�(ii) बजट�के�ऊपर��नयं�ण�, अथा�त�, बजट�पर�मतदान�करने�तथा�उसे�संशो�धत


करने�क��श��।�उ�ह�ने�नारा��दया�— ‘��त�न�ध�व�नह��तो�कर�नह�’ ।�धीरे-धीरे�करके�संवैधा�नक�सुधार��का�दायरा�बढ़ता�चला�गया।�

QUESTION 25.
�न�न�ल�खत�म��से��कसने�कां�ेस�को�‘राज�ोह�क��फै���’ कहा�?

a) लॉड��डफ�रन�
b) लॉड��कज�न�
c) लॉड��ल�सडाउन�
d) लॉड��चे�सफोड��
Correct Answer: A
Your Answer:
Explanation

Solution (a)

जब�कां�ेस��नरंतर�औप�नवे�शक�शासन�क��आलोचक�बन�रही�थी, सरकार�भी�मु���प�से�कां�ेस�क���न�दा�करने�म��लग�गई�और�रा�वा�दय�
को�“ राज�ोही��ा�ण�” , “ �न�ाहीन�बाबू�” आ�द�कहकर�पुकारने�लगी�। डफ�रन�ने�कां�ेस�को�‘राज�ोह�क��फै���’ कहा�।

QUESTION 26.
�न�न�ल�खत�म��से��कस��वभू�त�ने�राममोहन�राय�को�‘राजा’ क��उपा�ध��दान�क��?

a) अकबर�II
b) फ�� ��सयर�
c) �सराज�उद�दौला�
d) लॉड��ब��टक�

IASbaba
Score:
Web: http://ilp.iasbaba.com/
45.00 /
Email: ilp@iasbaba.com
Page 68 150
2019 - Test 9- History &
Exam Title :
Cu...
Email : vikasutsav@hotmail.com
Contact :
Correct Answer: A
Your Answer:
Explanation

Solution (a)

अकबर�शाह�II

1828 तक�, राजा�राममोहन�राय�भारत�म��एक�जानी-मानी�श��सयत�बन�चुके�थे�। 1830 म��, वह�मुगल�स�ाट�अकबर�शाह�II के��त�के


�प�म��इं�ल�ड�गए�थे, �ज�ह�ने�उनको�राजा��व�लयम�IV के�दरबार�म��जाने�के��लए�‘राजा’ क��उपा�ध��दान�क��थी�।

QUESTION 27.
भारत�प�रषद�के�बारे�म���न�न�ल�खत�कथन��पर��वचार�क��जए।�

1. इसका�गठन�भारत�सरकार�अ�ध�नयम�1858 के�अंतग�त��आ�था�।
2. इसक��अ�य�ता�भारत�के�वाइसराय��ारा�क��जाती�थी�।
3. भारत�के�रा�य�स�चव�इस�प�रषद�के�अ�य��थे�।
4. इसम��कोट� �ऑफ़�डायरे�टस��के�नामां�कत�सद�य�थे�।

उपरो��म��से�कौन�सा/से�कथन�स�य�है/ह��?

a) केवल�1 और�2
b) केवल�2 और�3
c) केवल�1 और�3
d) केवल�2 और�4
Correct Answer: C
Your Answer:
Explanation

Solution (c)

भारत�प�रषद�क���थापना�भारत�सरकार�अ�ध�नयम�1858 के�अंतग�त�क��गई�थी�। इसक��अ�य�ता�भारत�के�रा�य�स�चव��ारा�क��जाती�थी�


। चूँ�क�कोट� �ऑफ़�डायरे�टस��को�इस�अ�ध�नयम�के�ज�रए�भंग�कर��दया�गया�था, अत: इसम��कोट� �ऑफ़�डायरे�टस��का�कोई�नामां�कत�सद�य�
नह��था�।

QUESTION 28.
1880 के�अकाल�आयोग�के�संदभ��म���न�न�ल�खत�कथन��पर��वचार�क��जए।�

1. इसक���नयु���लॉड���रपन��ारा�क��गई�थी�।
2. �रचड����े ची�इसके�अ�य��थे�।
3. आयोग�क���रपोट� �के�आधार�पर�पहली�अकाल�सं�हता�का��नमा�ण��कया�गया�था�।

उपरो��म��से�कौन�से�कथन�स�य�ह��?

a) केवल�1 और�2
b) केवल�2 और�3
c) केवल�1 और�3
d) उपयु���सभी�
Correct Answer: B
Your Answer:
Explanation

Solution (b)

IASbaba
Score:
Web: http://ilp.iasbaba.com/
45.00 /
Email: ilp@iasbaba.com
Page 69 150
2019 - Test 9- History &
Exam Title :
Cu...
Email : vikasutsav@hotmail.com
Contact :

1880 म��भारत�के�गवन�र-जनरल�लॉड���लटन�ने��रचड����े ची�क��अ�य�ता�म��एक�अकाल�आयोग�क���नयु���क��।�इसने�स�म�शरीर�वाल��को


रोजगार, तथा�शारी�रक��प�से�अ�म�लोग��को�राहत��दान�करने�क��अनुशंसा�क��और�सुझाया��क�आव�यक�क���य�सहायता�के�साथ�अकाल
राहत��ांतीय�सरकार��का��मुख�उ�रदा�य�व�होना�चा�हए।�

स�म�त�क��अनुशंसा�के�आधार�पर, 1883 म��पहली�अकाल�सं�हता�का��नमा�ण��कया�गया�था�।

�सरा�अकाल�आयोग�क���नयु���लॉड��कज�न��ारा�1901 म��1899-1900 के�अकाल�के�प�ात�क��गई�थी�।

QUESTION 29.
�न�न�ल�खत�म��से�कौन�सा�यु�म�सही�सुमे�लत�है�?

1. �व�ीय��वक���करण�पर�मेयो�का���ताव�: 1870
2. �वक���करण�पर�शाही�आयोग�: 1905
3. �व�ीय��वक���करण�पर��रपन�का���ताव�: 1882

�न�न�ल�खत�म��से�कूट�चु�नए�

a) केवल�1 और�2
b) केवल�2 और�3
c) केवल�1 और�3
d) उपरो��म��से�कोई�नह��
Correct Answer: C
Your Answer:
Explanation

Solution (c)

लॉड���रपन�को�भारतीय��को��थानीय��व-सरकार�अ�ध�नयम�1882 के�मा�यम�से��वतं�ता�का�पहला�रस�वादन�करवाने�वाले�����के�तौर�पर
जाना�जाता�है�। �थानीय��व-सरकार�क��उसक��योजना�ने�नगरपा�लका�सं�थान��को��वक�सत��कया�जो����टश��ाउन��ारा�भारत�पर��कूमत
जमाने�के�बाद�से�दे श�म��वृ��मान�ह��।�उसने�अ�ध�नयम��क��उस��ृंखला�का���त�न�ध�व��कया��जनके�अंतग�त��ामीण�और�शहरी��नकाय��को�स्
थानीय��व-सरका�र���दान�क��गई�तथा��नवा��चत���त�न�धय��को�कुछ��ापक�अ�धकार��मले�। लॉड���रपन�को�भारत�म���थानीय��व-सरकार
का��पता�कहा�जाता�है�।

1870 का�मेयो�का���ताव�

1861 के�भारतीय�प�रषद�अ�ध�नयम�ने��वधायी�ह�तांतरण�क��नी�त�क��शु�आत�क��1870 म���व�ीय��वक���करण�पर�आया�मेयो�का���ताव


इसका�एक�संभा�वत�प�रणाम�था�।��शास�नक�उपयु�ता�और��व�ीय�कठोरता�ने�सा�ा�य�क��सरकार�को��ांतीय�सरकार��के��बंधन�को
�शासन�के�कुछ��व�श���वभाग�पुन: �वत�रत�करने�हेतु��े�रत��कया��जनम��अ�य��के�साथ-साथ��श�ा, �च�क�सक�य�सेवाएं�तथा�सड़क��शा�मल
थी।�

�वक���करण�पर�शाही�आयोग�क���नयु���1907 म��सर�हेनरी��व�लयम���मोस��क��अ�य�ता�म��क��गई�थी�।�यह�एक�छ: सद�यीय


�नकाय�था��जसम��अ�य�, पाँच�अ�य�सद�य�शा�मल�थे, �जनम���ेडे�रक�लेली, �ट��न�ग�एडगल�, रमेश�चं��द�, �व�लयम�मेयर�तथा��व�लयम
�हच�स�स��म�लत�थे।�

�वक���करण�पर�शाही�आयोग�क���नयु���12 �सतंबर�1907 को�भारत�सरकार�तथा�भारत�क���व�भ���ांतीय�सरकार��के�बीच, तथा��ांतीय


सरकार��व�उनके�अधीन�थ��ा�धकार��के�बीच��व�मान�संबंध��क��पड़ताल�करने�एवं�यह��रपोट� �दे ने��क��या��वक���करण�अथवा��कसी�अ�य
उपाय�से�उन�संबंध��को�सुधारा�अथवा�सरलीकृत��कया�जा�सकता�है�तथा��या�सरकार�क���णाली��व�भ���ांत��क��आव�यकता��क��पू�त�
करने�और�क�याण�को�बढ़ावा�दे ने�तथा��बना�इनक��श���और�एकता�को�बढाए�काय�कारी�श��य��को��थानीय�दशा��के��नकट�लाने�म��स�म
है, के��लए�क��गई�थी�।�आयोग�ने�फरवरी�1909 म��अपनी��रपोट� �स�पी�।

QUESTION 30.

IASbaba
Score:
Web: http://ilp.iasbaba.com/
45.00 /
Email: ilp@iasbaba.com
Page 70 150
2019 - Test 9- History &
Exam Title :
Cu...
Email : vikasutsav@hotmail.com
Contact :

1875 के�द�कन�दं ग��के�बारे�म���न�न�ल�खत�कथन��पर��वचार�क��जए।�

1. दं गे�म�ास��ांत�के�जनजातीय��े���म���ए�।
2. यह�मारवाड़ी�और�गुजराती�सा�कार��के��व���का�तकार��का��व�ोह�था�।
3. दं गाइय��का��वशेष�उ�े �य�सा�कार��के�क�जे�म��रखे�बांड्स, �ड��य��तथा�अ�य��प���को��ा�त�करना�और�न��करना�था�।

उपरो��म��से�कौन�सा/से�कथन�सही�है/ह�-

a) केवल�1 और�2
b) केवल�2 और�3
c) केवल�1 और�3
d) उपयु���सभी�
Correct Answer: B
Your Answer:
Explanation

Solution (b)

मई�और�जून�1875 म��, महारा��के�पुणे, सतारा�और�अहमदनगर��जल��के�कुछ�भाग��के��कसान��ने�बढ़ते�कृ�ष�दबाव�के�चलते��व�ोह�कर


�दया�। 1875 के�द�कन�दं ग��ने�सा�कार���ारा��न�म�त�ऋण�दासता�क��दशा��को��नशाना�बनाया�। दं गाइय��का��वशेष�उ�े �य�सा�कार��के
क�जे�म��रखे�बांड्स, �ड��य��तथा�अ�य��प���को��ा�त�करना�और�न��करना�था�।

1875 के�द�कन�दं गे��मुखतया�मारवाड़ी�और�गुजराती�सा�कार��के��व����नद� �शत�थे�।

अ�धक�भू-राज�य�क��सरकार�क��मांग��तथा�अमे�रक��गृह�यु��के�चलते�कपास�क��क�मत��म��आई��गरावट�के�योग�ने��कसान��को�गहरे�ऋण
क��ओर�धकेल��दया�था�।�सा�कार��ने��थानीय�का�तकार��क��जमीन��ह�थया�ली�थी।�दं गाई�का�तकार��ने�उन�बांड्स�को��नशाना�बनाया�जो
उ�ह�ने�सा�कार��को�ह�ता�र�करके��दए�थे, पूरे�पुणे��जले�को�जून�1875 म��आग�लगा�द��गई�।���थ�त�को��नयं��त�करने�के��लए�सरकार�को
सेना�बुलानी�पड़ी।�

QUESTION 31.
�न�न�ल�खत�म��से�कौन�सी�घटनाएं�भारत�के�गवन�र-जनरल�के��प�म��कज�न�के�काय�काल�के�दौरान�नह���ई�?

a) �व��व�ालय�आयोग�क���नयु���
b) पु�लस�आयोग�क���नयु���
c) �ाचीन��मारक�अ�ध�नयम�
d) कां�ेस�का�सूरत��वभाजन�
Correct Answer: D
Your Answer:
Explanation

Solution (d)

पु�लस�आयोग�{ सर�एंड���ेज़र�}

लॉड��कज�न�ने�सर�एंड���ेज़र�क��अ�य�ता�म��1902-03 म��एक�पु�लस�आयोग�क���थापना�क��।�इस�आयोग�क��अनुशंसा��के�आधार�पर,
भारतीय�घरेलु�आसूचना�के�एक�आधारभूत�ढ़ांचे�का��वकास��आ�और�1947 तक�यह�लगभग�ऐसा�ही�बना�रहा�।�उसने�आपरा�धक�आसूचना
�वभाग�(DCI) को�भारत�सरकार�के�साथ�जोड़��दया�जब�क����टश�भारत�के��ांत��म��आपरा�धक�जांच��वभाग��(CIDs) क���थापना�क��। 1
903 म��, ठगी�और�डकैती��वभाग�को�समा�त�कर��दया�गया�।

रालेघ�आयोग�

भारतीय��व��व�ालय�और�महा�व�ालय�धीरे-धीरे�करके�सरकार�के��खलाफ����चार�क��शरण�थली�बन�रहे�थे�। �व��व�ालय��को��नयं�ण�म�
लाने�के��लए�, लॉड��कज�न�ने�सर�थॉमस�रालेघ�क��अ�य�ता�म��रालेघ�आयोग�क���नयु��क��। रालेघ�आयोग�म��केवल�एक�भारतीय�सद�य�था,

IASbaba
Score:
Web: http://ilp.iasbaba.com/
45.00 /
Email: ilp@iasbaba.com
Page 71 150
2019 - Test 9- History &
Exam Title :
Cu...
Email : vikasutsav@hotmail.com
Contact :
सैयद��सैन��बल�ामी�लॉड��जब��ह����ने�इसका��वरोध��कया�तब�ज��टस�गु��दास�बनज��को�कलक�ा�उ�च��यायालय�से�बुलाकर�इसका
सद�य�बनाया�गया�। इस�आयोग�ने�1902 म��अपनी��रपोट� �स�पी�और�इससे�अनुसरण�म��रालेघ��वधेयक�लाया�गया�।�अ�ध�नयम�बन�जाने�पर�रा
लेघ��वधेयक�को�भारतीय��व��व�ालय�अ�ध�नयम�1904 कहा�गया�।�इस�अ�ध�नयम�ने��स�डीकेटो�के�गठन�को�पुनस�ग�ठत��कया�; महा�व�ा
लय��के�आ�धका�रक��नरी�ण�का��ावधान��कया�तथा�महा�व�ालय��क��संब�ता�व�असंब�ता�को�लेकर�अं�तम��नण�य�लेने�का�अ�धकार�भारत
सरकार�के�हाथ��म���दया�।�गोपाल�कृ�ण�गोखले�जैसे��व�ान���ारा�इस�अ�ध�नयम�क��कड़ी�आलोचना�क��गई।�उस�अ�ध�नयम�का��थम
�ावधान�था��क��व��व�ालय��क��गव�न�ग�बॉडी�को�पुन: संग�ठत��कया�जाए�और�सीनेट��के�आकार�को�घटाया�जाए।�अब�सीनेट��क���यूनतम
सं�या�50 और�अ�धकतम�सं�या�100 हो�सकती�थी�।�उनम��से���येक�का�काय�काल�6 वष��का�होगा�। बॉ�बे, कलक�ा�और�म�ास�के
�व��व�ालय��के��लए��नवा��चत�फेलोज�क��सं�या�50 रखी�गई�और�शेष��व��व�ालय��हेतु�यह�सं�या�15 �न��त�क��गई�।�इस�अ�ध�नयम�ने
सरकार�को��व��व�ालय�हेतु�अ�धसं�यक�फेलोज�क���नयु���क��अनुम�त��दान�क�।�गवन�र-जनरल�को�अब��व��व�ालय�क���े�ीय�सीमा�
के��नधा�रण�और��व��व�ालय��तथा�महा�व�ालय��के�बीच�स�ब�ता�के��नधा�रण�क��श����मल�गई�थी�। भारतीय��व��व�ालय�अ�ध�नयम�ने�
�व��व�ालय��तथा�महा�व�ालय��को�पूरी�तरह�सरकार�के��नयं�ण�म��ला��दया�। हालां�क, बेहतर��श�ा�और�शोध�के��लए�आगामी�5 वष��हेतु�
��त�वष��5 लाख��पए�के�अनुदान�को��वीकृ�त�द��गई�।�यह�भारत�म���व��व�ालयी�अनुदान�क��शु�आत�थी�जो�आगे�चलकर�भारत�म���श�ा�के
ढाँचे�क���थायी��वशेषता�बन�गई�।

�ाचीन��मारक�संर�ण�अ�ध�नयम�1904

�ाचीन��मारक�संर�ण�अ�ध�नयम�1904 , लॉड��कज�न�के�समय�पा�रत��ए�सवा��धक�उ�लेखनीय�अ�ध�नयम��म��से�एक�था�।�इस�अ�ध�नयम�ने
संर��त��मारक��को�प�ंचाई�जाने�वाली��कसी�भी��कार�क����त�को��व�ध��ारा�द�डनीय�बना��दया।�इस�अ�ध�नयम�ने�पुरात�व��वभाग�क��भी
�थापना�क���जसे�ऐ�तहा�सक�मह�व�के��प���को�एक��करने, उ�खनन�करने�तथा�ऐ�तहा�सक�जानकारी�को��काश�म��लाने�का��ज�मा��दया
गया�।

QUESTION 32.
�न�न�ल�खत�म��से�कौन�भारत�म��औप�नवे�शकरण�का/के�आ�थ�क�आलोचक�था/थे�?

1. दादाभाई�नौरोजी�
2. एम.जी. रानाडे�
3. आर.सी. द��

�न�न�ल�खत�म��से�कूट�का�चयन�क��जए�:

a) केवल�1 और�3
b) केवल�2 और�3
c) केवल�1 और�2
d) उपयु���सभी�
Correct Answer: D
Your Answer:
Explanation

Solution (d)

दादाभाई�नौरोजी, आर.सी. द�, रानाडे, गोखले, जी�सु�म�नयम�अ�यर�उन�लोग��म��से�थे��ज�ह�ने�हो�सन�और�ले�नन�से�भी�पहले


उप�नवेशवाद�पर��व��का�पहला�आ�थ�क�आलोचना�लेख�लाकर�भारतीय�रा�वाद�को�मजबूत�तरीके�से�सा�ा�यवाद��वरोध�पर�खड़ा��कया�।

�नकास��स�ांत�क���थापना�दादाभाई�नौरोजी��ारा�क��गई�थी�।�द��के�अनुसार�बढ़���ई�गरीबी�और�कम�मज�री�दर��औप�नवे�शक�शासन�के
अ��य��प�रणाम�थे।�

QUESTION 33.
Q.33 ) महारानी��व�टो�रया�क��उ�ोषणा�(1858) के��न�न�ल�खत�म��से�कौन�से��मुख�उ�े �य�थे�?

1. भारतीय�रा�य��को��वलय�करने�के��कसी�भी�इरादे �का�प�र�याग�करना�।

IASbaba
Score:
Web: http://ilp.iasbaba.com/
45.00 /
Email: ilp@iasbaba.com
Page 72 150
2019 - Test 9- History &
Exam Title :
Cu...
Email : vikasutsav@hotmail.com
Contact :
2. भारतीय��शासन�को����टश��ाउन�के�अंतग�त�लाना�।
3. भारत�के�साथ�ई�ट�इं�डया�कंपनी�के��ापार�को��व�नय�मत�करना�।

�न�न�ल�खत�म��से�कूट�का�चयन�क��जए:

a) केवल�1 और�2
b) केवल�2 और�3
c) केवल�1 और�3
d) उपयु���सभी�
Correct Answer: A
Your Answer:
Explanation

Solution (a)

1858 क��उ�ोषणा�के�प�ात�एक�वा�ण��यक�इकाई�के��प�म��कंपनी�का�अ��त�व�समा�त�हो�गया�। अत: कथन�(3) गलत�है�।�उ�ोषणा�का


उ�े �य��थानीय�सा�ा�य��और�सूब��का�एक�समूह�तैयार�करना�था��जसे�1857 जैसा��व�ोह�दोबारा�घ�टत�होने�पर�भारतीय��े���क��सुर�ा�हेतु
�योग��कया�जा�सके�।�यह�अधीन�थ�संघ�क��नी�त�क��शु�आत�थी�जो�1935 तक�चली�।�रानी�क��उ�ोषणा�ने�कंपनी�के�सभी��े���और�उनके
�शासन�को����टश�सरकार�के�हाथ��म���थानांत�रत�कर��दया�।

QUESTION 34.
�यामजी�कृ�ण�वमा��के�बारे�म���न�न�ल�खत�कथन��पर��वचार�क��जए�:

1. उ�ह�ने�द��इं�डयन�सो�शओलो�ज�ट�नामक�एक�मा�सक�प��का��का�शत�क��।
2. उ�ह�ने�इं�ल�ड�म��इं�डयन�हाउस�क���थापना�क��।
3. उ�ह�ने��वदे श��म��पढ़�रहे�भारतीय��के��लए�छा�वृ�त��था�पत�क��।

उपयु���म��से�कौन�से�कथन�सही�ह�?

a) केवल�1 और�3
b) केवल�1 और�2
c) केवल�2 और�3
d) उपरो��म��से�कोई�नह��
Correct Answer: B
Your Answer:
Explanation

Solution (b)

�यामजी�कृ�ण�वमा��एक�भारतीय��ां�तकारी, वक�ल�और�प�कार�थे��ज�ह�ने�लंदन�म��इं�डयन�होम��ल�सोसाइट�, इं�डयन�हाउस�तथा�द�


इं�डयन�सो�शओलो�ज�ट�क���थापना�क��।

1905 म��उ�ह�ने�इं�डयन�हाउस�तथा�द��इं�डयन�सो�शओलो�ज�ट�क���थापना�क��जो�एक�समय���टे न�म��रह�रहे�भारतीय�छा���के�बीच�के


प�रवत�नकारी�रा�वा�दय��हेतु�तेजी�से�एक�संग�ठत�बैठक��ब���एवं�भारत�के�बाहर��ां�तकारी�भारतीय�रा�वाद�के�सव��मुख�क���के��प�म�
�वक�सत��आ�।�इस�संगठन�के�सबसे���स��सद�य��म��वीर�सावरकर�थे।��यामजी�कृ�ण�वमा��1907 म��अ�भयोग�से�बचने�के��लए�पे�रस�चले
गए�। 1935 म��उनक��मृ�यु�हो�गई�।

बाद�म��1905 म���यामजी�ने�इं�डयन�होम��ल�सोसाइट��के�एक���त�न�ध�के��प�म��हॉलबोन��टाउन�म��आयो�जत�यूनाइटे ड�कां�ेस�ऑफ़


डेमो�ेट् स�म��भाग��लया�। भारत�पर�उनके���ताव�को�संपूण��कां��स�म��एक�उ�सा�हत�करने�वाला�समथ�न��ा�त��आ�।�इं�ल�ड�म���यामजी�क�
ग�त�व�धय��ने����टश�सरकार�क���च�ताएं�बढ़ा�द��: उ�ह��इनर�टे �पल�से��नकाल��दया�गया�तथा�द��इं�डयन�सो�शयोलॉ�ज�ट�म�����टश-�वरोधी
लेख��लखने�के��लए�30 अ�ैल�1909 को�सद�यता�सूची�से�हटा��दया�गया�।�अ�धकांश����टश��ेस��यामजी��वरोधी�थी�और�उसने�उनके�और
उनके�समाचार�प��के��व���अपमानजनक�आरोप�लगाए।�उसने�साह�सक�ढं ग�से�अपना�बचाव��कया।�द��टाइ�स�ने�उ�ह��“कु�यात�कृ�णवमा�”
के�नाम�से�संद�भ�त��कया।�अनेक�समाचार�प���ने�उन��ग�तशील����टश�नाग�रक��क��आलोचना�क��जो��यामजी�और�उनके����कोण�का

IASbaba
Score:
Web: http://ilp.iasbaba.com/
45.00 /
Email: ilp@iasbaba.com
Page 73 150
2019 - Test 9- History &
Exam Title :
Cu...
Email : vikasutsav@hotmail.com
Contact :
समथ�न�कर�रहे�थे।����टश�ख़ु�फ़या�सेवा�उनक��ग�त�व�धय��पर��नकट�से�����बनाए��ए�थी, अत: उ�ह�ने�इं�डया�हाउस�का��भार�वीर�सावरकर
को�स�प�कर�अपना�मु�यालय�पे�रस��थानांत�रत�कर��लया।��यामजी�ने�सरकार��ारा�उ�ह���गर�तार�करने�का��यास�करने�से�पूव��गोपनीय�तरीके
से���टे न�छोड़��दया।�

QUESTION 35.
���टश�सरकार�क��आ�थ�क�आलोचना�म��" गृह��भार�" एक��मुख�भू�मका�रखते�ह��। " गृह��भार�" �न�न�ल�खत�म��से��कन�कोष��से��मलकर
बनता�था�?

1. लंदन�म��भारत�काया�लय�के�समथ�न�म���यु��कोष�।
2. भारत�म��काय�रत����टश�कम�चा�रय��के�वेतन�और�प�शन��के�भुगतान�म���यु��कोष�।
3. अं�ेज���ारा�भारत�से�बाहर�छे ड़े�जाने�वाले�यु���म���यु��कोष�।

�न�न�ल�खत�म��से�कूट�का�चयन�क��जए:

a) केवल�1 और�2
b) केवल�1 और�3
c) केवल�2 और�3
d) उपयु���सभी�
Correct Answer: A
Your Answer:
Explanation

Solution (a)

आर�सी�द���ारा��लखी�गई�इकनो�मक��ह����ऑफ़�इं�डया�से�यह��प��है��क�: " भारतीय�नजराने�को�चाहे��याय�के�पैमाने�पर�तौला�जाए�या


हमारे��हत��के�आलोक�म��दे खा�जाए, वह�मानवता, सामा�य�समझ�और�अथ�शा��ीय��व�ान�के��ा�त��स�ांत��से�अंतर��लए�होगा�। उस�समय
वा�तव�म��भारतीय�खजाने�के�नजराने�से�भारत�सरकार�के�गृह��भार��जैसे�भ�व�य�के�भुगतान��को��कया�जाना�स�ची�समझदारी�होगी�।�ये��भार
संभवत: ई�ट�इं�डया��टॉक�पर�लाभांश, गृह�ऋण�पर��याज, अ�धका�रय��के�वेतन, भारत�सरकार�के�गृह��वभाग�के�साथ�जुड़े�भवन��क�
�थापना, भारतीय�सेना�तथा��स�वल�सेवा�के�सद�य��क��गैर-हा�जरी�तथा�घर�पर�होने�के�दौरान�सेवा�नवृ�त�भुगतान, भारत�म��सेवा�दे �रहे����टश
सै�नक��से�जुड़े�सभी��ववरण��का��भार�तथा����टश�सै�नक��को�भारत�से�लाने, ले�जाने�क��लागत�के�एक��ह�से�का�भुगतान�आ�द�ह�गे�" ।

QUESTION 36.
�न�न�ल�खत�म��से�कौन�सी�घटना�भारत�के�गवन�र-जनरल�के��प�म��लॉड���लटन�के�काय�काल�से�संबं�धत�नह��है�?

a) वैधा�नक��स�वल�सेवा��का��नमा�ण�।
b) ��तीय�अफगान�यु��।
c) वना��यूलर��ेस�अ�ध�नयम�का�पा�रत�होना�।
d) �थम�फै����अ�ध�नयम�का�पा�रत�होना�।
Correct Answer: D
Your Answer:
Explanation

Solution (d)

�थम�फै����अ�ध�नयम�1881 म��लॉड���रपन�(जो�लॉड���लटन�का�उ�रा�धकारी�था) �ारा�पा�रत��कया�गया�था�। वैधा�नक��स�वल�सेवाएं�18


78-89 म��शु��क��गई�।�इसके�अंतग�त�सरकार�“अ�छे �प�रवार�और��या�त” वाले�भारतीय��को�भारत�के�रा�य�स�चव�के�अं�तम�अनुमोदन�के
अधीन�रहते��ए��ांतीय�सरकार��क��अनुशंसा�पर�SCC म���नयु��कर�सकती�थी�।

वना��यूलर��ेस�अ�ध�नयम�1878 म���ा�यवाद�/दे शज�भाषा��म��होने�वाले��काशन��पर��नयं�ण�लगाने�हेतु�लाया�गया�था�।�इसके�अंतग�त�एक


म�ज��े ट�को��कसी�भी��कार�के��काशक�को�बुलाकर�उनसे�एक�बॉ�ड�(जो��क�सरकार�के��व���वैमन�य�क��भावना��को�उकसावा�दे �सकने
वाली�कोई�भी�साम�ी�न�छापने�का�वचनप��होता�था) भरवाने�क��श����ा�त�थी�।�वह�जमानत�रा�श�क��माँग�कर�सकता�था�और�उसे�ज�त�कर
सकता�था�तथा�छपाई�के�उपकरण��को�ज�त�कर�सकता�था।�

IASbaba
Score:
Web: http://ilp.iasbaba.com/
45.00 /
Email: ilp@iasbaba.com
Page 74 150
2019 - Test 9- History &
Exam Title :
Cu...
Email : vikasutsav@hotmail.com
Contact :
QUESTION 37.
सामा�जक�सुधार�आंदोलन��क���न�न�ल�खत�म��से�कौन�सी�सीमाएं�थे�?

1. संक�ण��सामा�जक�आधार�

2. आंदोलन��ामीण�भारत�तक�नह��प�ँचा।�

3. जा�तवाद�मजबूत�बना�रहा।�

4. सामा�जक�स�ाव�बढ़ा।�

�न�न�ल�खत�म��से�कूट�का�चयन�क��जए:

a) केवल�1, 2 और�3
b) केवल�1, 3 और�4
c) केवल�1 और�4
d) उपयु���सभी�
Correct Answer: A
Your Answer:
Explanation

Solution (a)

सामा�जक�सुधार�आंदोलन��क��सीमाएं�

संक�ण��सामा�जक�आधार�

धरातल�पर�सुधार��ने�एक�अ�यंत�अ�प�सं�या�को��भा�वत��कया।�केवल��श��त�और�शहरी�म�यम�बग��ही�सामा�जक�सुधार�आ�दोलन�से�जुड़ा
था, जब�क�का�तकार��तथा�शहरी�गरीब��क��अ�धसं�या�क��ज�रत��पर��यान�नह���दया�गया।�

आंदोलन��ामीण�भारत�तक�नह��प�ँचा�

�ामीण��े���म���ापक��नर�रता�के�होने�तथा�आधु�नक�तथा��व�वध�संचार�नेटवक��के�अभाव�के�कारण��ोता��क��सं�या�का�कम�होना
�न��त�था।�इस��कार, अपनी��ायो�गक�अपील�म��भी�आंदोलन�अपनी�अ�य�सीमा��के�साथ-साथ�शहरी�बना�रहा।�

जा�तवाद�मजबूत�बना�रहा�

जातीय�अंतर�मजबूत�बने�रहे�और�धा�म�क�व�सामा�जक��थाएं�समा�त�नह��हई।�जा�त�और��रवाज��को�भारतीय�अवचेतना�से�हटाना�क�ठन
�स���आ।�सुधारक��क��भूतकाल�क��महानता�क��अपील�क���वृ�त�तथा�धम��ंथीय�स�ा�पर��नभ�र�बने�रहने�ने�धम��का�वग�करण�कर��दया
और�साथ�ही�उ�च�जा�त�के��ह����को��न�नजातीय��ह����से�अलग�कर��दया।�

सां�दा�यक�चेतना�

सं�कृ�त�के�धा�म�क�और�दाश��नक�पहलु��पर�अ�धक�जोर�दे ने�तथा�पंथ�नरपे�ता�वाले�पहलु��को�उपे��त�करने�के�चलते��ह����ाचीन
भारतीय�इ�तहास�क���शंसा�करने�लगे�और�मु��लम�म�यकालीन�इ�तहास�तक��समट�गए।�इसने�लोग��के�दो�अलग�भाग�होने�का�भाव�उ�प�
�कया�तथा�सां�दा�यक�चेतना�को�बढ़ाया।�

�या�आप�जानते�ह��?

· इं�डयन�नेशनल�सोशल�कां��स��जसक���थापना�1887 म��एम.जी. रानाडे�तथा�रघुनाथ�राव��ारा�क��गई�थी, क��बैठक��कां�ेस�के�वा�ष�क


�को���के�इतर��आ�करती�थी�और�यह�सामा�जक�मु���पर�चचा��करती�थी।�इसे�कां�ेस�का�सामा�जक��को��भी�कहा�जाता�है।�हालां�क, 18
95 म���तलक�और�अ�य�गरमपं�थय��के��वरोध�के�कारण�सोशल�कां��स�को��वयं�को�पूण�त: कां�ेस�से�अलग�करना�पड़ा�।

सो�चए�

IASbaba
Score:
Web: http://ilp.iasbaba.com/
45.00 /
Email: ilp@iasbaba.com
Page 75 150
2019 - Test 9- History &
Exam Title :
Cu...
Email : vikasutsav@hotmail.com
Contact :

· सामा�जक�सुधार�आंदोलन�का�च�र�।�

QUESTION 38.
छा���के�यंग�बंगाल�समूह�के�बारे�म���न�न�ल�खत�कथन��पर��वचार�क��जए�:

1. उ�ह�ने��ह���धम��क��धा�म�क�और�सामा�जक���ढ़वा�दता�को�एक�बौ��क�चुनौती�द��।

2. ���टश�और�प��मी�सीख�म��उनक��पूण��आ�था�थी��जसने�उ�ह��जनसमुदाय�से�अलग�कर��दया�।

3. वे�हेनरी��व�वयन�डेरो�जयो�से��भा�वत�थे�और�सोसाइट��फॉर��ांसले�ट�ग�यूरो�पयन�साइंसेज�क���थापना�हेतु�उ�रदायी�थे�।

उपयु����दए�गए�कथन��म��से�कौन�सा�सही�है?

a) केवल�1 और�2
b) केवल�1 और�3
c) केवल�2 और�3
d) उपयु���सभी�
Correct Answer: A
Your Answer:
Explanation

Solution (a)

छा���का�एक��ववा�दत�समूह�यंग�बंगाल, जो��क�कलक�ा�के��ह���कॉलेज�के�एक�यूरे�शयाई��श�क�( हेनरी��व�वयन�डेरो�जयो�) से��भा�वत


था, सोसाइट��ऑफ़�ए�वीजीशन�ऑफ़�जनरल�नॉलेज�क���थापना�हेतु�उ�रदायी�था�।

यह�समूह�अपने�सामा�जक��व�ोह�के�कारण�कु�यात�हो�गया�।�उनका��व�ोह�धा�म�क�दायरे�तक��व�तृत��आ�और��ह��धम��क��धा�म�क�और
सामा�जक���ढवा�दता�को�एक�बौ��क�चुनौती�द�।�

���टश�और�प��मी�सीख�म��उनक��पूण��आ�था�थी��जसने�उ�ह��जनसमुदाय�से�अलग�कर��दया�।

�या�आप�जानते�ह��?

डेरो�जयो�उन�कुछ��ल�भ��श�क��म��से�था��जनका��ान�के�साथ�जुड़ाव, स�य�को��ेम, तथा�बुराई�से�घृणा�ने�उनके�संपक��म��आने�वाले�लोग��पर


गहरा��भाव�छोड़ा।�सुकरात�क��तरह, उसने�उसका�अनुसरण��कया�जो�सही�था�और�उस�पर�युवा��को��द���मत�करने�का�आरोप�लगाया
गया।�कुछ��भावशाली��ह����ने�उसे�सेवा�से�हटवा��दया�और�उसके�कुछ�समय�प�ात, उनक��मृ�यु�हो�गई।��क�तु�डेरो�जयो�का��भाव�जारी
रहा�और�इसे�यंग�बंगाल�आंदोलन�के�नाम�से�जाना�जाता�था।�उस�समय�के�सभी��मुख�आंदोलन�इससे�जुड़े�थे।�

सो�चए�

· यंग�बंगाल�आंदोलन�

· हेनरी�लुइस��व�वयन�डेरो�जयो�का�योगदान�और�उनके�काय��

QUESTION 39.
आरं�भक�उ�ीसव��शता�द��के�सामा�जक-धा�म�क�सुधारक��क���न�न�ल�खत�म��से��या��वशेषताएं�थी�?

1. इन�सुधारक��ने�प��मी��श�ा��ा�त�क��और�वे�प��म-�ायो�जत�थे�।

2. इन�सुधारक���ारा�आगे�बढाए�गए�सुधार��का�उ�े �य�पूरे�समाज�को�ढ़ांचागत��प�से�पुनग��ठत�करना�नह��था�।

3. इन�सुधारक���ारा�उठाए�गए�मु�े�उ�च�वग��पर�लागू�होते�थे�और�उ�ह��के��लए�आम�थे�तथा��नचले�तबक��से�उनका�कम�लेना-दे ना�था�।

IASbaba
Score:
Web: http://ilp.iasbaba.com/
45.00 /
Email: ilp@iasbaba.com
Page 76 150
2019 - Test 9- History &
Exam Title :
Cu...
Email : vikasutsav@hotmail.com
Contact :

सही�उ�र�चु�नए-

a) केवल�1 और�2
b) केवल�1 और�3
c) केवल�2 और�3
d) उपयु���सभी�
Correct Answer: C
Your Answer:
Explanation

Q. 39 ) Solution (c)

आरं�भक�उ�ीसव��शता�द��ने�भारतीय�समाज�का��पांतरण�दे खा�और�ऐसी�पहल��उन�लोग��के��ारा�क��गई��ज�ह�ने�समाज�के�पूव��नधा��रत
�नयम��के��व����व�ोह��कया�।

· इन�सुधारक��ने�हालां�क�प��मी��श�ा��ा�त�क���क�तु�वे�प��म-�ायो�जत�नह��थे�। �क�तु�उ�ह�ने��वयं�सुधार��क���थापना�क��और�उ�ह�
बढ़ावा��दया�।

· यह�भी�मना�नह���कया�जा�सकता��क�सुधार�आंदोलन�का�एक�मजबूत�बौ��क�आधार�था��जसने�उ�ह��जनसमूह�से��र�रखा�और�यह�समाज
के��श��त�तबक��तक�ही�सी�मत�था�।

· इन�सुधार��का�उ�े �य�समाज�के�वं�चत�और�पी�ड़त�तबक��के�लाभ�के��लए�पूरे�समाज�को�ढ़ांचागत��प�से�पुनग��ठत�करना�नह��था�। इन
सुधार��का�उ�े �य��व�मान�सामा�जक�ढ़ांचे�म��एक�नया�जीवन�भरना�था�।

· अ�धकांश�इ�तहासकार��ने�यह�दे खा�है��क�इन�सुधारक���ारा�उ�ीसव��शता�द��के�आरं�भक�भाग�म��उठाए�गए�मु�े�उ�च�वग��पर�लागू�होते�थे
और�उ�ह��के��लए�आम�थे�तथा��नचले�तबक��के��लए�उनका�कोई�अथ��नह��था�।

· उ�ीसव��शता�द��के�बाद�के�भाग�के�आंदोलन��के�साथ-साथ�बीसव��शता�द��के�आंदोलन�वे�थे�जो�समाज�के��ापक��ह�से�तक�प�ँचे��जसम�
समाज�के��नचले�और�द�मत�तबके�भी�शा�मल�थे�। ( नोट�: ���आरं�भक�19 व��शता�द��के�बारे�म��है�) ।

QUESTION 40.
‘आ�म-स�मान�आंदोलन’ के�बारे�म���न�न�ल�खत�कथन��पर��वचार�क��जए।�:

1. इसक���थापना�केरल�म���ा�णवाद�के��व���ई.वी. रामा�वामी��ारा�क��गई�थी।�

2. इसका�उ�े �य�ऐसे�समाज�का��नमा�ण�था�जहाँ��पछड़े�तबक��के�पास�समान�अ�धकार�ह�।�

3. इस�आंदोलन�ने�अंतरजातीय��ववाह��को��ो�सा�हत��कया�जो��कसी��ा�ण�पुजारी��ारा�नह��संप��कराए�गए।�

उपयु���म��से�कौन�से�कथन�सही�ह��?

a) केवल�2 और�3
b) केवल�1 और�2
c) केवल�1
d) उपरो��म��से�कोई�नह��
Correct Answer: A
Your Answer:
Explanation

Solution (a)

IASbaba
Score:
Web: http://ilp.iasbaba.com/
45.00 /
Email: ilp@iasbaba.com
Page 77 150
2019 - Test 9- History &
Exam Title :
Cu...
Email : vikasutsav@hotmail.com
Contact :

�व-स�मान�आंदोलन�एक�ऐसा�आंदोलन�है��जसका�उ�े �य�एक�ऐसे�समाज�को��ा�त�करना�है�जहाँ��पछड़ी�जा�तय��के�पास�समान�मानवा�धकार
ह�, तथा�इसने�एक�ऐसे�जा�त-आधा�रत�समाज�म���पछड़ी�जा�तय��को��ो�सा�हत��कया��जसने�इ�ह��सोपान�के�सबसे�नीचे�के�पायदान�पर�समझा
था�।

यह�ई.वी. रामा�वामी�( पे�रयार�के�नाम�से��व�यात�) �ारा�त�मलनाडु �( केरल�म��नह��) म��शु��क��गई�थी।�

इसका��मुख�उ�े �य�त�मल��क���न�न�जा�तय��क��सामा�जक-आ�थ�क�दशाएं�सुधारना�था।�आगे�चलकर�इसके�गहरे��भाव�पड़े।�इस�आंदोलन
का��मुख�उ�े �य�राजनी�तक��श�ा�के��ान�का��सार�करना�था।�गौरव�और�आ�म-स�मान�के�साथ�जीवन�जीने�का�अ�धकार�तथा�अ�ध�व�ास�
व�मा�यता��पर�आधा�रत�शोषक��णाली�को�समा�त�करना�इसका�उ�े �य�था।�सामा�जक�कु�था��का�अंत�तथा�म�हला�अ�धकार��क��र�ा
करना।�अनाथ��और��वधवाओ�के��लए�गृह��का��नमा�ण�और�उनका�रखरखाव�तथा��श�ण�सं�थान��को�उनके��लए�खोलना।�इस�आंदोलन�ने
शी��ही�लोक��यता��ा�त�क��और�यह�एक�राजनी�तक�मंच�बन�गया।�

सो�चए�

· �व-स�मान�आंदोलन�के�समाज�पर��या��भाव�ह��?

QUESTION 41.
भारत�के�वाइसराय�के�बारे�म���न�न�ल�खत�कथन��पर��वचार�क��जए�:

1. 1858 के�अ�ध�नयम�के�प�ात�गवन�र�जनरल�का�पद�समा�त�कर�इसे�वाइसराय�के�पद�से���त�था�पत�कर��दया�गया।�

2. लॉड��कै�न�ग�भारत�के��थम�वाइसराय�थे।�

उपयु���म��से�कौन�सा�कथन�गलत�है?

a) केवल�1
b) केवल�2
c) 1 और�2 दोन��
d) न�तो�1 न�ही�2
Correct Answer: A
Your Answer:
Explanation

Solution (a)

नोट�: गलत��वक�प�छांटने�ह�।�

वाइसराय�का�पद�1858 के�भारत�सरकार�अ�ध�नयम��ारा�सृ�जत��कया�गया�था, �क�तु�गवन�र-जनरल�का�पद�समा�त�नह���कया�गया�।

लॉड��कै�न�ग�ने�1856 से�1862 तक�गवन�र-जनरल�का�पद�पदभार�संभाला�।�उसके�काय�काल�के�दौरान, 1858 का�भारत�सरकार


अ�ध�नयम�पा�रत��कया�गया��जसने�वाइसराय�के�पद�का�सृजन��कया��जस�पर�वही������वराजमान�होगा��जसके�पास�गवन�र-जनरल�का�पद
है।�इस��कार, लॉड��कै�न�ग�भारत�के��थम�वाइसराय�बने�।

सो�चए�

· भारत�का��थम�गवन�र-जनरल�

QUESTION 42.
1905 के�‘�वदे शी�आंदोलन’ के�बारे�म���न�न�ल�खत�कथन��पर��वचार�क��जए�:

1. यह�पहला�गांधीवाद��आंदोलन�था।�

2. यह�बंगाल�के��वभाजन�के�चलते��ारंभ��आ।�

IASbaba
Score:
Web: http://ilp.iasbaba.com/
45.00 /
Email: ilp@iasbaba.com
Page 78 150
2019 - Test 9- History &
Exam Title :
Cu...
Email : vikasutsav@hotmail.com
Contact :

3. �वदे शी��कान��के�बाहर�धरना�दे ने�म���कसान��ने�बड़ी�सं�या�म��भागीदारी�क�।�

उपयु���म��से�कौन�से�कथन�अस�य�ह��?

a) केवल�1
b) 1 और�3
c) केवल�2
d) 2 और�3
Correct Answer: B
Your Answer:
Explanation

Solution (b)

नोट�: गलत��वक�प�छांटने�ह�।�

�वदे शी�आंदोलन�

�वदे शी�आंदोलन�भारत�का�पहला�जन�आंदोलन�था।�यह�सरकार��ारा�बंगाल�का��वभाजन�का��नण�य�लेने�के�कारण�शु���आ।�सरकार�के
अनुसार, यह�कदम��शास�नक�स��लयत�के��लए�उठाया�गया�था��य��क�बंगाल�एक�बड़ी��ेसीड�सी�थी।��क�तु�इसका�असल�कारण�फूट�डालो
राज�करो�का�अनुसरण�करना�था।�

भारत�का�अ�धकाँश�बु��जीवी�वग��बंगाल�से�था�और�वे�अं�ेज��क��नी�तय��क��आलोचना��कया�करते�थे।�यह��वभाजन�इस��कार��कया�गया
था��क�बां�ला�बोलने�वाले�मु��लम��क��ब�सं�या�पूव��बंगाल�म��चली�गई�और�प��म�बंगाल�म���ह�द��और�ओ�डया�बोलने�वाले�लोग�अ�पसं�यक
बन�कर�रह�गए।�

यहाँ��यान�रखने�यो�य�बात�यह�है��क�यह�एक�शहरी�आंदोलन�था।�अत: इसम���कसान��क��कोई�भागीदारी�नह��थी�।

नोट�: अब�तक�महा�मा�गाँधी�भारतीय��वतं�ता�संघष��के�पद� �पर�नमूदार�नह���ए�थे।�

सो�चए�

· उदारवा�दय��क��उपल��धयां�

· सूरत��वभाजन�

QUESTION 43.
भारत�क��पहली�जनगणना��ई�थी�:

a) 1872 म��
b) 1881 म��
c) 1891 म��
d) 1901 म��
Correct Answer: A
Your Answer:
Explanation

Solution (a)

एक��व��थत�और�आधु�नक�जनगणना�अपने�वत�मान��व�प�म��1865 से�1872 के�बीच�दे श�के��व�भ��भाग��म��अ��मक�ढं ग�से�क��गई


थी।�1872 म��चरमो�कष��पर�प�ँचा�यह��यास�लोक��य��प�से�भारत�क��पहली�जनगणना�कहलाता�है।�हालां�क, भारत�क��पहली���मक
जनगणना�1881 म���ई�थी।�तबसे, जनगणना��बना��कावट�के���येक�दस�वष��पर�होती�आ�रही�है।�

IASbaba
Score:
Web: http://ilp.iasbaba.com/
45.00 /
Email: ilp@iasbaba.com
Page 79 150
2019 - Test 9- History &
Exam Title :
Cu...
Email : vikasutsav@hotmail.com
Contact :
QUESTION 44.
�न�न�ल�खत�म��से�कौन�दादाभाई�नौरोजी�के�साथ�संब��ह��?

1. �ान��सारक�म�डली�
2. पावट��एंड�अन-���टश��ल�इन�इं�डया�
3. बॉ�बे�गजट�
4. बॉ�बे�एसो�सएशन�

�न�न�ल�खत�म��से�कूट�का�चयन�क��जए:

a) 1 और�2
b) 1, 2 और�4
c) 1, 2 और�3
d) 2 और�3
Correct Answer: B
Your Answer:
Explanation

Q. 44 ) Solution ( b )

दादाभाई�नौरोजी�का�योगदान/काय��

‘ कां�ेस�’ का�अथ��है�लोग��क��सभा।�कां�ेस�का�यह�नाम�संगठन�को�दादाभाई�नौरोजी��ारा�सुझाया�गया�था�।

दादाभाई�नौरोजी�ने�1866 म��ई�ट�इं�डयन�एसो�सएशन�क���थापना�क�।�

दादाभाई�नौरोजी�‘ पावट��एंड�अन-���टश��ल�इन�इं�डया�’’ नामक�पु�तक�के�लेखक�ह���जसम����स��“�नकास��स�ांत” �दया�है।�

दादाभाई�नौरोजी�को�“भारत�के�वयोवृ��पु�ष” के�नाम�से�भी�जाना�जाता�है।�वह�पहले�भारतीय�थे�जो��लबरल�पाट��क���टकट�पर�हाउस�ऑफ़
कॉम�स�के�सद�य�बने।�वह�तीन�बार�1886, 1893 तथा�1906 म��आईएनसी�के�अ�य��बने।�उ�ह�ने�1852 म��‘�ान��काश�मंडली’ तथा
बॉ�बे�एसो�सएशन�क���थापना�क�।�उ�ह��‘भारतीय�अथ�शा���और�राजनी�त�का��पता’ कहा�जाता�है।�

QUESTION 45.
�न�न�ल�खत�म��से�कौन�आईएनसी�का�अ�य��बनने�वाला��थम��वदे शी�था�?

a) ए.ओ. ��म�
b) एनी�बेस�ट�
c) जॉज��यूल�
d) हेनरी�कॉटन�
Correct Answer: C
Your Answer:
Explanation

Solution (c)

जॉज��यूल�इं�ल�ड�और�भारत�म��एक��कॉ�टश��ापारी�था�जो�1888 म��इलाहाबाद�म���ए�भारतीय�रा�ीय�कां�ेस�के�चौथे�अ�धवेशन�के�अ�य�
बने।�यह�पद�धारण�करने�वाले�वह�पहले�गैर-भारतीय�थे।�

इसके�पहले�तीन�अ�य���के��मश: �ह��, पारसी�और�मु��लम�होने�के�बाद�कां�ेस�का�चौथा�अ�धवेशन�इलाहाबाद�म���आ�तथा�इसने��थम�बार


एक�गैर-भारतीय�को�अपना�अ�य��बनाया।�

ऐसा�करने�म�, इसने�ऐसे�����के�बारे�म���वचार��कया�जो�भारतीय��के�साथ�पहचान�नह��रखता�था��क�तु�उनके�क�याण�और��ग�त�म��उसक�
स�चे�मन�से���च�थी: वह�����थाजॉज��यूल।�

IASbaba
Score:
Web: http://ilp.iasbaba.com/
45.00 /
Email: ilp@iasbaba.com
Page 80 150
2019 - Test 9- History &
Exam Title :
Cu...
Email : vikasutsav@hotmail.com
Contact :

ड��यू. सी. बनज��ने��म�वत�दबाव�बनाकर�उ�ह��कां�ेस�के�इलाहाबाद�अ�धवेशन�क��अ�य�ता�करने�के��लए�तैयार�कर��लया।�

अपने�स�पूण��भारतीय�क�रयर�के�दौरान, जॉज��यूल�ने�अपने�संपक��म��आने�वाले���येक�����का�स�मान, �शंसा�और�आदर��ा�त��कया।�इसम�


भारतीय�और�यूरोपीय, अ�धकारी�और�गैर-अ�धकारी�सब�शा�मल�थे।�

सो�चए�

· वे�अ�य��वदे शी�कौन�थे�जो�INC के�अ�य��बने�?

QUESTION 46.
बहरामजी�मालाबारी�भारत�के�सवा��धक��मुख�पारसी�समाज�सुधारक��म��से�एक�थे।�उनके�बारे�म���न�न�ल�खत�म��से�कौन�से�कथन�स�य�ह��?

1. उ�ह�ने��ह�����ारा��वधवा-पुन�व�वाह�के���तबंध�क��आलोचना�क��और�इसका�आरोप���य�त: उनके�“अ�ील�पूवा��ह�” हेतु�धा�म�क


“पुरो�हत�वग�” तथा�“सामा�जक�एका�धकारवा�दय�” (अथा�त��ा�ण�जा�त) पर�लगाया।�

2. मालाबारी�ने�इ�फ�ट�मै�रज�एंड�ए�फॉ�ड���वडो�ड�पर�नोट् स�का�एक�सं�ह��का�शत��कया�और�इसे�उ�ह�ने��मुख�अं�ेज��और��ह����को
भेजा।�

3. उ�ह�ने�1894 म��ऐज�ऑफ़�कंस�ट�अ�ध�नयम�के�पा�रत�करवाने�म���मुख�भू�मका�अदा�क�।�

4. मालाबारी�ने�इं�ल�ड�म��“एन�अपील��ॉम�द�डॉटज��ऑफ़�इं�डया” नामक�पु�तक��का�शत�क�।�

�न�न�ल�खत�म��से�सही�कूट�का�चयन�क��जए�:

a) 1,2 और�3
b) 2,3 और�4
c) 1,3 और�4
d) उपयु���सभी�
Correct Answer: D
Your Answer:
Explanation

Solution (d)

बहरामजी�मेरवानजी�मालाबारी�एक�भारतीय�क�व, �काशक, लेखक�तथा�सामा�जक�सुधारक�थे�जो��वशेष��प�से�म�हला��के�अ�धकार��के


एक��खर�समथ�क�के�तौर�पर�जाने�जाते�ह�।�

उ�ह�ने��ववाह�के��लए�लड़�कय��क��आयु�बढाए�जाने�के�बारे�म���ापक��प�से��लखा��जसे�भारत�के�लोग��क��सहानुभू�त��ा�त��ई।�उनके
�ा�यान��को�सुनने�वाले�बनारस�और�मथुरा�के�पं�डत��और�शा���य��ने�उनके�तक��के�खरेपन�को��वीकारना�आरंभ�कर��दया।�मालाबारी�ने
इं�ल�ड�म��“एन�अपील��ॉम�द�डॉटज��ऑफ़�इं�डया” नामक�पु�तक��का�शत�क�।�

उ�ह�ने�प�का�रता�को�अपने�पेशे�के��प�म��चूना�और�अपना�जीवन��ह����म���च�लत�बाल-�ववाह�और�थोपे�गए�वैध��क��बुराइय��से�लड़ने�म�
सम�प�त�कर��दया।�वह�एक�भयर�हत�प�कार�थे।�उ�ह�ने�भारत�क���ापक��मण��कया�और�तीन�बार�इं�ल�ड�क��या�ा�क�।�वह�बाल��ववाह�और
उससे�जुड़ी�बुराइय��को�समा�त�करवाने�के�अपने�संघष��म��मुखर�थे।�म�हला��क��मु���क��माँग�करते��ए�उ�ह�ने�वकालत�क�, “य�द�नए�भारत
को�मु��और��बु��बेट��क��एक�पीढ़��आशीवा�द�के��प�म��चा�हए�, एक�ऐसा�दे श�चा�हए�जो�अपने�मामले�खुद�संभाल�सके, तो�आज�के
�ह����को�अपनी�मान�सकता��म��मु�, �बु��माता��क��एक�न�ल�को�दे खना�चा�हए�होगा�” । लोकमा�य�बाल�गंगाधर��तलक�ने�मालाबारी
को�अपने�खुद�के�समुदाय�क��म�हला��के��हत��क��ओर�दे खने�का�सुझाव��दया, �क�तु�मालाबारी�अपने��यास��म��अ�डग�बने�रहे�और�सफल
�ए।�

वह��ह����वधवा��के�पुन�व�वाह�क���वतं�ता�का�मु�ा�उठाकर���स���ए��जस�पर�पूरे�भारत�म��वाद-�ववाद��आ।�1884-1891 के�उनके
सफल�अ�भयान�का�प�रणाम�1894 म��ऐज�ऑफ़�कंस�ट�अ�ध�नयम�के�पा�रत�होने�के��प�म��सामने�आया।�उनका��ख��याय�और�मानवता�के
�स�ांतो�पर�आधा�रत�था।�उनक��अमू�य�“नोट् स�ऑन�इ�फ�ट�मै�रज�एंड�ए�फॉ�ड���वडो�ड” 1894 म���का�शत��ई�तथा�यह�सामा�जक
सुधारक��के��लए�एक�मह�वपूण��पु�तक��स���ई।�

IASbaba
Score:
Web: http://ilp.iasbaba.com/
45.00 /
Email: ilp@iasbaba.com
Page 81 150
2019 - Test 9- History &
Exam Title :
Cu...
Email : vikasutsav@hotmail.com
Contact :
QUESTION 47.
19 व��सद��के�उ�रा�� �ने�वना�कुलर��ेस�म��एक�उ�लेखनीय�वृ���दे खी�। इ�होने�जनसमुदाय�म��सामा�जक-राजनी�तक�चेतना�पैदा�करने�म��एक
मह�वपूण��भू�मका��नभाई�।�वे����टश�सरकार�क��शोषक�नी�तय��का�पदा�फाश�करने�म��सहायक�भी�बने।�इस�वृ���से�सचेत�होकर�सरकार�ने
एक�वना�कुलर��ेस�अ�ध�नयम�पा�रत��कया।�इस�अ�ध�नयम�के�बारे�म���न�न�ल�खत�म��से�कौन�से�कथन�सही�ह��?

1. यह�1878 म��लॉड���रपन��ारा�पा�रत��कया�गया�था�।

2. �जल��के�म�ज��े ट��को��कसी�भी��कार�के�छपाईकता��और��काशक�को�बुलाकर�उनसे�एक�बॉ�ड�(जो��क�सरकार�के��व���वैमन�य�क�
भावना��को�“उकसावा” दे �सकने�वाली�कोई�भी�साम�ी�न�छापने�का�वचनप��होता�था) भरवाने�क��श����ा�त�थी�।

3. म�ज��े ट�को�एक�जमानत�रा�श�जमा�करवाने�का�अ�धकार��ा�त�था�जो�छपाईकता���ारा�बॉ�ड�का�उ�लंघन��कए�जाने�पर�ज�त�क��जा
सकती�थी�।

�न�न�ल�खत�म��से�कूट�का�चयन�क��जए:

a) केवल�1 और�2
b) केवल�2 और�3
c) केवल�1 और�3
d) उपयु���सभी�
Correct Answer: B
Your Answer:
Explanation

Solution (b)

वना�कुलर��ेस�अ�ध�नयम�लॉड���लटन��ारा�पा�रत��कया�गया�था�और�आगे�चलकर�लॉड���रपन��ारा�इसे�र��कर��दया�गया�।

इस�अ�ध�नयम�के��ावधान����म��सही��कार��दए�गए�ह��।

QUESTION 48.
बंगाल�के�नील��व�ोह�के�बारे�म���न�न�ल�खत�कथन��पर��वचार�क��जए�:

1. यह�मु�य��प�से�अ�ह�सक�था�और��कसान��ने��वरोध�के�कानूनी�तरीक��का��योग��कया।�

2. आधु�नक��श��त�म�यमवग��ने�का�तकार��का�समथ�न��कया।�

3. सरकार��ारा�नील�आयोग�क���नयु���क��गई��जसने��लांटस��को�दोषी�ठहराया�और�उन��ारा�नील�क��खेती�करने�वाल��के���त�अवपीड़क
�व�धय��के��योग�हेतु�उनक��आलोचना�क��।

उपयु���म��से�कौन�से�कथन�स�य�ह��?

a) केवल�1
b) 2 और�3
c) 1 और�3
d) उपयु���सभी�
Correct Answer: D
Your Answer:
Explanation

Solution (d)

नील��व�ोह�:

IASbaba
Score:
Web: http://ilp.iasbaba.com/
45.00 /
Email: ilp@iasbaba.com
Page 82 150
2019 - Test 9- History &
Exam Title :
Cu...
Email : vikasutsav@hotmail.com
Contact :

�व�ोह�बंगाल�के�ना�दया��जले�के�गो�व�दपुर��ाम�से�आरंभ��आ�जहाँ��ब�वास�बंधु��ने�नील�क��खेती��याग�द��।�इसके�बाद�ल�ठयाल��के�साथ
संघष���आ�तथा��व�ोह�बंगाल�के�कई�भाग��म��फ़ैल�गया�।�हड़ताल�, कानूनी�काय�वा�हयाँ, �ह�सा, �लांटस��का�सामा�जक�ब�ह�कार�आ�द��व�ोह�म�
�योग��कए�गए�कुछ�उपकरण�थे�।��कसान�संगठन��तथा�कुछ�हद�तक��ह��-मु��लम�एकता�तथा�बंगाल�के�बु��जीवी�वग��के�समथ�न�ने��व�ोह�को
और��भावी�बनाया।�अंतत: नील�आयोग�क���नयु���क��गई��जसने��लांटस��को�दोषी�ठहराया�और�उन��ारा�नील�क��खेती�करने�वाल��के���त
अवपीड़क��व�धय��के��योग�हेतु�उनक��आलोचना�क��। कंपनी�ने�रैयत��को�उनके�वत�मान�अनुबंध��को�पूरा�करने�को�कहा��क�तु�यह�भी�कहा��क
वे�भ�व�य�म��नील�उ�पादन�करने�से�मना�कर�सकते�ह��। यह��कसान��के��लए�एक�बड़ी�राहत�थी�और�बंगाल�म��धीरे-धीरे�नील�क��खेती�समा�त�हो
गई�।

यह�आंदोलन�मु�य��प�से�अ�ह�सक�था��जसम��ब�त�कम�घटनाएं��ई�।�इस��व�ोह�के��मुख�नेता��म��ना�दया�के��ब�वास�बंधु�{ �दगंबर
�ब�वास�तथा��ब�णु��ब�वास�}, पाबना�के�कादर�मु�ला, मैदा�के�रफ़�क�मंडल�आ�द�थे�।

इन�नील��कसान��क���द� शा�का�वण�न�द�न�बंधु��म��के�बंगाली�नाटक�‘नील�दप�ण’ म���कया�गया�है�।

QUESTION 49.
1885 म��भारतीय�रा�ीय�कां�ेस�क���थापना�के�प�ात�भारतीय�रा�ीय�आंदोलन�को�एक�ग�त��मली।�1906 तक�इसम��उदारवा�दय��का
वच��व�रहा।�उदारवा�दय��क���न�न�ल�खत�म��से�कौन�सी�मांगे�थी�?

1. भारत�हेतु�डो�म�नयन�दजा�।�

2. कृ�ष�और�आधु�नक�उ�ोग��का�ती���वकास�कर�गरीबी�को�हटाना।�

3. उ�च��शास�नक�सेवा��का�भारतीयकरण।�

4. बोलने�और��ेस�क��आजाद�।�

�न�न�ल�खत�म��से�कूट�का�चयन�क��जए�:

a) 1,2 और�3
b) 2,3 और�4
c) 1,3 और�4
d) उपयु���सभी�
Correct Answer: B
Your Answer:
Explanation

Solution (b)

1885 से�1905 तक�कां�ेस�नेता��के�ल�य�और�उ�े �य�:

रा�ीय�आंदोलन�के�आरं�भक�चरण�म��दादाभाई�नौरोजी, सुर���नाथ�बनज�, �फरोजशाह�मेहता, गोपाल�कृ�ण�गोखले�आ�द�का�वच��व�था।�उनका


शां�तपूण��तथा�संवैधा�नक��व�धय��म���व�ास�था।�

उ�ह�ने��न�न�ल�खत�उ�े �य��क���ा��त�का��यास��कया�:

· प�रषद��के��लए��ापक�श��य��क��माँग�तथा��व-शासन�म����श�ण।�

· कृ�ष�और�आधु�नक�उ�ोग��का�ती���वकास�कर�गरीबी�को�हटाना।�

· उ�च��शास�नक�सेवा��का�भारतीयकरण।�

· अपने�नाग�रक�अ�धकार��क��र�ा�हेतु�बोलने�और��ेस�क��आजाद�।�

उदारवाद��नेता��ने�इस��कार�रा�ीय�चेतना�जगाने�और����टश�सा�ा�यवाद�के��व���आम�राय�बनाने�का��यास��कया।�उ�ह�ने�रा�ीय�संघष�
के��लए�एक�साझे�मंच�का��नमा�ण��कया।�

IASbaba
Score:
Web: http://ilp.iasbaba.com/
45.00 /
Email: ilp@iasbaba.com
Page 83 150
2019 - Test 9- History &
Exam Title :
Cu...
Email : vikasutsav@hotmail.com
Contact :

उदारवाद��नेता��के�तरीके�:

· उ�ह�ने�अपने�ल�य��क���ा��त�के��लए�शां�तपूण��एवं�संवैधा�नक�साधन�अपनाए।�

· उनका�अं�ेज��क���याय��णाली�म��पूरा�भरोसा�था, अत: उ�ह�ने����टश�शासक��के��व���एक��म�वत�रवैया�अपनाया।�

· उ�ह��संवैधा�नक�सुधार��म���व�ास�था।�वे����टश�सरकार�को�इस�उ�मीद�म��या�चकाएं�भेजते�थे��क�वे�उ�ह��आजाद��दे �द� गे।�

· हालां�क, उदारवाद��अपने�ल�य�क���ा��त�म��असफल�रहे, �य��क����टश�सरकार�ने�उनके�साथ�सहयोग�नह���कया।�इसने�1905 के�प�ात


कां�ेस�म��चरमपंथ�का�माग���श�त��कया।�

QUESTION 50.
�न�न�कथन�म���कसका�उ�रण��दया�जा�रहा�है�?

“ अब�जनसमूह�भारत�क��एकमा��आशा�है।�उ�च�वग��शारी�रक�और�नै�तक��प�से�मृत�पड़ा�है।�”

a) महा�मा�गाँधी�
b) र�ब��नाथ�टै गोर�
c) सरदार�पटे ल�
d) �वामी��ववेकानंद�
Correct Answer: D
Your Answer:
Explanation

Solution (d)

�वत: �प��

यह�व����वामी��ववेकानंद��ारा�उ�च�जा�तय��और�उ�च�अमीर�वग��क��आलोचना�के��लए��दया�गया�था�जो�उनके��हसाब�से����गत
�वला�सता��म��डू बे��ए�थे�और�मातृभू�म�क��उ�ह��कोई��च�ता�नह��थी�।

QUESTION 51.
‘ इंड���यल��स�यू�रट��एने�स�(ISA) हाल�म���कस�बात�को�लेकर�चचा��म��था-

a) �लोबल�एनवायरनम�ट�फै�स�लट��को�लेकर�
b) अंतरा��ीय�परमाणु�उजा��एज�सी�को�लेकर�
c) परमाणु�आपू�त�कता��समूह�को�लेकर�
d) भारत-अमे�रका�र�ा�सहयोग�को�लेकर�
Correct Answer: D
Your Answer:
Explanation

Solution (d)

इंड���यल��स�यू�रट��एने�स�(ISA) भारतीय��नजी��े��को�सं.रा. अमे�रका�के�र�ा�उ�ोग�के�साथ�सहयोग�क��अनुम�त�दे गा।�

Source: https://www.thehindu.com/news/national/india-us-sign-landmark-comcasa-
deal/article24881277.ece

QUESTION 52.
‘ �व�श�ग�’ मद�का��योग��कस�स�दभ��म���कया�जाता�है-

IASbaba
Score:
Web: http://ilp.iasbaba.com/
45.00 /
Email: ilp@iasbaba.com
Page 84 150
2019 - Test 9- History &
Exam Title :
Cu...
Email : vikasutsav@hotmail.com
Contact :

a) ���टोकर�सी�
b) इ�न�शयल�प��लक�ऑफ�र�ग�(IPOs)
c) �थम�सूचना��रपोट� �(FIRs)
d) उपरो��म��से�कोई�नह��
Correct Answer: D
Your Answer:
Explanation

Solution (d)

“ �व�श�ग�” अथवा�वोइस�“�फ�श�ग”

अपने�आप�को�ब�क�का���त�न�ध�बताकर�ये�लोग�शक�न�करने�वाले�खाता�धारक��को�फोन�करते�ह��ता�क��व�भ��बहाने�बनाकर�उनक�
���गत�जानकारी��ा�त�कर�सक�।�

यह�सूचना�धन�को�ई-वॉलेट�म��अवैध�तरीके�से�ह�तांत�रत�करने�हेतु��योग�क��जाती�है।�

Source: https://www.thehindu.com/news/national/lens-on-premises-of-jharkhand-cyber-
crime-gangs/article24884642.ece

QUESTION 53.
�कसी�को�उसके�जीवन�को�लंबा�बनाने�का�कोई�उपाय��कए��बना�मरने�दे ना�कहलाता�है-

a) �वै��छक�इ�छामृ�यु�
b) �न���य�इ�छामृ�यु�
c) गैर-�वै��छक�इ�छामृ�यु�
d) स��य�इ�छामृ�यु�
Correct Answer: B
Your Answer:
Explanation

Solution (b)

�न���य�इ�छामृ�यु�: उपचार�अथवा�जीवन�बढाने�वाली��च�क�सक�य�सहायता�हटा�दे ना।�

QUESTION 54.
�न�न�ल�खत�म��से�कौन�सा�रा�य�बां�लादे श�के�साथ�सबसे�लंबी�सीमा�साझा�करता�है�?

a) असम�
b) ��पुरा�
c) �मजोरम�
d) मेघालय�
Correct Answer: B
Your Answer:
Explanation

Solution (b)

बां�लादे श�और�भारत�4,156 �कमी�(2,582 मील�) लंबी�अंतरा��ीय�सीमा�साझा�करते�ह�।�यह��व��क��पाँचवी�सबसे�लंबी�भू-सीमा�है�जो


असम�म��262 �कमी�(163 मील�), ��पुरा�म��856 �कमी�(532 मील�), �मजोरम�म��180 �कमी�(110 मील�), मेघालय�म��443 �कमी�(2
75 मील�), तथा�प. बंगाल�म��2,217 �कमी�(1,378 मील�) पड़ती�है।�

IASbaba
Score:
Web: http://ilp.iasbaba.com/
45.00 /
Email: ilp@iasbaba.com
Page 85 150
2019 - Test 9- History &
Exam Title :
Cu...
Email : vikasutsav@hotmail.com
Contact :

Source: https://www.thehindu.com/news/national/other-states/assam-governor-flags-
cattle-smuggling-on-the-border/article24886692.ece

QUESTION 55.
भारतीय�दं ड�सं�हता�(IPC) क��धारा�377 के�बारे�म���न�न�ल�खत�कथन��पर��वचार�क��जए।�

1. यह�इं�ल�ड�के�‘बगरी�अ�ध�नयम�1533’ को���तमान�मानकर�बनाई�गई�है।�

2. इसे�भारत�म��‘�सपाही��व�ोह’ होने�के�बाद�लाया�गया�था।�

3. सव��च��यायालय�ने�‘नवतेज��स�ह�जोहर�बनाम�भारत�संघ�वाद’ म��सव�सहम�त�से��व�था�द���क�धारा�377 ‘उस�सीमा�तक�असंवैधा�नक


है�जहाँ�तक�यह�समान��ल�ग�वाले�वय�क��के�बीच�सहम�त�से�बने�यौन�संबंध��को�अपराध�ठहराती�है’।�

सही�कथन�चु�नए�

a) 1 और�3
b) 2 और�3
c) केवल�3
d) 1, 2 और�3
Correct Answer: D
Your Answer:
Explanation

Solution (d)

भारतीय�दं ड�सं�हता�(IPC) क��धारा�377 भारतीय�दं ड�सं�हता�क��वह�धारा�है��जसे�भारत�म�����टश�शासन�के�दौरान�1861 म��लाया�गया


था।�इं�ल�ड�के�‘बगरी�अ�ध�नयम�1533’ को���तमान�मानकर�लाई�गई�यह�धरा�“ �कृ�त�के��नयम�के��व���” होने�वाली�यौन�ग�त�व�धय��को
अपराध�मानती�है।�6 �सतंबर�2018 को�भारत�के�सव��च��यायालय�ने�घो�षत�करने�का��नण�य��लया��क�वय�क��के�बीच�सहम�त�से�बने
समल��गक�संबंध��पर�धारा�377 का�लागू�होना�असंवैधा�नक, “ अता�क�क, अर�णीय�और��प���प�से��वे�छाचारपूण��” होगा�, �क�तु�धारा�3
77 अ�पवय�क�, �बना�सहम�त�से�बने�संबंधो�तथा�मानव-पशु�यौन�संबंध��पर�लागू�रहेगी�।

6 �सतंबर�2018 को�सव��च��यायालय�ने�‘नवतेज��स�ह�जोहर�बनाम�भारत�संघ�वाद’ म��सव�सहम�त�से��व�था�द���क�धारा�377 ‘उस�सीमा


तक�असंवैधा�नक�है�जहाँ�तक�यह�समान��ल�ग�वाले�वय�क��के�बीच�सहम�त�से�बने�यौन�संबंध��को�अपराध�ठहराती�है’।�

धारा�377 �नण�य�- मु�य�बात��

· सव��च��यायालय�ने�भारतीय�दं ड�सं�हता�(IPC) क��धारा�377 के�उन��ावधान��को�समा�त�कर��दया�जो�समल��गक�यौन�संबध��को�अपराध


ठहराते�थे�

· सव��च��यायालय�ने��व�था�द���क�यह�क़ानून�नाग�रक��के�मौ�लक�अ�धकार��का�उ�लंघन�करता�है।�

· �यायालय�ने�रेखां�कत��कया��क�IPC क��धारा�377 का��योग�LGBTQ समुदाय�के�लोग��को�उ�पी�डत�करने�के��लए��कया�जाता�है


�जसके�प�रणाम�व�प�भेदभाव�उ�प��होता�है।�

· ब�मत�क��राय�तथा�लोक��य�नै�तकता�संवैधा�नक�अ�धकार��को�आदे �शत�नह��कर�सकती।�

· �नण�य�म��कहा�गया��क�इ�तहास�LGBTQ समुदाय�के���त��मा�ाथ��है।�साथी�का�चुनाव��नजता�का�मौ�लक�अ�धकार�है।�

· इसम��कहा�गया��क�यह�धारा�पशुगमन�जैसे�“अ�ाकृ�तक” यौनाचार��पर�लागू�रहेगी।��बना�सहम�त�के�बनाए�गए�यौन�संबंध�धारा�377 के
अंतग�त�अपराध�क���ेणी�म��बने�रह�गे।�

Source: https://www.thehindu.com/opinion/lead/drawing-a-curtain-on-the-past/
article24937833.ece

IASbaba
Score:
Web: http://ilp.iasbaba.com/
45.00 /
Email: ilp@iasbaba.com
Page 86 150
2019 - Test 9- History &
Exam Title :
Cu...
Email : vikasutsav@hotmail.com
Contact :
QUESTION 56.
भारत�ने�सं.रा. अमे�रका�के�साथ��न�न�ल�खत�म��से�कौन�से�बु�नयाद��समझौत��पर�ह�ता�र��कए�ह��?

1. जनरल��स�यू�रट��ऑफ़��म�ल���इनफाम�शन�ए�ीम�ट�(GSOMIA)

2. बे�सक�ए�सच�ज�एंड�कोऑपरेशन�ए�ीम�ट�फॉर��जयो-�पे�सयल�कोऑपरेशन�(BECA)

3. क�यु�नकेशंस�क�पे�ट�ब�लट��एंड��स�यू�रट��ए�ीम�ट�(COMCASA)

4. लो�ज��ट�स�ए�सच�ज�मेमोर�डम�ऑफ़�ए�ीम�ट�( LEMOA)

सही�कूट�चु�नए�:

a) केवल�3 और�4
b) 1, 3 और�4
c) 2, 3 और�4
d) उपयु���सभी�
Correct Answer: B
Your Answer:
Explanation

Solution (b)

भारत�ने�बे�सक�ए�सच�ज�एंड�कोऑपरेशन�ए�ीम�ट�फॉर��जयो-�पे�सयल�कोऑपरेशन�(BECA) को�छोड़कर�सभी�समझौत��पर�ह�ता�र�कर
�दए�ह�।�

COMCASA

· भारत�तथा�अमे�रका�ने�गु�वार�को�मह�वपूण��क�यु�नकेशंस�क�पे�ट�ब�लट��एंड��स�यू�रट��ए�ीम�ट�(COMCASA) समझौते�पर�ह�ता�र
�कए।�

· इससे�अमे�रका�उसके�यहाँ�से�उ��मत�सी�-17, सी�-130 तथा�पी�-8I एयर�ा�ट�जैसे��लेटफॉ�स��हेतु�आव�यक�कूटब��संचार��हेतु


�वशेषीकृत�उपकरण�ह�तांत�रत�कर�पाएगा।�

· यह�तुरंत�ही��भावी�हो�गया�है�और�10 वष��तक�वैध�रहेगा।�

जनरल��स�यू�रट��ऑफ़��म�ल���इनफाम�शन�ए�ीम�ट�(GSOMIA)

· यह�अमे�रक��सरकार�तथा�अमे�रक��कंप�नय��से��ा�त�गोपनीय�सूचनाएं�भारत�सरकार�और�र�ा��े��के�साव�ज�नक��े�क�उप�म��(PSUs)
(�क�तु��नजी�भारतीय�कंप�नय��के�साथ�नह�) के�साथ�साझा�करने�क��अनुम�त��दान�करता�है।�

Read More - https://www.financialexpress.com/defence/what-is-comcasa-security-


agreement-between-india-us-signed-during-22-summit-a-boost-for-defence-
preparedness/1304439/

Source: https://www.thehindu.com/news/national/lemoa-already-fully-operational/
article24904359.ece

QUESTION 57.
�डफे�स�टे �नोलॉजी�एंड��े ड�इ�न�शए�टव�(DTTI) को��ाय: भारत�और��कस�अ�य�दे श�के�संदभ��म��सु�ख़�य��म��दे खा�जाता�है-

a) सं.रा. अमे�रका�
b) इजराइल�
c) जापान�

IASbaba
Score:
Web: http://ilp.iasbaba.com/
45.00 /
Email: ilp@iasbaba.com
Page 87 150
2019 - Test 9- History &
Exam Title :
Cu...
Email : vikasutsav@hotmail.com
Contact :

d) �ांस�
Correct Answer: A
Your Answer:
Explanation

Solution (a)

समाचार�: अमे�रका�क���डफे�स�इनोवेशन�यू�नट�(DIU) तथा�इं�डयन��डफे�स�इनोवेशन�आग�नाईजेशन�– इनोवेशन�फॉर��डफे�स�ए�सील�स�


(DIO-iDEX) के�बीच�MoU पर�ह�ता�र��कए�गए�जो��डफे�स�टे �नोलॉजी�एंड��े ड�इ�न�शए�टव�(DTTI) के�मा�यम�से�सह-उ�पादन�तथा
सह-�वकास�क��प�रयोजना��को�दे खेगा।�

DTTI का�उ�े �य�है�:

· ��प�ीय�र�ा�संबंध��को�ऐसा�बनाना��क�उनक��सीमाएं�केवल��वतं��साम�रक��नण�य��को�लेकर�ह��न��क�नौकरशाही�संबधी�बाधा��और
अकुशल����या��को�लेकर।�

· पारंप�रक�“�ेता-�व�ेता: संबंध��से�एक�अ�धक�सहयोगी����कोण�क��ओर�गमन�के�मा�यम�से�भारत�के�र�ा�उ�ोग�आधार�को�मजबूत
करना।�

· सह-�वकास�तथा�सह-उ�पादन�के�मा�यम�से��कए�जाने�वाले��व�ान�तथा��ौ�ो�गक��सहयोग�से��ौ�ो�गक�य��वकास�के�नए��े���का�अ�वेषण
करना।�

· अमे�रका-भारत��ापार�संबध��का��व�तार�करना।�

Source: https://www.thehindu.com/todays-paper/tp-national/indias-interests-secured-
in-new-pact-officials/article24887267.ece

QUESTION 58.
�कसी�दे श�के�संदभ��म��, ‘CENTRIXS’ तक�प�ँच�बनाने�के��या��न�हताथ��ह�?

a) उस�दे श�को�परमाणु�आपू�त�कता��समूह�से�यूरे�नयम�खरीदने�का��वशेषा�धकार��ा�त�हो�जाएगा।�
b) वह�दे श��वत: परमाणु�आपू�त�कता��समूह�का�सद�य�बन�जाएगा।�
c) उस�दे श�के�सै�य���त�ान�अंतरा��ीय�परमाणु�उजा��एज�सी�(IAEA) के��नरी�ण�के�अंतग�त�आ�जाएंगे।�
d) उपरो��म��से�कोई�नह��
Correct Answer: D
Your Answer:
Explanation

Solution (d)

स�ट�र�स�( CENTRIXS )

· कंबाइंड�एंटर�ाइज�रीजनल�इनफाम�शन�ए�सच�ज��स�टम�या�CENTRIXS, अमे�रका�क��एक�सुर��त�संचार��णाली�का�नेटवक��है।�

· भारत�इस�तक�COMCASA के�एक�भाग�के��प�म��प�ँच�बनाएगा।�

· CENTRIXS लगे�नौसेना�के�जहाज�अमे�रक��नौसेना�के�साथ�आव�यकता�पड़ने�पर�सुर��त�ढं ग�से�संचार�कर�सकते�ह��तथा�उनके��ारा


तैनात�जहाज़��और��वमान��क��बड़ी�सं�या��व�मान�होने�के�कारण��े��क���ापक�प�र��थ�तज�य�त�वीर��ारा�लाभ��ा�त�कर�सकते�ह�।�

· CENTRIXS म��एक�कले�शन�ऑफ़�कोए�लशन�वाइड�ए�रया�नेटवक��(WAN) लगा�है��जसे�‘ए��लेव’ कहा�जाता�है।�

· यह�एक�मह�वपूण��स�मकता��है�जो�दो�दे श��के�बीच�श�द��तथा�वेब-आधा�रत��ा�प�म���शप-टू -�शप�संचालनीय�वाता��करने�क��अनुम�त�दे ता


है।�

IASbaba
Score:
Web: http://ilp.iasbaba.com/
45.00 /
Email: ilp@iasbaba.com
Page 88 150
2019 - Test 9- History &
Exam Title :
Cu...
Email : vikasutsav@hotmail.com
Contact :

Source: https://www.thehindu.com/todays-paper/tp-national/india-can-track-chinas-
ocean-moves-better/article24897825.ece

QUESTION 59.
‘ चेक��रप��लक�’ क��सीमा��न�न�ल�खत�म��से��कस�दे श�से�नह��लगती�?

1. जम�नी�

2. �लोवा�कया�

3. पोल�ड�

4. �लोवे�नया�

सही�कूट�चु�नए�:

a) 1 और�3
b) 2 और�4
c) केवल�3
d) केवल�4
Correct Answer: D
Your Answer:
Explanation

Solution (d)

चेक��रप��लक�म�य�यूरोप�म��एक�भू-आब��दे श�है��जसक��सीमा�प��म�म��जम�नी, द��ण�म��ऑ���या, पूव��म���लोवा�कया�तथा�पूव��र�म�


पोल�ड�से�लगती�है।�

QUESTION 60.
‘ मो�बलाइज�योर��सट��’ साझेदारी�के��वषय�म���न�न�ल�खत�कथन��पर��वचार�क��जए।�

1. यह�सं.रा. मरा�केश�पाट� नर�शप�फॉर��लोबल��लाइमेट�ए�शन�के�अंतग�त�एक�अंतरा��ीय�प�रवहन�गठबंधन�है।�

IASbaba
Score:
Web: http://ilp.iasbaba.com/
45.00 /
Email: ilp@iasbaba.com
Page 89 150
2019 - Test 9- History &
Exam Title :
Cu...
Email : vikasutsav@hotmail.com
Contact :

2. इसका�उ�े �य�लाभाथ��साझेदार�(अथा�त�रा�ीय�और��थानीय�सरकार�) को�उनक��नेशनल�अब�न�मो�ब�लट��पॉ�लसीस�एंड�इ�वे�टम�ट��ो�ा�स


(NUMPs) तथा�स�टे नेबल�अब�न�मो�ब�लट���ला�स�(SUMPs) को�तैयार�करने�म��सहयोग�करना�है।�

3. इसे��ांस�और�जम�नी�क��सरकार��का�अनुसमथ�न��ा�त�है�और�कां��स�ऑफ़�पाट�ज�(COP21) क��21 व��बैठक�म��इसक��शु�आत�क�


गई�थी।�

सही�कथन�चु�नए�

a) 1 और�2
b) 2 और�3
c) 1 और�3
d) उपयु���सभी�
Correct Answer: D
Your Answer:
Explanation

Solution (d)

मो�बलाइज�योर��सट��(MYC)

· मो�बलाइज�योर��सट��साझेदारी�शहर��और�दे श��का�एक�वै��क�एवं�समावेशी�नेटवक��है�तथा�साथ�ही�यूरोपीय��वकास�सहयोग�का�एक
अ��ेला��ांड�भी�है�जो��वशेष��प�से�धारणीय�शहरी�प�रवहन�के��े��से�संबं�धत�है।�इस�साझेदारी�का�उ�े �य�लाभाथ��साझेदार�(अथा�त�रा�ीय
और��थानीय�सरकार�) को�उनक��नेशनल�अब�न�मो�ब�लट��पॉ�लसीस�एंड�इ�वे�टम�ट��ो�ा�स�(NUMPs) तथा�स�टे नेबल�अब�न�मो�ब�लट�
�ला�स�(SUMPs) को�तैयार�करने�म��सहयोग�करना�है।�यह��भावी��नवेश�और�शहरी�प�रवहन�अवसंरचना�व�सेवा��के�सतत��वकास�के
साथ-साथ��व�नयामक�य�मृ��उपाय��हेतु��ेमवक��दशा��क���थापना�को�सुगम�बनाता�है।�

· यह�अंतरा��ीय�पहल�का�एक�भाग�है��जसे��ांस�और�जम�नी�क��सरकार��का�अनुसमथ�न��ा�त�है�और�कां��स�ऑफ़�पाट�ज�(COP21) क�
�दसंबर�2015 म���ई�21 व��बैठक�म��इसक��शु�आत�क��गई�थी।।�

· इसके�अ�त�र��, मो�बलाइज�योर��सट��साझेदारी�उभरते, �वकासशील�तथा�ईयू�के�पड़ोसी�दे श��म��एक�कृत�अब�न�मो�ब�लट���ला�न�ग�हेतु�एक


वै��क�जलवायु�साझेदारी�है�तथा�मरा�केश�पाट� नर�शप�फॉर��लोबल��लाइमेट�ए�शन�के�अंतग�त�एक�अंतरा��ीय�प�रवहन�गठबंधन�है।�यह�एक
म�ट�-डोनर�ए�शन�है��जसे�संयु���प�से�यूरोपीय�आयोग�के�डायरे�टरेट-जनरल�फॉर�इंटरनेशनल�कोऑपरेशन�एंड�डेवलपम�ट�(DG
DEVCO), �ांस�क���म�न����ऑफ़�इकोलॉ�जकल��ांजीशन�एंड�सॉ�लडे�रट��(MTES), �ांस�क��फै�स�लट��फॉर��लोबल�एनवायरनम�ट�(F
FEM), तथा�जम�न�फ़ेडरल��म�न����फॉर�द��एनवायरनम�ट, नेचर�का�सव�शन�एंड��यू��लयर�से�ट��(BMU) �ारा��व�पो�षत��कया�जा�रहा
है।�यह�साझेदारी�संयु���प�से�इसके�काया��वयन�साझेदार��AFD, CEREMA, CODATU, EBRD, GIZ, KfW, तथा�WUPPER
TAL INSTITUTE �ारा�लागू�क��जा�रही�है।�अंतरा��ीय�जलवायु����या�को�योगदान�दे ने�के�अ�त�र��यह�सं.रा. के�एज�डा�2030 {�वशे
षकर�सतत��वकास�ल�य�(SDG) 11: Make cities inclusive, safe, resilient and sustainable } म��भी�योगदान�करती
है।�

समाचार��म��

· 2015 म��Agence Française de Développement (AfD) �ारा��कए�गए���ताव�के�आधार�पर, यूरोपीय�संघ�ने�AFD के


मा�यम�से�मो�बलाइज�योर��सट��(MYC) के�भीतर��वशेषीकृत��नवेश��तथा�तकनीक��सहायता�अवयव��म��योगदान�दे ने�के��लए�3.5 �म�लयन
यूरो�दे ने�क��सहम�त�द��है।�

· इसका�ल�य�तीन�पायलट�शहर��नामत: नागपुर, को�ची�तथा�अहमदाबाद�को�सहयोग�दे ने�का�है।�यह�सहयोग�सतत�प�रवहन�नी�त�म��सुधार


हेतु��थानीय��तर�पर�अब�न�मो�ब�लट���ला�स�लागू�कर�शहरी�प�रवहन�से�संबं�धत��ीन�हाउस�गैस�(GHG) के�उ�सज�न��को�कम�करके�तथा
रा�ीय��तर�पर�भारत�क��सहायता�करके��कया�जाएगा।�

· काय��म�के�अंतग�त�चुने�गए�तीन�पायलट�शहर��के�साथ-साथ�MoHUA भी�टे ��नकल�अ�स�ट� स�ग�त�व�धय��से�लाभ��ा�त�करेगा।�

��ता�वत�सहायता�के�मु�य�अवयव�ह��:

· सतत�शहरी�प�रवहन�प�रयोजना��के��नयोजन�और�काया��वयन�को�समथ�न�दे ना�,

IASbaba
Score:
Web: http://ilp.iasbaba.com/
45.00 /
Email: ilp@iasbaba.com
Page 90 150
2019 - Test 9- History &
Exam Title :
Cu...
Email : vikasutsav@hotmail.com
Contact :

· अब�न�मो�ब�लट��के��व�नयमन, आगे�ले�जाने�तथा��नयोजन�क��सं�थागत��मता�को�मजबूत�बनाने�को�समथ�न�दे ना�तथा�

· उ�म��वहार��के��व�नमय�हेतु�भारत�के�अ�य�शहर��के�साथ�ल�न�ग�तथा�ए�सच�ज�फोम�ट्स।�

Source: http://pib.nic.in/PressReleseDetail.aspx?PRID=1545277

QUESTION 61.
‘ ��लने�स�’ के�बारे�म���न�न�ल�खत�म��से�कौन�सा�कथन�स�य�है�?

a) यह�भारत�और��ीलंका�के�बीच�एक�नौसै�नक�यु�ा�यास�है।�
b) यह�भारत�और��स�गापुर�के�बीच�एक�एयर�कॉ�बैट�ए�सरसाइज�है।�
c) यह�भारत�और��स�गापुर�के�बीच�एक�नौसै�नक�यु�ा�यास�है।�
d) यह�भारत�और��ीलंका�के�बीच�एक�एयर�कॉ�बैट�ए�सरसाइज�है।�
Correct Answer: A
Your Answer:
Explanation

Solution (a)

��लने�स�( �ीलंका�भारत�नौसेना�अ�यास�) भारतीय�नौसेना�तथा��ीलंका�क��नौसेना�के�बीच�नौसै�नक�अ�यास��क��एक��ृंखला�है।�

Source: http://pib.nic.in/newsite/PrintRelease.aspx?relid=183328

QUESTION 62.
‘ ई�ट�गोदावरी��रवर�ए�चूएराइन�इको�स�टम�(EGREE)’ के�बारे�म���न�न�ल�खत�कथन��पर��वचार�क��जए।�

1. गोदावरी�मै��ोव�के�चार��ओर���थत�EGREE प. बंगाल�म��सुंदरबन�के�प�ात��सरा�सबसे�मै��ोव��े��है।�

2. इसे��लोबल�एनवायरनम�ट�फै�स�लट��(GEF) �ारा��व�पो�षत��कया�जाता�है।�

सही�कथन�चु�नए�

a) केवल�1
b) केवल�2
c) 1 और�2 दोन��
d) न�तो�1 न�ही�2
Correct Answer: C
Your Answer:
Explanation

Solution (c)

ई�ट�गोदावरी��रवर�ए�चूएराइन�इको�स�टम�(EGREE) �े��

· आं��दे श�म����थत�EGREE भारत�के�पूव��तट�पर���थत��सरा�सबसे�बड़ा�मै��ोव�वन�है।�

· इसके�रा�ीय�और�वै��क�जैव-�व�वधता�मह�व�को�मा�यता�दे ते��ए�EGREE के�एक��ह�से�को�को�र�गा�व�यजीव�अ�यारण�के��प�म�


अ�धसू�चत��कया�गया�है।�

· इस��े��म���ामीण�समुदाय��नवास�करते�ह��जो�आजी�वका�हेतु�मै��ोव�तथा�अ�य�समु���संसाधन��पर�आ��त�ह�।�

· EGREE फाउंडेशन�क���थापना�आं��दे श�सोसाइट��पंजीकरण�अ�ध�नयम�2001 के�अंतग�त�क��गई�है।�

IASbaba
Score:
Web: http://ilp.iasbaba.com/
45.00 /
Email: ilp@iasbaba.com
Page 91 150
2019 - Test 9- History &
Exam Title :
Cu...
Email : vikasutsav@hotmail.com
Contact :

· यह�फाउंडेशन�EGREE म��संचा�लत�हो�रहे�उ�पादन��े���के�मा�यम�से�जैव-�व�वधता�संर�ण�पहल��के�काया��वयन�को�सुगम�बनाने�हेतु�एक
�ॉस-से�टोरल��लेटफाम��है।�

· यह�EGREE �े��म��पादपजात�और��ा�णजात�के�संर�ण�हेतु�GOI-UNDP-GEF क��एक�प�रयोजना�है।�

Source: https://www.thehindu.com/news/national/andhra-pradesh/pondicherry-shark-
spotted-near-kakinada/article24910627.ece

QUESTION 63.
‘ बथूक�मा�उ�सव�’ ‘�गा��नवरा�ी’ के�दौरान�नौ��दन��तक�मनाया�जाने�वाला�एक�पु�पो�सव�है।�यह�मु�यत: �न�न�ल�खत�म��से��कस�रा�य�म�
मनाया�जाता�है�?

a) गुजरात�
b) तेलंगाना�
c) प. बंगाल�
d) म�य��दे श�
Correct Answer: B
Your Answer:
Explanation

Solution (b)

बथूक�मा�उ�सव�

· यह�एक�पु�पो�सव�है�जो�मु�यत: तेलंगाना�क���ह���म�हला���ारा�मनाया�जाता�है।�

· यह��गा��नवरा�ी�के�दौरान�नौ��दन��तक�मनाया�जाता�है।�

· बथूक�मा�फूल��का�एक�अंबार�होता�है�जो�इस��कार�से��व��थत��कए�जाते�ह��जैसे��कसी��ह���मं�दर�का�गोपुरम�ह�।�

· तेलंगाना�क��सं�कृ�त�को���त�ब��बत�करने�वाले�इस�उ�सव�को�इसके�मह�व�के�कारण�बोनालू�के�साथ�तेलंगाना�का�राजक�य�उ�सव�घो�षत
�कया�गया�है।�

· बथूक�मा�के�प�ात�बोददे �मा�आता�है�जो�एक�7 �दवसीय�उ�सव�है।�

· बोददे �मा�उ�सव�जहाँ��थू�क��समा��त�का�संकेत�होता�है�वह��बथूक�मा�उ�सव�शरद�अथवा�शरथ��थू�का�आरंभ�माना�जाता�है।�

Source: https://www.thehindu.com/news/national/telangana/heritage-tag-for-two-
irrigation-facilities-in-telangana/article24910458.ece

QUESTION 64.
‘ अ�नकुट�’ संद�भ�त�करता�है-

a) वषा�जल�बंद�को�
b) सै�य�अ�धका�रय��को�अनुदान�म��द��जाने�वाली�भू�म�को�
c) भू�म�के��स�चाई�हेतु��योग��कए�जाने�वाले�जलच��को�
d) कृ�षयो�य�भू�म�म��बदल�द��गई�उ�खात�भू�म�को�
Correct Answer: A
Your Answer:
Explanation

Solution (a)

IASbaba
Score:
Web: http://ilp.iasbaba.com/
45.00 /
Email: ilp@iasbaba.com
Page 92 150
2019 - Test 9- History &
Exam Title :
Cu...
Email : vikasutsav@hotmail.com
Contact :

समाचार�:

· भारत�म��पहली�बार�एक��स�चाई�प�रयोजना�को�ICID �ारा�एक��वरासत�संरचना�के�तौर�पर�मा�यता�द��गई�है।�

· ICID ने�2 संरचना��नामत: तेलंगाना�म��गोदावरी�नद��पर��नम�ल��जले�के�सदाम�त�अ�नकुट�तथा�कामारे�ी��जले�के�पे�ा�चे�वु�को�मा�यता


द��है।�

· अ�नकुट�तेलुगु�भाषा�के�अना-क�ा�हेतु��यु��एक�अं�ेजी�श�द�है�का�अथ��है�वषा�जल�बंद।�

इंटरनेशनल�कमीशन�ऑन�इरीगेशन�एंड��ेनेज�(ICID)

· यह�एक�अ�णी�वै�ा�नक, तकनीक�, अंतरा��ीय�गैर-लाभकारी, गैर-सरकारी�संगठन�है।�

· यह��स�चाई, �ेनेज�तथा�बाढ़��बंधन�के��े��म��काय��करने�वाले��व��भर�के��वशेष���का�एक�पेशेवर�नेटवक��है।�

· इसका�मु�य�उ�े �य�सतत��ामीण��वकास�के�मा�यम�से�ऐसे�जल�सुर��त��व��हेतु�‘सतत�कृ�ष�जल��बंधन’ को�बढ़ावा�दे ना�है�जो�गरीबी�और


भूख�से�मु��हो।�

· ICID एक��ान�साझा�करने�का�मंच�है�जो�उन�मु���को�सम�प�त�है��जनके�अंतग�त�वषा��आधा�रत�कृ�ष�से�लेकर�पूरक��स�चाई, ल�ड��ेनेज,


डे�फ�सट�इरीगेशन�से�लेकर�फुल�इरीगेशन�आ�द�तक�जल��बंधन��वहार��का�एक�पूरा�दायरा�समा�हत�है।�

· इसके�अ�त�र�, कृ�ष�भू�मय��का��ेनेज�इस��मशन�क��ग�त�व�धय��क��क���य�थीम�है।�भावी�जलवायु�प�रवत�न�के�प�रणाम�व�प�होने�वाली
बाद�एवं�सुखा�जैसी�दो��ब�कुल�अलग�जलवायवीय�घटनाएं�भी�इसक��ग�त�व�धय��के�क���म��ह�गी।�

Source: https://www.thehindu.com/news/national/telangana/heritage-tag-for-two-
irrigation-facilities-in-telangana/article24910458.ece

QUESTION 65.
‘ हाट��समुदाय�’ �मुखतया�पाया�जाता�है-

a) �वदभ��म��
b) रायलसीमा�म��
c) �हमाचल��दे श�म��
d) ओ�डशा�म��
Correct Answer: C
Your Answer:
Explanation

Solution (c)

हाट��समुदाय�

समाचार�: हाट��समुदाय�को�जनजा�त�का�दजा���दए�जाने�क��माँग�

संबं�धत�त�य�

· हाट���वशेष��कार�के�लोग�ह��जो��हमाचल��दे श�के��गरी�पार��े���म���नवास�करते�ह�।�

· उनम��से�कुछ�उ�राखंड�म��ह��और�जनजा�त�के�दज��के�साथ-साथ�उसके�प�रणामी�लाभ��ा�त�कर�रहे�ह�।��हमाचल�म�, वे�लगातार�उपे��त
अनुभव�कर�रहे�ह�।�

· �सरमौर��जले�का��गरी-पार��े��उ�राखंड�के�पूव�वत��जौनसार�बावर��े��का�भाग�था�जहाँ�हाट��समुदाय�को�अनुसू�चत�जनजा�त�का�दजा��दे
�दया�गया�है।�

IASbaba
Score:
Web: http://ilp.iasbaba.com/
45.00 /
Email: ilp@iasbaba.com
Page 93 150
2019 - Test 9- History &
Exam Title :
Cu...
Email : vikasutsav@hotmail.com
Contact :

· दोन���े��शाही��सरमौर��रयासत�का�भाग�थे।�

· पशु�ब�ल, लोक�नृ�य�जैसी�स�दय��पुरानी�पर�पराएं�और�उनका��व�श��पहनावा�था�का�अथवा�लो�हया�न�केवल�उ�ह��एक�अलग�समुदाय
बनाते�ह�, ब��क�एक�ऐसा�समुदाय�भी��जसने�अभी�तक�आधु�नक�करण�नह��अपनाया�है।�

· ट�स�नद���ारा�अलग�होने�वाला��गरी-पार��े��नद��के�दोन��ओर�रहने�वाले�लोग��के�बीच�क��सामा�जक-आ�थ�क�और�सामा�जक-शै��क�खाई
का���त�न�ध�व�करता�है।�

Source: https://www.thehindu.com/todays-paper/tp-national/give-tribal-status-to-hatti-
community-hp-cm/article24912023.ece

QUESTION 66.
‘ वा�श�गटन�एकॉड��’ के�बारे�म���न�न�ल�खत�कथन��पर��वचार�क��जए।�

1. यह�उन��नकाय��के�बीच�एक�अंतरा��ीय�समझौता�है�जो��नातक�से�नीचे�के�इं�ज�नय�र�ग��ड�ी��ो�ा�स�को�मा�यता�दे ने�हेतु�उ�रदायी�ह�।�

2. भारत�का�नेशनल�बोड��ऑफ़�अ�ेडटै शन�2018 म��वा�श�गटन�एकॉड��का��थायी�सद�य�बन�गया�है।�

सही�कथन�चु�नए�

a) केवल�1
b) केवल�2
c) 1 और�2 दोन��
d) न�तो�1 न�ही�2
Correct Answer: A
Your Answer:
Explanation

Solution (a)

1989 म��मूल��प�से�छ: दे श��के�बीच�ह�ता��रत�वा�श�गटन�एकॉड��उन��नकाय��के�बीच�एक�अंतरा��ीय�समझौता�है�जो��नातक�से�नीचे�के


इं�ज�नय�र�ग��ड�ी��ो�ा�स�को�मा�यता�दे ने�हेतु�उ�रदायी�ह�।�यह�इन��नकाय���ारा���या�यत��कए�गए��ो�ा�स�क��ता��वक�समानता�को�मा�यता
दे ता�है�तथा�अनुशंसा�करता�है��क�ह�ता�र�करने�वाले��कसी�भी��नकाय��ारा���या�यत�इन��ो�ा�स�के��नातक��को�अ�य��नकाय���ारा�इस
�कार�मा�यता�द��जाए��क�जैसे�उ�ह�ने�उनके��याया�धकार��े��म��इं�ज�नय�र�ग�क���ै��टस�हेतु��वेश�क��आव�कताएं�पूरी�कर�ली�ह�।�

वा�श�गटन�एकॉड��क��सद�यता�सद�य�दे श��ारा���ता�वत�क��जा�रही�अंडर�ेजुएट�इंजी�नय�र�ग��श�ा�क��गुणव�ा�क��अंतरा��ीय�मा�यता�है�तथा
इसे��व���तरीय��ेणी�म��लाने�का�एक��थान�है।�यह�अंतरा��ीय��तर�पर�इंजी�नय�र�ग��नातक��और�पेशेवर��क��मो�ब�लट��को��ो�सा�हत�करता
और�सुगम�बनाता�है।�

भारत�का�नेशनल�बोड��ऑफ़�अ�ेडटै शन�13 जून�2014 को�वा�श�गटन�एकॉड��का��थायी�सद�य�बन�गया�है।�

�टयर�-1 सं�थान��ारा���ता�वत��कए�जा�रहे�NBA ��या�यत��ो�ा�स�वा�श�गटन�एकॉड��के�अ�य�ह�ता�रकता����ारा�मा�यता�हेतु�पा��ह�।�

सो�चए�!

· व�ड��स�मट�ऑन�अ�ेडटै शन�(WOSA)

Source: http://pib.nic.in/PressReleseDetail.aspx?PRID=1545431

QUESTION 67.
‘ म�ल�न�ट�कटारल�फ�वर�(MCF)’ �न�न�ल�खत�म��से��कससे�संबं�धत�है�?

a) �लेमी�डया�बै�ट��रया�

IASbaba
Score:
Web: http://ilp.iasbaba.com/
45.00 /
Email: ilp@iasbaba.com
Page 94 150
2019 - Test 9- History &
Exam Title :
Cu...
Email : vikasutsav@hotmail.com
Contact :

b) हप�स�वायरस�
c) गोनो�रया�बै�ट��रया�
d) �सफ�लस�बै�ट��रया�
Correct Answer: B
Your Answer:
Explanation

Solution (b)

म�ल�न�ट�कटारल�फ�वर�(MCF)

· यह�सामा�यत: पशु��तथा�आ�ट�योडै��टला�क��अ�य��जा�तय��का�घातक�रोग�है�जो�जीनस�माकावायरस�के�चय�नत�हप�सवायरस�के
सं�मण�के�कारण�होता�है।�

· यह�एक�सं�ामक��वषाणु�रोग�है��जसका�मूल�द��ण�अ��का�है�तथा�जो�मवे�शय��को��भा�वत�करता�है।�

· सबसे�पहले�इसक��पहचान�10 वष��पूव��कना�टक�म��क��गई�थी।�

· इसके��सार�को�रोकने�हेतु�न�तो�कोई�उपचार�है�और�न�ही�कोई�ट�का।�

· यह�एक�हप�स��वषाणु�है�जो�भेड़�से�मवेशी�म��संचा�रत�होता�है।�एक�बार�जब�यह�वार�करता�है, मवेशी�बमु��कल�छ: �दन�जी�वत�रह�पाता�है।

· इस�रोग�को�गोवंशीय�उपप�रवार�तथा�मृग�कुल�प�रवार�क���जा�तय��को�सं��मत�करने�हेतु�जाना�जाता�है, �क�तु�यह�घरेलु�सुअर�, �जराफ


तथा��ांगेला�फना�उपप�रवार�से�संबंध�रखने�वाली�एंट�लोप�क���जा�तय��म��भी�पहचानी�जाती�है।�

Source: https://www.thehindu.com/todays-paper/tp-national/tp-karnataka/catarrhal-
fever-sheep-screened-in-tumakuru/article24939222.ece

QUESTION 68.
‘ �मले�स�-18’ एक�सै�य�अ�यास�है�जो��न�न�ल�खत�म��से��कस�ब�प�ीय�संगठन�से�संबं�धत�है�?

a) शंघाई�सहयोग�संगठन�
b) आग�नाईजेशन�ऑफ़�इ�ला�मक�कोऑपरेशन�
c) बे�ऑफ�बंगाल�इनी�शए�टव�फॉर�म�ट��से�टोरल�टे ��नकल�ए�ड�इकोनॉ�मक�को-ऑपरेशन�
d) को��ेह��सवे�एंड��ो�े�सव�ए�ीम�ट�फॉर��ांस-पै�स�फक�पाट� नर�शप�
Correct Answer: C
Your Answer:
Explanation

Solution (c)

पहला��ब�सटे क�सै�य�अ�यास�- �मले�स�-18

· यह�पुणे�म��शु���आ।�

· नेपाल�और�थाईल�ड�ने�अपने�पय�वे�क�भेजे�जब�क�अ�य�दे श�पूण��भागीदार�थे।�

Source: https://www.thehindu.com/news/national/bimstec-embarrassment-for-india/
article24928750.ece

QUESTION 69.
�न�न�ल�खत�संयु��सै�य�अ�यास��और�भागीदार�दे श��पर��वचार�क��जए-

1. ग�ड़�श���– भारत�तथा��ीलंका�

IASbaba
Score:
Web: http://ilp.iasbaba.com/
45.00 /
Email: ilp@iasbaba.com
Page 95 150
2019 - Test 9- History &
Exam Title :
Cu...
Email : vikasutsav@hotmail.com
Contact :

2. यु��अ�यास�– भारत�तथा�नेपाल�

3. नोमे�डक�एलीफ�ट�– भारत�तथा�भूटान�

�न�न�ल�खत�म��से�कौन�गलत�सुमे�लत�है�?

a) 1 और�2
b) 2 और�3
c) 1 और�3
d) उपयु���सभी�
Correct Answer: D
Your Answer:
Explanation

Solution (d)

· ग�ड़�श���:: भारत�तथा�इंडोने�शया�

· यु��अ�यास�- भारत�तथा�अमे�रका�

· नोमे�डक�एलीफ�ट�- भारत�तथा�मंगो�लया�

QUESTION 70.
�न�न�ल�खत�कथन��पर��वचार�क��जए�तथा�जनजा�त�क��पहचान�क��जए-

1. यह���पु�रस�के�प�ात���पुरा�क���सरी�सबसे�अ�धक�आबाद��वाली�जनजा�त�है।�

2. वे�कोकबोरोक�भाषा�नामक�बोली�बोलते�ह���जसका�उदगम��त�बत-बमा��से�है।�

सही�कूट�चु�नए�:

a) कापू�
b) धनगर�
c) �ू�
d) ग�ड�
Correct Answer: C
Your Answer:
Explanation

Solution (c)

�ू�/ �रयांग�

· ये���पुरा�रा�य�क��21 अ�धसू�चत�जनजा�तय��म��से�एक�ह�।�

· �ू�लोग�सारे���पुरा�म��पाए�जाते�ह�।�हालां�क�वे��मजोरम, असम, म�णपुर�म��भी��मलते�ह�।�

· वे�कोकबोरोक�भाषा�क���रयांग�बोली�बोलते�ह��जो��जसका�उदगम��त�बत-बमा��से�है�तथा��जसे��थानीय��प�से�काउ��ू�कहा�जाता�है।�

· यह���पु�रस�के�प�ात���पुरा�क���सरी�सबसे�अ�धक�आबाद��वाली�जनजा�त�है।�

· �रयांग�लोग��क��अ�धकांश�सं�या��ह���धम��क��वै�णव�शाखा�से�संबं�धत�है�और�वे��वयं�को����य�मानते�ह�।�ईसाईय��क��एक�बढ़ती�सं�या
��पुरा�और��मजोरम�दोन��म���नवास�करती�है।�

IASbaba
Score:
Web: http://ilp.iasbaba.com/
45.00 /
Email: ilp@iasbaba.com
Page 96 150
2019 - Test 9- History &
Exam Title :
Cu...
Email : vikasutsav@hotmail.com
Contact :

· वे�ब�दे ववाद��ह��और�एक�से�अ�धक�दे �वय��और�दे वता��म���व�ास�करते�ह�।�

�या�आप�जानते�ह��?

· 1997 म��एक�न�लीय�संघष��ने��ू�जनजा�त�के�हजार��लोग��को��मजोरम�म��अपने�घर�छोड़ने�पर��ववश�कर��दया�और�तबसे, �व�था�पत��ू


लोग�उ�री���पुरा�म��कंचनपुर�नाम�क�बे�(�मजोरम-��पुरा�सीमा�पर���थत) म��शरण��लए��ए�ह�।।�

· अब, वे�जमुई�पहा�ड़य��(जो���पुरा�को��मजोरम�तथा�बां�लादे श�से�अलग�करती�ह�) पर���थत�सात�शरणाथ��कै�प��म��फैल��ह�।�

· क��, ��पुरा�और��मजोरम�के�बीच�एक���प�ीय�समझौते�पर�ह�ता�र�होने�के�प�ात�उनका��मजोरम�म��पुन�था�पन�होना�तय�है।�

Source: https://www.thehindu.com/news/national/other-states/return-to-mizoram-or-
give-up-sops-centre-to-brus/article24930313.ece

QUESTION 71.
‘ रेड-�ब�ड�पाकु�’, जो��क�एक��वदे शी��जा�त�है, केरल�के��था�नक�जलीय�पा�रतं��और�जैव-�व�वधता�के��लए�एक�खतरा�है।�यह��था�नक�है-

a) अमेज़न��ोणी�हेतु�
b) कांगो��ोणी�हेतु�
c) �म�स�सपी��ोणी�हेतु�
d) नील��ोणी�हेतु�
Correct Answer: A
Your Answer:
Explanation

Solution (a)

बाढ़�के�प�ात�केरल�म��मछली�क��एक��वदे शी��जा�त�

समाचार�:

· �कसी�जल��नकाय�म��मछली�क���वदे शी��जा�त��था�नक�जलीय�पा�रतं��और�जैव-�व�वधता�के��लए�एक�खतरा�होती�है।�

· वै��क��प�से�बाढ़�के�जल�आ�ामक��जा�तय��के��सार�का�एक��मुख�माग���स���ए�ह��और�इस��कार�बाढ़�के�मैदान��के�साथ-साथ�भावी
आ�ामक��जा�तय��क��कृ�ष�करते�समय�एक�सावधानी�भरा����कोण�आव�यक�है।�

रेड-�ब�ड�पाकु�(Piaractus brachypomus)

· यह�द��ण�अमे�रका�म����थत�अमेज़न�तथा�ओ�रनोको��ोणी�क��दे शज��जा�त�है।�

· यह�सवा�हारी�है�तथा�फल�, बीज��और�मेव��से�लेकर�क�ड़�, छोट��मछ�लय�, खोल�वाले�जीव��(�सटे �शयन) तथा�पाद�लवक��सभी�को�खा


जाती�है।�

अरपाइमा�( अरपाइमा��गगस�)

· यह�द��ण�अमे�रका�के�अमेज़न�तथा�ए�सेकुइबो�बे�सन�हेतु��था�नक�है।�

· यह�लंबाई�म��4.5 मीटर�तथा�शरीर�के�वजन�म��200 �क�ा. तक�होती�है।�

· यह�भयानक��प�से�मछ�लय��का�भ�ण�करती�है�और�इस��कार�केरल�क��न�दय��म���मलने�वाली��वदे शी�मछ�लय��पर�गंभीर�खतरा�बन
सकती�है।�

एलीगेटर�गर�(Atractosteus spatula)

· यह�अमे�रका�और�मे��सको�का�दे शज�है।�

IASbaba
Score:
Web: http://ilp.iasbaba.com/
45.00 /
Email: ilp@iasbaba.com
Page 97 150
2019 - Test 9- History &
Exam Title :
Cu...
Email : vikasutsav@hotmail.com
Contact :

· इसक��लंबाई�3 मीटर�तक�तथा�वजन�लगभग�137 �क�ा. तक��मलता�है।�

Source: https://www.thehindu.com/news/national/kerala/an-alien-invasion-post-floods-
in-state-waters/article24929272.ece

QUESTION 72.
�न�न�ल�खत�म��से�कौन�एक�हाल�ही�म��सु�ख़�य��म��रहे�‘VC 11184’ क��सव��म��ा�या�है�?

a) बै�ल��टक��मसाइल��ै �क�ग��शप�
b) �यू��लयर-पावड��पनडु �बी�
c) टारपीडो�लॉ�च�एंड��रकवरी�वेसल�
d) �यू��लयर-पावड��एयर�ा�ट�कै�रयर�
Correct Answer: A
Your Answer:
Explanation

Solution (a)

भारत�का�पहला��मसाइल��ै �क�ग��शप�(VC 11184)

· इसे��ह���तान��शपयाड���ल�मटे ड�(HSL) �ारा�बनाया�गया�है।�

· यह�भारत�का�पहला�बै�ल��टक��मसाइल��ै �क�ग��शप�होगा।�

· यह�नेशनल�टे ��नकल��रसच��आग�नाईजेशन�हेतु��न�म�त��कया�जा�रहा�है�जो��धानमं�ी�काया�लय�तथा�रा�ीय�सुर�ा�सलाहकार�के�पय�वे�ण�म�
काय��करने�वाली�टे ��नकल�इंटे�लज�स�एज�सी�है।�

· इसका�नाम�नौसेना�म��इसके�शा�मल�होने�के�प�ात�रखा�जाएगा।�अब�के��लए, इसे�VC 11184 से�संद�भ�त��कया�जाता�है।�

· इससे�भारत�चार�दे श��(अमे�रका, �स, चीन�तथा��ांस) के�इलीट��लब�म��शा�मल�हो�जाएगा।�

· इसे�दो�9000 KW डीजल�इंजनो��ारा��णो�दत��कया�जाएगा।�

· यह�21 नॉट् स�क��औसत�ग�त�से�गमन�कर�सकता�है।�

सो�चए�!

· नेशनल�टे ��नकल��रसच��आग�नाईजेशन�(NTRO)

Source: https://www.thehindu.com/sci-tech/science/indias-first-missile-tracking-ship-is-
readying-for-sea-trials/article24929012.ece

QUESTION 73.
�न�न�ल�खत�म��से�कौन�सा/से�कथन�‘ इरी��या�’ के�बारे�म��स�य�है/ह��?

a) यह�लाल�सागर�क��ओर�खुलता�है।�
b) इसक��सीमा�केवल�इ�थयो�पया�और�सूडान�से�लगती�है।�
c) (a) और�(b) दोन��
d) न�तो�(a) न�ही�(b)
Correct Answer: A
Your Answer:
Explanation

IASbaba
Score:
Web: http://ilp.iasbaba.com/
45.00 /
Email: ilp@iasbaba.com
Page 98 150
2019 - Test 9- History &
Exam Title :
Cu...
Email : vikasutsav@hotmail.com
Contact :

Solution (a)

इरी��या�हॉन��ऑफ़�अ��का�म����थत�एक�दे श�है, �जसक��राजधानी�अ�मारा�है।�इसक��सीमा�प��म�म��सूडान, द��ण�म��इ�थयो�पया�तथा


द��ण�पूव��म���जबूती�से�लगती�है।�इरी��या�के�पूव��र�के�और�पूव��भाग��क��लाल�सागर�के�साथ��व�तृत�सीमा�है।�

QUESTION 74.
�धानमं�ी�अ�दाता�आय�संर�ण�अ�भयान�(PM-AASHA) के��वषय�म���न�न�ल�खत�कथन��पर��वचार�क��जए।�

1. PM-AASHA के��ाथ�मक�उ�े �य��म��फ��ड��तर�पर��स�चाई��णाली�म���नवेश�को�आक�ष�त�करना, दे श�म��जोती�जाने�वाली�भू�म�को


�वक�सत�करना�और�उसका��व�तार�करना, जल�का�अप�य��यूनतम�करने�के��लए�खेत�के�जल�का�उपयोग�बढ़ाना, जल-बचत��ौ�ो�गक�य�
तथा��ी�स�न��स�चाई�को�लागू�कर���त�बूँद�अ�धक�फसल��ा�त�करना�शा�मल�ह�।�

2. यह�एक�अ��ेला�योजना�है��जसम��वत�मान�म��चल�रही�मं�ालय�क��योजनाएं�जैसे�मेगा�फ़ूड�पाक�, एक�कृत�शीतगृह��ृंखला�और�मू�य�वध�न
अवसंरचना, फ़ूड�से�ट��एंड��वा�लट��ए�योर�स�इं�ा���चर�आ�द�समा�हत�हो�गई�ह�।�

3. यह�सु�न��त�करती�है��क��तलहन, दाल�और�कोपरा�उगाने�वाले��कसान��को�असल�म����त�वष��वादे �अनुसार��यूनतम�समथ�न�मू�य�(MSP)


�ा�त�हो।�

सही�कथन�चु�नए�

a) 1 और�2
b) 1 और�3
c) केवल�3
d) 2 और�3
Correct Answer: C
Your Answer:
Explanation

Solution (c)

IASbaba
Score:
Web: http://ilp.iasbaba.com/
45.00 /
Email: ilp@iasbaba.com
Page 99 150
2019 - Test 9- History &
Exam Title :
Cu...
Email : vikasutsav@hotmail.com
Contact :

�धानमं�ी�अ�दाता�आय�संर�ण�अ�भयान�(PM-AASHA)

· इसका�ल�य��कसान��को�क���य�बजट�2018 क��घोषणा�अनुसार�उनक��फसल��हेतु�लाभकारी�मू�य�दे ना�सु�न��त�करना�है।�

· यह�सु�न��त�करती�है��क��तलहन, दाल�और�कोपरा�उगाने�वाले��कसान��को�असल�म����त�वष��वादे �अनुसार��यूनतम�समथ�न�मू�य�(MSP)


�ा�त�हो।�

· इस�योजना�के���या�वयन�हेतु�अगले�दो�वष��म��₹15,053 करोड़��पए��य�करने�के�अ�त�र��कै�बनेट�ने�खरीद�करने�वाली�एज��सय��क��
₹16,550 करोड़��पए�क��अ�त�र��सरकारी�ऋण�गारंट��का�भी�अनुमोदन��कया।�

इसम���कसान��हेतु�लाभकारी�मू�य��को�सु�न��त�करने�वाला�तं��स��म�लत�है�तथा�इसके�तीन�घटक/उप-योजनाएं�ह��-

· मू�य�समथ�न�योजना�(PSS)

· मू�य�कमी�भुगतान�योजना�(PDPS)

· पायलट�ऑफ़��ाइवेट��ो�योरम�ट�एंड��टॉ�क�ट��क�म�(PPPS).

खरीद�ग�त�व�ध�म���नजी��े��

· यह�तय��कया�गया�है��क��तलहन��के��लए�रा�य��के�पास�चय�नत��जल��/ �नजी��टॉ�क�ट�क��भागीदारी�वाले��जले�क��APMC(s) म�
पायलट�आधार�पर��ाइवेट��ो�योरम�ट�एंड��टॉ�क�ट��क�म�(PPSS) लाने�का��वक�प�होगा।�

· पायलट��जला/�जले�क��चय�नत�APMC(s) �तलहन�क��उस�एक�या�अ�धक�फसल�को�करेगा��जसके��लए�MSP अ�धसू�चत�क��गई�है।�

· चूँ�क�यह�इस�लए�PSS जैसी�है��य��क�इसम��अ�धसू�चत�व�तु�का�भौ�तक��य�शा�मल�है, अत: यह�पायलट��जल��म��PSS/PDPS का


�थान�लेगी।�

PSS

· मू�य�समथ�न�योजना�(PSS) म��दाल�, �तलहन��तथा�कोपरा�क��भौ�तक�खरीद�क���य�नोडल�एज��सय���ारा�क��जाएगी��जसम��रा�य


सरकार��क��अ�स��य�भू�मका�होगी।�

· यह�भी�तय��कया�गया�है��क�NAFED के�अ�त�र��भारत�खा���नगम�भी�रा�य�/�जल��म��PSS काय�वा�हयाँ�करेगा।�

· �स�ांत�के�अनुसार�खरीद��य�तथा�खरीद�के�चलते�होने�वाली���तयां�क���सरकार��ारा�वहन�क��जाएंगी।�

PDPS

· मू�य�कमी�भुगतान�योजना�(PDPS) के�अंतग�त�उन�सभी��तलहन��को�कवर�करने�का���ताव�है��जनके��लए�MSP अ�धसू�चत�क��गई�है।�

· MSP तथा��व�य/मोडल�मू�य�के�बीच�के�अंतर�के�इस���य��भुगतान�म��उन�पूव�-पंजीकृत��कसान��को�भुगतान��कया�जाएगा�जो�पारदश�
नीलामी����या�के�मा�यम�से�अ�धसू�चत�बाजार��े��म��अपनी�उपज�बेच�गे।�

· सभी�भुगतान���य�त: �कसान�के�पंजीकृत�ब�क�खाते�म���कए�जाएंगे।�

· इस�योजना�म��फसल��क��भौ�तक�खरीद�शा�मल�नह��है��य��क��कसान��को�अ�धसू�चत�बाजार�म��आने�पर�MSP मू�य�तथा��व�य�/ मोडल


मू�य�के�बीच�के�अंतर�का�भुगतान��कया�जाता�है।�

· PDPS के��लए�क���सरकार�का�समथ�न��नयामानुसार��दया�जाएगा।�

· इसे��तलहन��कसान��को�सुर�ा�दे ने�हेतु�म�य��दे श�सरकार�क��भावांतर�भुगतान�योजना�(BBY) क��तज��पर�लाया�गया�है।�

�या�आप�जानते�ह��?

IASbaba
Score:
Web: http://ilp.iasbaba.com/
45.00 /
Email: ilp@iasbaba.com
Page 100 150
2019 - Test 9- History &
Exam Title :
Cu...
Email : vikasutsav@hotmail.com
Contact :

· धान, गे�ं�और��यू��-सी�रय�स/मोटे �अनाज�क��खरीद�हेतु�खा��और�साव�ज�नक��वतरण��वभाग�क��अ�य��व�मान�योजनाएं�तथा�कपास�तथा


जूट�हेतु�कपड़ा�मं�ालय�क��योजनाएं�इन�फसल��हेतु��कसान��को�MSP �दान�करने�हेतु�जारी�रह�गी।�

· जब�कभी�बाजार�म��क�मत��अ�चसु�चत�MSP से�नीचे��गरती�ह��तथा�जब�कभी�भी�बाजार�म���वेश�हेतु�रा�य/संघ�शा�सत��े��क��सरकार
�ारा�अ�धकृत��कया�जाता�है, चय�नत��नजी�एज�सी��ाइवेट��ो�योरम�ट��टॉ�क�ट��क�म��दशा�नद� श��के�अनु�प�पंजीकृत��कसान��से
अ�धसू�चत�अव�ध�के�दौरान�अ�धसू�चत�बाजार�म��MSP पर�व�तु�क��खरीद�करेगी�; अ�धसू�चत�MSP का�अ�धकतम�15 ��तशत�सेवा
शु�क�दे य�होगा�।

· भारत�खा���नगम�जो��क�अनाज��क��खरीद�और��वतरण�हेतु�सरकार�क��नोडल�एज�सी�है�पहले�से�ही�MSP पर�गे�ं�और�चावल�क��खरीद
करती�है�ता�क�राशन��कान��तथा�क�याण�योजना��के�मा�यम�से�उनक��आपू�त��क��जा�सके।�

Source: https://www.thehindu.com/todays-paper/new-procurement-policy-cleared/
article24939202.ece

QUESTION 75.
‘ �लोबल�बड�न�ऑफ़��डजीज��टडी�(GBD)’ रोग�भार�का�एक��ापक��े�ीय�और�वै��क�शोध�काय��म�है�जो��मुख�रोग�, चोट��और
जो�खम�कारक��से�होने�वाली�मृ�यता�का�आकलन�करता�है।�यह�कहाँ���थत�है�

a) मे�ड�स�स�स�स��ं�टयस��
b) इं��ट�ूट�फॉर�हे�थ�मै���स�एंड�इवै�यूएशन�
c) �व���वा��य�संगठन�
d) लांसेट��लोबल�हे�थ�
Correct Answer: B
Your Answer:
Explanation

Solution (b)

GBD वा�श�गटन��व��व�ालय�म��इं��ट�ूट�फॉर�हे�थ�मै���स�एंड�इवै�यूएशन�(IHME) म����थत�है�और�इसका��व�पोषण��बल�एंड�मे�ल�डा


गेट्स�फाउंडेशन��ारा��कया�जाता�है।�

‘ इं�डया��टे ट-लेवल��डजीज�बड�न�इ�न�शए�टव�’ �रपोट� �

· यह�भारतीय��च�क�सा�शोध�प�रषद�(ICMR), प��लक�हे�थ�फाउंडेशन�ऑफ़�इं�डया�(PHFI) और�इं��ट�ूट�फॉर�हे�थ�मै���स�एंड


इवै�यूएशन�(IHME) क���वा��य�एवं�प�रवार�क�याण�मं�ालय�के�साथ�संयु��पहल�है��जसम��100 से�अ�धक�भारतीय�सं�थान��के��वशेष�
और��हतधारक�संब��ह�गे।�

· यह�अ�ययन��लोबल�बड�न�ऑफ़��डजीज��टडी�के�भाग�के��प�म��1990 से�2016 के�बीच�भारत�से��ा�त�सभी�पहचान�यो�य


ए�पडे�मओलो�जकल�डेटा�के��व�े�ण�पर�आधा�रत�है।�

मह�वपूण���ब���

· भारतीय��म��1990 से�2016 के�बीच�इ�चे�मक�हाट� �रोग��और���ोक�म��50% वृ���दज��क��गई�है।�

· मधुमेह�के�मामले�26 �म�लयन�से�बढ़कर�65 �म�लयन�हो�गए�ह�।�

· बार-बार�होने�वाली���तरोधी�फेफड़��क��बीमारी�से���त�लोग��क��सं�या�28 �म�लयन�से�बढकर�55 �म�लयन�हो�गई�है।�

· इ�चे�मक�हाट� �रोग��के�मामले�म��पंजाब�सबसे�ऊपर�बना��आ�है, इसके�बाद�त�मलनाडु �का��थान�है।�मधुमेह�के�मामले�म��यह��म�उ�टा�है।�

· प. बंगाल���ोक�के�मामल��म��सव��च��थान�पर�बना��आ�है�तथा�इसके�बाद�ओ�डशा�का��थान�है।�

· क�सर�के�मामल��म��केरल�सव��च��थान�पर�है��जसके�बाद�असम�का��थान�है।�

IASbaba
Score:
Web: http://ilp.iasbaba.com/
45.00 /
Email: ilp@iasbaba.com
Page 101 150
2019 - Test 9- History &
Exam Title :
Cu...
Email : vikasutsav@hotmail.com
Contact :

· इ�चे�मक�हाट� �रोग��और�मधुमेह�के�मामल��म��सवा��धक�वृ���दर�भारत�के�कम��वक�सत�रा�य��म��पाई�गई�है।�

· भारत�म��क�सर�से�रोग�भार�का�आनुपा�तक�योगदान�1990 के�बाद�से�दोगुना�हो�गया�है��क�तु�इकाई�क�सर�के�मामले�रा�य��के�बीच��भ�-�भ�
ह�।�

आ�मह�याएं�

· भारत�म��आ�मह�या�15-39 आयु�समूह�के�लोग��के�बीच�मृ�यु�का�एक�सव��मुख�कारण�है।�

· �व��भर�म��म�हला��म��होने�वाली�इस��कार�क��मृ�यु��का�सैत�स���तशत�भारत�म��पाया�जाता�है।�

· भारत�म��आ�मह�या�मृ�यु�दर�(SDR) इसके�सामा�जक-जनां�कक�य�सूचकांक��तर�(�वशेषकर�म�हला��म�) के��हसाब�से�आशा�से�कह�


अ�धक�है�तथा�पु�ष��से�म�हला��के�अनुपात�म��रा�य�दर�रा�य�काफ��अंतर�है।�

Source: https://www.thehindu.com/sci-tech/health/indias-health-report-reads-worse/
article24937845.ece

IASbaba
Score:
Web: http://ilp.iasbaba.com/
45.00 /
Email: ilp@iasbaba.com
Page 102 150

También podría gustarte